Test Bank For Introduction To Clinical Pharmacology 10th Edition by Visovsky Watermark

You might also like

Download as pdf or txt
Download as pdf or txt
You are on page 1of 250

TEST BANK FOR

INTRODUCTION TO
CLINICAL
PHARMACOLOGY
10TH EDITION
BY VISOVSKY

Authentic Nurse Guru: For more of these materials :masterclasstutor1@gmail.com


TEST BANK FOR INTRODUCTION TO
CLINICAL PHARMACOLOGY 10TH EDITION
BY VISOVSKY

Chapter 01: Pharmacology and the Nursing Process in LPN Practice


Visovsky:Introduction to Clinical Pharmacology, 10th Edition

MULTIPLE CHOICE

1. The LPN is collecting data for the initial assessment of a patient upon admission to a
Long-term care facility before giving the patient’s prescribed drugs. Which action should the LPN
consider to be the highest priority?

a. Obtain any special equipment that will be needed to give the patient’s drug.

b. Monitor the patient for a response to the drug given.

c. Collect data about the patient and the patient’s health condition.

d. Review the nursing care plan to verify that it is accurate.


ANS: C
Collecting and documenting data about the patient and the patient’s health condition is a critical
step before any drugs are given. Information regarding the present illness, any signs and
symptoms, review of medical records, drug history, and vital signs are needed before drugs are
given. Deciding on special equipment that will be needed to give the patient’s drug is part of the
planning phase of the nursing process. Monitoring the patient for his response to given drug is
part of the evaluation stage of the nursing process. Reviewing the nursing care plan to verify that
it is being followed accurately is part of the implementation stage of the nursing process.

DIF: Cognitive Level: Applying REF: p. 2

Authentic Nurse Guru: For more of these materials :masterclasstutor1@gmail.com


2. The LPN is working with a patient in the planning stage of the nursing process related to
the patient’s prescribed drugs. Which action should the LPN take during this stage?

a. Develop a nursing goal to plan the procedures needed to give drug.

b. Develop a teaching plan for the patient regarding the drug’s actions.

Authentic Nurse Guru: For more of these materials :masterclasstutor1@gmail.com


c. Determine that the patient is experiencing the expected response to his drug.

d. Determine how much the patient understands about his drug.


ANS: D

Determining how much the patient understands about his drug is part of the diagnosis phase of
the nursing process. Developing a nursing goal to plan the procedures needed to give drug and
developing a teaching plan for the patient regarding the drug’s actions are part of the planning
phase of the nursing process.

DIF: Cognitive Level: Applying REF: p. 2

3. You are teaching a patient with depression about the potential adverse effects of a
prescribed drug. What part of the nursing process related to drug therapy are you engaging in at
this point of the teaching plan?

a. Assessment

b. Implementation

c. Evaluation

d. Diagnosis
ANS: C
In the evaluation phase of the nursing process, the LPN understands and teaches to the patient
the drug’s therapeutic effects, expected side effects, and potential adverse effects.

DIF: Cognitive Level: Remembering REF: p. 2

4. Which of the following is an example of subjective data?

a. The patient states she has pain in her left arm.

b. The medical chart has a recorded blood pressure of 128/88.

c. The serum potassium level is 3.8 mmol/L.

Authentic
d. Nurse
The Guru:
patient’s ECGFor more
shows of these
normal materials :masterclasstutor1@gmail.com
sinus rhythm.
ANS: A
Reports from the patient or patient’s caregiver are considered subjective data. Symptoms such as
pain, nausea, or dizziness are examples of symptoms that cannot be “seen” and are data collected
from the patient, caregiver, or others. Laboratory values, ECG results, or vital sign data from a
medical chart are examples of objective data.

DIF: Cognitive Level: Remembering REF: p. 2

5. Which statement provides an example of objective data?

a. The wife states the patient was confused last night.

b. Grimacing with movement is present during the examination.

c. The patient reports moderate alcohol consumption.

d. The patient states pain is severe.


ANS: B
Measurable data obtained during a physical exam such as grimacing with movement is an
example of objective data. Subjective data includes information presented by the patient or
Family that cannot be substantiated such as a wife’s report of a patient’s confusion, patient report
of degree of alcohol consumption, and a patient’s pain rating.

DIF: Cognitive Level: Remembering REF: p. 3

6. The LPN/VN is assessing a patient before giving a drug for blood pressure management.
The nurse notes the blood pressure to be 90/50 mm Hg. What is the nurse’s best action?

a. Hold the drug and report the blood pressure to the RN.

b. Give the patient a full glass of water before giving the drug.

c. Come back in 30 minutes and recheck the blood pressure.

Authentic Nurse Guru: For more of these materials :masterclasstutor1@gmail.com


d. Have the patient perform pursed lip breathing before giving the drug.
ANS: A
The best action is to hold the drug and contact the RN. The patient may need an adjustment to
the dose of the blood pressure drug or switching to another drug. Giving water with the drug is
not contraindicated but does not recognize the patient’s risk for hypotension. Pursed lip breathing
has no role in this situation.

DIF: Cognitive Level: Remembering REF: p. 4

7. The LPN is collecting objective data for inclusion in the nursing assessment. Which piece
of information indicates that the LPN has a clear understanding of objective assessment data?

a. A patient’s rating of chest pain as 8 on a 1 to 10 scale.

b. Family members report that patient has been experiencing pain for 1 month.

c. Detailed history of the patient’s current illness upon admission.

d. Compilation of past laboratory results and x-ray reports.


ANS: D
The patient’s past laboratory and x-ray results are examples of objective data. A pain rating of
8/10, a family member’s description of the patient’s pain, and history of current illness are
examples of subjective data.

DIF: Cognitive Level: Remembering REF: p. 3

8. A patient recently began a taking blood pressure drug and presents for a follow-up
appointment. The office nurse reviews the patient’s daily blood pressure recordings. Which stage
of the nursing process corresponds to this review?

a. Assessment
Authentic Nurse Guru: For more of these materials :masterclasstutor1@gmail.com
b. Planning

c. Diagnosis

d. Evaluation
ANS: D
The evaluation phase involves examining the results that occur when the plan is implemented.
Reviewing the patient’s daily blood pressure recording examines the patient’s response to the
drug. The assessment phase provides initial information about the patient, the problem, and
anything that may change the choice of treatment. The planning phase involves using patient
assessment data and diagnoses to set goals and write care plans. The diagnosis phase involves
decision-making about the patient’s problems, including medical diagnoses made by the
healthcare provider and nursing diagnoses developed through the North American Nursing
Diagnosis Association (NANDA).

DIF: Cognitive Level: Remembering REF: p. 2

9. After receiving report, the LPN gives drugs to her assigned patients on the evening shift.
With which stage of the nursing process does this activity correspond?

a. Implementation

b. Assessment

c. Planning

d. Diagnosis
ANS: A
The implementation phase involves actively following the plan of care and accurately giving
ordered drug to the patients. The assessment phase involves obtaining initial information about
the patient, the problem, and anything that may change the choice of treatment. The planning
phase involves using patient assessment data and diagnoses to set goals and write care plans. The
diagnosis phase involves decision-making about the patient’s problems, including medical
diagnoses made by the healthcare provider and nursing diagnoses developed through the North
Authentic Nurse Guru: For more of these materials :masterclasstutor1@gmail.com
American Nursing Diagnosis Association (NANDA).

DIF: Cognitive Level: Remembering REF: p. 5

10. You are reviewing a patient’s new antihypertensive drug order. The order as written is
unclear as to the number of times per day the drug is to be given. What is your best action?

a. Call the healthcare provider to clarify the order.

b. Refer the question to the hospital pharmacy.

c. Give the drug according to the information in a drug handbook.

d. Hold the drug until the healthcare provider returns the following day.
ANS: A
Your responsibility as a nurse giving drugs is to apply knowledge about the specific drug and
drug orders. No part of the drug order should be unclear. Any questions related to the drug, dose
or appropriateness for the specific patient should be answered before the drug is given.

DIF: Cognitive Level: Understanding REF: p. 4

11. A patient is receiving an antibiotic for pneumonia. On the third day of the treatment
regimen, a rash appears on her chest, and she reports itching and shortness of breath. Which term
describes the effect that has occurred?

a. Therapeutic effect

b. Adverse effect

c. Side effect

d. Overdose effect
ANS: B
An itchy rash with shortness of breath that develops in response to drug is an example of an
allergic reaction or adverse effect to the antibiotic. Therapeutic effects occur when an antibiotic
fights infection without causing any adverse effects. Side effects of drugs are known potential
Authentic Nurse Guru: For more of these materials :masterclasstutor1@gmail.com
effects of the antibiotic that range from mild to moderate. An overdose occurs if a patient
receives too much of a drug.

DIF: Cognitive Level: Understanding REF: p. 9

12. An LPN enters a patient’s room to give a scheduled drug. Before administration, the
patient states, “I can’t take that drug; I’m allergic to it.” What should the nurse do first?

a. Reassure the patient that the drug is needed and observations regarding possible allergic
symptoms will be made.

b. Review the patient record and encourage the patient to take the drug if no allergies have
been documented.

c. Assess the patient’s allergic history and notify the healthcare provider to determine a
course of action.

d. Document patient refusal and leave a note on the patient chart for the healthcare provider.
ANS: C
The patient has shared information that indicates the potential for the ordered drug to cause
adverse effects. Before giving the drug, the nurse should investigate further by obtaining a more
detailed drug history and notifying the healthcare provider who wrote the order. Although the
order may be accurately written, determining whether the drug’s benefits outweigh the risks is
not an action within the legal scope of the nurse’s practice. The nurse should not offer false
reassurance and as an advocate for patient safety, should investigate further before giving the
drug. The patient has raised concerns regarding the drug that should promptly be brought to the
provider’s attention. A note on the chart leaves potential for information to be missed.

DIF: Cognitive Level: Applying REF: p. 4

Authentic Nurse Guru: For more of these materials :masterclasstutor1@gmail.com


13. The LPN is preparing to give the initial dose of an antibiotic to a patient diagnosed with
an infection. The patient says, “I broke out in a rash the last time I took that pill.” What action
should the LPN take next?

a. Give the drug and check the patient in 30 minutes for a rash.

b. Document that the patient refused the drug per agency policy.

c. Leave the drug at the bedside while checking the chart for the patient’s allergies.

d. Notify the registered nurse or healthcare provider.


ANS: D
This is a possible adverse reaction, and the RN or healthcare provider should be notified
immediately. You would never give the drug to see if it does cause a rash. Drug should never be
left at the bedside. The patient did not refuse the drug.

DIF: Cognitive Level: Applying REF: p. 9

14. Which priority assessment must you make before giving any patient a drug by mouth?

a. Quiz the patient about the action of each drug.

b. Make sure the patient can swallow.

c. Find out whether the patient prefers cold or room temperature liquids.

d. Ask the patient to repeat his or her name and birthdate.


ANS: B
Before the patient can take any drug by mouth, they must be able to swallow. Asking the patient
to repeat his name and birthdate may be part of using two identifiers but this is important with all
patients. Preferences are also important, but the priority is that the patient be able to swallow the
drug.

DIF: Cognitive Level: Understanding REF: p. 8

Authentic Nurse Guru: For more of these materials :masterclasstutor1@gmail.com


MULTIPLE RESPONSE

1. You are preparing to give the morning drugs to your assigned patients. Before giving
each drug, which steps are considered to be “rights” of giving a drug? (Select all that apply.)

a. The right plan

b. The right time

c. The right dose

d. The right patient

e. The right to self-administer

f. The right drug


ANS: B, C, D, F
The nine rights associated with giving drugs are as follows: right patient, right drug, right time,
right dose, right route, right documentation, right reason, right response, and right to refuse.

DIF: Cognitive Level: Remembering REF: p. 5

2. A licensed practical/vocational nurse has a specific set of roles and responsibilities.


Which responsibilities are within the scope of an LPN/LVN? (Select all that apply.)

a. Writing medical orders

b. Assessment of admission vitals to report to the admitting RN

c. Conducting a comprehensive admission assessment

d. Independent initiation of comprehensive plan of care

e. Management of patient care under RN supervision


ANS: B, E
Authentic Nurse Guru: For more of these materials :masterclasstutor1@gmail.com
Assisting the RN in gathering specific data such as vital signs is within the scope of LPN/LVN
practice. The LPN/LVN works directly with the patient and manages care and assists the RN in
data collection and care plan development. The LPN/LVN does not have the legal authority to
write medical orders or carry out the admission assessment. LPNs/LVNs are more dependent in
the planning and evaluation phase of the nursing process. An LPN/LVN can collect data to assist
the RN with care plan development but is unable to initiate a comprehensive plan of care
independently.

DIF: Cognitive Level: Remembering REF: p. 3

3. The LPN is collecting subjective data for inclusion in the nursing assessment. What
information indicates that the LPN has a clear understanding of the nature of subjective
assessment data? (Select all that apply.)

a. Pulse rate

b. ECG brought in by the patient

c. Chief complaint of chest pain in the patient’s own words

d. Auscultation of lung sounds

e. The patient’s statement of current over the counter and prescription drugs

f. The patient’s laboratory test results


ANS: C, E
Subjective data include information given by the patient or family, like concerns or symptoms
felt by the patient, and their statement of mediations. The pulse rate, recent ECG, laboratory test
results, and lung auscultation obtained through physical examination are examples of objective
data.

DIF: Cognitive Level: Remembering REF: p. 3

Authentic Nurse Guru: For more of these materials :masterclasstutor1@gmail.com


4. A 72 year-old female patient has just been admitted to a rehabilitation center for extended
physical therapy following a stroke. You are obtaining the patient’s drug history. Which
information should you collect? (Select all that apply.)

a. Drugs currently being taken

b. Over-the-counter drugs being taken

c. Referrals to specialists

d. Diseases or conditions requiring drugs

e. Use of herbal supplements

f. Past medical history

g. Alcohol use

h. Drug allergies
ANS: A, B, D, E, G, H
Important information to gather from a patient’s drug history are the current prescription drugs,
OTC drugs, the conditions that denote the patient’s need for a drug, alcohol use, drug allergies,
and the use of nutritional or herbal supplements. The remaining choices are items in the historyof
present or past illnesses, and not concerned with drugs.

DIF: Cognitive Level: Understanding REF: p. 3

Authentic Nurse Guru: For more of these materials :masterclasstutor1@gmail.com


Chapter 02: Legal, Regulatory, and Ethical Aspects of Drug Administration
Visovsky: Introduction to Clinical Pharmacology, 10th Edition

MULTIPLE CHOICE

1. You have obtained a liquid narcotic for a patient in pain from the unit’s locked cabinet.
However, the dose was accidentally spilled before giving it to the patient. What is the best action to
take at this point?

a. Ask another nurse to cosign the inventory record describing the situation.

b. Immediately obtain another dose from the narcotic control system.

c. Document the occurrence in the drug record.

d. Clean up the spill and notify the supervisor.

ANS: A

Accounting for controlled substances is a legal requirement. If the drug is accidentally dropped,
contaminated, or spilled two nurses must sign the inventory report and describe the situation.

DIF: Cognitive Level: Remembering REF: p. 16

2. As you arrive to work, a nurse from the previous shift tells you that she has completed the
narcotic count for your shift. What action should be taken?

a. There is no need for any additional action as this is the standard procedure.

b. Accept the keys to the narcotic cabinet and recount the drugs yourself.

c. Recount the narcotics again with a nurse from the previous shift.

d. Recount the drugs yourself at the end of your shift.

ANS: C

Authentic Nurse Guru: For more of these materials :masterclasstutor1@gmail.com


At the end of each shift, the contents of the locked cabinet are counted together by one nurse from
each shift in order to verify the narcotics count. If not done properly, the nurse risks being held
accountable for any shortages or discrepancies, and may be found guilty of falsifying the narcotic
count records.

DIF: Cognitive Level: Applying REF: p. 16

3. A one-time order for a controlled substance drug has been written for a patient you are caring
for. However, the drug ordered is available only in a larger dose than is needed. What should you
do with the remaining drug?

a. Give the full dose that is available.

b. Flush the remaining drug in the toilet of the patient’s bathroom.

c. Save the remaining drug in case the patient needs it again.

d. Have another nurse to witness wasting of the leftover drug, and document according to policy.

ANS: C

If the ordered dose is smaller than the dose provided (so that some drug must be discarded), two
nurses witness the wasting of the drug, and sign the controlled substance inventory report
according to institution policy.

DIF: Cognitive Level: Understanding REF: p. 16

4. An elderly patient in an assisted living home requests an over-the-counter cough preparation for
a mild cough she is experiencing.

What is your best response?

a. “I do not have any of this drug for you at this time, but can give you a dose from another
patient’s supply.”

b. “I will bring it to you right away, but I must keep it with your other prescription drugs.”

c. “I will need to contact your healthcare provider for an order before I can give this drug to you.”

d. “You may have this, but your family will need to bring it in for you.”
Authentic Nurse Guru: For more of these materials :masterclasstutor1@gmail.com
ANS: C

Over-the-counter (OTC) drugs do not require a prescription for purchase, but a healthcare
provider’s order is required before it can be given by the LPN/LVN in an institutional setting. OTC
drugs may interact with a patient’s prescribed drugs, especially in the elderly.

DIF: Cognitive Level: Applying REF: p. 17

5. Which of the following orders is an example of a single drug order?

a. Atenolol 50 mg orally daily

b. Morphine sulfate 4 mg IV stat

c. Cefazolin 1 g IV 8 a.m. before surgery

d. Tramadol 50 mg orally as needed for pain

ANS: D

A single drug order is a drug that is scheduled to be given at a specified time for one dose only.

DIF: Cognitive Level: Remembering REF: p. 18

6. Diphenhydramine 50 mg IV push is ordered by the healthcare provider to be given “stat.” When


should this drug be given?

a. Immediately

b. As need upon the patient’s request

c. Within 1 hour of receiving the order

d. When you have completed giving the oral drugs first

ANS: A

A stat order is a type of drug order that is to be given immediately.

DIF: Cognitive Level: Knowing REF: p. 18

7. You are giving drugs to the patients assigned to you when you realize that you gave a drug to the
Authentic Nurse
wrong patient. Guru:
What actionFor more
should you of these materials :masterclasstutor1@gmail.com
take?
a. Evaluate the patient’s condition and notify the healthcare provider.

b. Submit a report only if the patient has an adverse reaction.

c. Inform the patient and complete an incident report.

d. Document the occurrence in the patient record.

ANS: A

When it is discovered that an error has been made, you should immediately evaluate the patient for
any adverse reactions and notify the healthcare provider as soon as possible. An incident report
should be completed, and the supervisor notified.

DIF: Cognitive Level: Applying REF: p. 21

MULTIPLE RESPONSE

1. Which of the following drugs is considered a high alert drug? (Select all that apply.)

a. Percocet

b. Insulin

c. Heparin

d. Herceptin

e. Potassium

f. Indomethacin

ANS: B, C, E

Categories of common high alert drugs can be remembered using the acronym “PINCH.” P is for
potassium; I is for insulin, N is for narcotics (opioids), C is for cancer chemotherapy drugs, and H
is for heparin or any drug type that interferes with blood clotting.

DIF: Cognitive Level: Remembering REF: p. 22

2. Which of the following scenarios may be a sign of possible drug diversion on a unit? (Select all
that apply.)
Authentic Nurse Guru: For more of these materials :masterclasstutor1@gmail.com
a. A patient is dissatisfied with the drug administration schedule.

b. A patient receiving oral antibiotics has an excess number of pills.

c. A patient is unaware that the nurse mixed a drug in applesauce.

d. A patient receiving opioids reports increased pain.

e. A nurse reports the narcotic count is inaccurate.

f. A patient brings his pain drug from home to the hospital.

ANS: D, E

Drug diversion is defined as the illegal transfer of regulated drugs (like narcotics) from the patient
for whom it was prescribed, to another person, such as a nurse, for their own (or others) use. Drug
diversion should also be suspected if patients continually report pain despite appropriate drug
treatment, and if inaccurate narcotic counts are noted. While it is not acceptable for patients to
bring or use home-based drugs in the hospital, it is not a sign of drug diversion by a staff member.

DIF: Cognitive Level: Remembering REF: p. 14

3. A 90-year-old woman with dementia is refusing to take her prescribed morning drugs. Another
nurse urges you to mix her drugs into some applesauce and feed it to her as a way to give her drug.
Which of the following actions should you take before you give a drug mixed into food or drink?
List the appropriate actions. (Select all that apply.)

a. Inform the patient or family.

b. Assure the patient she does not need to take her prescribed drugs.

c. Thoroughly crush only pills with an enteric coating.

d. Only mix liquid drugs into food.

e. Give the prescribed drug mixed in food during a regular patient meal.

f. Document the mixing of drugs in food or drink in the chart.

g. Have the family give the food containing the drugs.

Authentic Nurse Guru: For more of these materials :masterclasstutor1@gmail.com


h. Inform the healthcare provider.

ANS: A, F, H

Covert drug administration is discouraged. Therefore, nurses are under obligation to inform the
healthcare provider who ordered the drug, and the patient or family. Some drugs may not be mixed
with certain foods or drinks, or may not be crushed, so checking the drug handbook is a necessary
step to ensure patient safety. The mixing of drugs with food or drink must be documented in the
patients care plan, and on the drug administration chart to address the legal aspects of this practice.

DIF: Cognitive Level: Applying REF: p. 14

4. You have just completed giving the patient drugs for your shift when you discover that you
made a drug administration error. What steps should you take? (Select all that apply.)

a. Fill out an incident report as soon as possible.

b. Fill out an incident report only if the patient suffers an adverse event.

c. Call the healthcare provider immediately.

d. Check your patient and assess vital signs.

e. Report the drug error to the Joint Commission.

f. Discuss the potential cause of the error with the nurse manager.

ANS: A, C, D

The priority action is related to patient safety. So, the first step taken is to check the patient, and
assess vital signs, then notify the healthcare provider. All drug errors must be reported, typically
though an incident report. The incident report is then used to uncover what may have led to the
error to prevent the error from re-occurring.

DIF: Cognitive Level: Applying REF: p. 21

5. You are caring for an elderly patient who was just admitted the rehabilitation unit following a
hospitalization for a fractured hip.

You suspect that in the transfer from the hospital to the rehabilitation unit an error in the patient’s
Authentic Nurse Guru: For more of these materials :masterclasstutor1@gmail.com
drug orders may have been made.

Which steps should you take to identify possible drug errors in a patient’s drug orders? (Select all
that apply.)

a. Call the pharmacy.

b. Clarify anything that is unreadable.

c. Ask the patient about the drug.

d. Reconcile the drug list with an old drug record.

e. Clarify vague orders with the healthcare provider.

f. Check the original written order with the healthcare provider.

g. Inform the nurse manager.

ANS: A, B, E, F

Checking with a reliable source, such as the pharmacist, clarifying vague orders or anything that is
difficult to read (if handwritten), and checking the original written order are all ways to avoid drug
errors. Informing the nurse manager is not only contraindicated but also not essential to
investigating the source of the potential drug error.

DIF: Cognitive Level: Applying REF: p. 21

6. Which levels of regulation must you adhere to when giving drugs to a patient? (Select all that
apply.)

a. City Regulations

b. State Regulations

c. County Regulations

d. Federal Regulations

e. Institutional Regulations

f. Health Insurance
Authentic Regulations
Nurse Guru: For more of these materials :masterclasstutor1@gmail.com
ANS: B, D, E

Nurses who are responsible for giving drugs to patients in their care have three levels of
regulations to follow: federal (describes and controls), state (regulates who dispenses), and
individual hospital or agency (additional guidelines or policies).

DIF: Cognitive Level: Remembering REF: p. 18

7. A narcotic control system is used in any hospital or agency. Which of the following are special
regulations applied for control of narcotics that you must follow? List the special regulations
applied for the control of narcotics that the nurse must follow. (Select all that apply.)

a. Narcotics may be borrowed from patient to patient for emergency use.

b. Narcotics are stored in a special locked cabinet.

c. You may return unused narcotics to the patient’s family upon discharge.

d. An inventory of the narcotics on a unit must be kept and verified by two nurses.

e. You are responsible for signing out every narcotic drug used for a patient.

f. Narcotics control is the responsibility of everyone on the unit.

ANS: B, D, E

Narcotics are stored in special, limited-access, locked cabinets. A nurse records all controlled-
substance drug during the shift. The inventory report form is completed before the drug is removed
from the cabinet.

DIF: Cognitive Level: Applying REF: p. 16

8. A discrepancy in the narcotics inventory for morphine 5 mg/mL vials is discovered when the
narcotics count is performed. The count is short by one vial. Which of the following steps should
you take to reconcile the count? (Select all that apply.)

a. No action needs to be taken for small discrepancies.

b. Notify the nursing supervisor and the pharmacy of the discrepancy.

c. Identify if any nurse forgot to record any of the narcotics removed.


Authentic Nurse Guru: For more of these materials :masterclasstutor1@gmail.com
d. Ask only the nurses who used narcotics about the drugs they have given.

e. Check drug records to reconcile if narcotics given and not signed for.

f. Notify the security department of the institution if drug diversion is suspected.

ANS: B, C, E, F

All nurses must be asked about narcotics that may have been given. Steps must be retraced to see if
someone forgot to record any drug. Patient charts might also be checked to see if drug was given
that was not signed for on the inventory report. If errors in the report cannot be found, both the
pharmacy and the nursing service office must be notified. If drug diversion is suspected, the
hospital administrator and security police are usually contacted.

DIF: Cognitive Level: Applying REF: p. 16

Authentic Nurse Guru: For more of these materials :masterclasstutor1@gmail.com


Chapter 03: Principles of Pharmacology Visovsky: Introduction to Clinical
Pharmacology, 10th Edition

MULTIPLE CHOICE

1. Which is always true regarding the generic name for a drug?

a. The generic name is capitalized when written.

b. The generic name is similar to the chemical name.

c. The generic name is the same in any country.

d. The generic name is assigned by a specific manufacturer.

ANS: C

The generic name is the common drug name used. This name is the same in all countries. The
generic drug name is written in lower-case letters, whereas the trade name or brand name of a
drug is capitalized.

DIF: Cognitive Level: Remembering REF: p. 26

2. What is the difference between the generic and the trade names of drugs?

a. The generic name is approved and owned by the Federal Drug Standards Committee.

b. The generic name represents the proprietary name given by a drug company.

c. The trade name is one that is registered to a specific drug manufacturer.

d. The trade name, or chemical name, represents the chemicals in the drug.

ANS: C

The trade name is the proprietary or brand name for the drug and is registered to a specific drug
company.

DIF: Cognitive Level: Remembering REF: p. 26

3. You are teaching a new nursing student about the way drugs attach to its receptor sites. Which
Authentic Nurse Guru: For more of these materials :masterclasstutor1@gmail.com
statement made indicates the students understanding about the way drugs and receptor sites work?

a. “The drug attaches to a receptor site and activates it.”

b. “The drug prevents activation of the receptor site.”

c. “The receptor site is activated, allowing DNA replication to occur”.

d. “The receptor sites are activated once the drug leaves the receptor site.”

ANS: A

Drug and receptor sites work by having the drug attached to the receptor site and activating the
receptor.

DIF: Cognitive Level: Understanding REF: p. 27

4. You are giving the drug antagonist naloxone (Narcan) to a patient who has experienced an
opioid overdose. What response would you expect in this patient from a drug antagonist?

a. An opioid antagonist would result in negative activation of cell receptors.

b. An opioid antagonist would result in positive activation of cell receptors.

c. An opioid antagonist would result in partial activation of cell receptors.

d. An opioid antagonist would result in no activation of cell receptors.

ANS: D

When a drug attaches at a drug receptor site but does not activate or unlock it, there is no increase
in cell activity and the drug is an antagonist.

DIF: Cognitive Level: Remembering REF: p. 27

5. You have just given a patient a drug that is a known agonist. The patient asks how this drug is
expected to work. What is your best response?

a. “This drug works by deactivation of all cellular receptor sites.”

b. “This drug works to partially block receptor sites.”

Authentic Nurse Guru: For more of these materials :masterclasstutor1@gmail.com


c. “This drug works by interacting with other drugs.”

d. “This drug works by attaching to the receptor site to activate it.”

ANS: D

A drug agonist works by activating or unlocking receptors, and has the same actions as the body’s
own chemicals.

DIF: Cognitive Level: Applying REF: p. 27

6. You are giving an oral drug to a patient who asks you to explain the reason for taking water
with an oral tablet. What is your best response?

a. “Water is important for proper metabolism.”

b. “Juice or milk can keep it from dissolving.”

c. “Water helps to dissolve the drug.”

d. “Water keeps the drug potent.”

ANS: C

All drugs must be dissolved in body fluid before they enter body tissues. When the patient takes
water with a tablet, it not only helps in swallowing but also helps dissolve the drug and increase
its solubility.

DIF: Cognitive Level: Understanding REF: p. 28

7. You are about to give an IV antibiotic to a patient hospitalized with severe cellulitis of the right
leg. The patient asks why the drug is being given by the intravenous route instead of by mouth.
What is your best response?

a. “This antibiotic is not available in oral form.”

b. “This antibiotic can act faster if given by IV than if given by mouth.”

c. “This antibiotic can cause stomach ulcers if given by mouth.”

d. “This antibiotic was ordered to be given by the IV route.”


Authentic Nurse Guru: For more of these materials :masterclasstutor1@gmail.com
ANS: B

Drugs injected intravenously into the bloodstream have the fastest action.

DIF: Cognitive Level: Remembering REF: p. 28

8. What is the process by which drug molecules move from a region of higher concentration to
one of lower concentration?

a. Diffusion

b. Osmosis

c. Filtration

d. Solubility

ANS: A

In diffusion, molecules move from a region of higher concentration to one of lower concentration.

DIF: Cognitive Level: Remembering REF: p. 28

9. A 62-year-old male patient with liver disease asks you why he is receiving a drug intravenously
rather than by mouth. What is your best response?

a. “Many oral drugs are inactivated as you get older.”

b. “Your liver disease impairs the transformation of a drug into its active form.”

c. “Intravenous drugs reduce toxicity to the liver through first-pass metabolism.”

d. “Individuals with liver disease have a genetic impairment that prevents drug activation.”

ANS: B

Many drugs must be activated by enzymes before they can be used in the body. This
biotransformation happens in the liver. Liver disease impairs this process.

DIF: Cognitive Level: Applying REF: p. 29

10. A patient who is beginning a new cholesterol-lowering drug tells you he plans to take this
Authentic Nurse Guru: For more of these materials :masterclasstutor1@gmail.com
drug in the evening with his usual glass of grapefruit juice. What is your best response?

a. “Taking grapefruit juice can reduce the effectiveness of the drug.”

b. “Taking grapefruit juice can increase the absorption of the drug.”

c. “The vitamin C in grapefruit juice enhances the action of cholesterol-lowering drugs.”

d. “The acid environment of grapefruit juice promotes drug metabolism.”

ANS: A

Grapefruit juice affects (usually reduces) the absorption of many drugs such as antihistamines,
cholesterol-lowering drugs, HIV drugs, and transplant drugs.

DIF: Cognitive Level: Applying REF: p. 30

11. You give a patient 400 mg of an oral drug at noon. This drug has a half-life of 2 hours. At
what time will 100 mg remain?

a. 2:00 p.m.

b. 6:00 p.m.

c. 8:00 p.m.

d. 4:00 p.m.
ANS: D

With a half-life of 2 hours, the amount of drug remaining in the blood decreases by 50% every 2
hours. So, when 400 mg are given at noon, by 2:00 p.m., 200 mg remain; by 4:00 p.m., 100 mg
remain.

DIF: Cognitive Level: Understanding REF: p. 30

12. When teaching a patient about a new drug that has been prescribed, the patient asks you to
explain the difference between a side effect and an adverse reaction. What is your best response?

a. “Side effects are mild, and adverse effects are severe.”

b. “Both side effects and adverse reactions are mild.”

Authentic
c. “Both Nurse Guru:
side effects For more
and adverse of these
reactions materials :masterclasstutor1@gmail.com
are severe.”
d. “Side effects are severe, and adverse reactions are mild.”

ANS: A

Side effects are usually seen as mild but annoying responses to a drug. Adverse reactions, or
adverse effects, usually imply more severe symptoms or problems that develop because of the
drug.

DIF: Cognitive Level: Applying REF: p. 30

13. A 42-year-old female patient recovering from a recent fall tells you that she is experiencing
sleepiness after taking a drug prescribed for back pain. What is your best response?

a. “You are having a severe drug interaction.”

b. “You are having an idiosyncratic response to this drug.”

c. “You are having the desired effect of this drug.”

d. “You are having an expected drug side effect.”

ANS: D

Drugs may influence many body systems at the same time, so the effect of the drug is often not
restricted to the desired action. Side effects are usually seen as mild but annoying responses to the
drug. In the case of many drugs for pain, sleepiness is an expected side effect.

DIF: Cognitive Level: Applying REF: p. 30

14. A 27-year-old male patient with a sexually transmitted infection who was given intramuscular
penicillin develops hives, itching, and facial swelling after the injection. What type of drug
reaction is this patient experiencing?

a. An adverse reaction

b. A paradoxical reaction

c. A hypersensitivity reaction

d. An anaphylactic reaction

Authentic Nurse Guru: For more of these materials :masterclasstutor1@gmail.com


ANS: C

Some drugs (sulfa products, aspirin, and penicillin) can produce allergic (hypersensitivity)
reactions that usually occur when an individual has taken the drug and the body has developed
antibodies to it.

DIF: Cognitive Level: Understanding REF: p. 31

15. A 21-year-old woman has been prescribed a 10-day course of an oral antibiotic for a sinus
infection. What information should be included in the teaching plan for this patient?

a. Antibiotics can have a synergistic effect when given with opiates.

b. Antibiotics can mask the signs of allergic reactions.

c. Antibiotics interact with birth control pills making them less effective.

d. Antibiotics interfere with P-450 enzyme pathways in women.

ANS: C

Drug interactions can produce adverse effects. Many antibiotics make birth control tablets less
effective, thus making it more likely that a woman will get pregnant while taking both drugs if she
is sexually active. Therefore, the interactions of antibiotics with birth control pills should be
taught to women of childbearing age.

DIF: Cognitive Level: Applying REF: p. 32

16. Probenecid is sometimes given to patients taking penicillin to increase the amount of
penicillin absorbed. Which effect is this an example of?

a. Additive effect

b. Drug interaction

c. Synergistic effect

d. Antagonistic effect

ANS: A

Authentic Nurse Guru: For more of these materials :masterclasstutor1@gmail.com


Some drugs are given together on purpose to produce an additive effect that is helpful in
increasing the levels of penicillin in the blood. Probenecid does this by blocking penicillin’s
elimination pathway via the kidneys (by inhibiting renal tubular secretion).

Maintaining high blood levels of penicillin is important in treating severe infections.

DIF: Cognitive Level: Understanding REF: p. 32

17. A patient who has been prescribed an antianxiety drug to be taken in the evening tells you he
has 2 to 3 beers with dinner each night. What is your best response?

a. “The effect of alcohol on this drug is not a concern.”

b. “Drinking any alcohol with this drug can have a harmful additive effect.”

c. “Limit your alcohol use to 8 ounces a day with a meal.”

d. “Eating a full meal before taking the drug will negate any side effects.”

ANS: B

When taken together with alcohol, drugs such as antianxiety agents can result in an additive effect
that may be harmful.

DIF: Cognitive Level: Applying REF: p. 32

18. You are caring for a patient with liver failure who will be starting on an oral anticoagulant for
the treatment of a deep vein thrombosis. What effect of this drug should you expect?

a. The effect of the anticoagulant will be prolonged and pose a risk for bleeding.

b. The effect of the anticoagulant will be reduced, requiring additional doses.

c. The effect of the anticoagulant will result in an increased risk for infection.

d. There will be no effect of the anticoagulant on the deep vein thrombosis.

ANS: A

A patient with liver or the kidney disease will retain a drug much longer, increasing the risk for
adverse and toxic effects. In the case of an oral anticoagulant, the risk for bleeding is increased in
Authentic Nurse Guru: For more of these materials :masterclasstutor1@gmail.com
a patient with liver disease.

DIF: Cognitive Level: Applying REF: p. 33

19. You are caring for a patient who has been prescribed an oral opioid for pain following the
surgical repair of a fractured ankle. The patient’s social history states that he drinks alcohol on a
daily basis. Which statement regarding pain relief for this patient is true?

a. A lower dose of opioids is needed for patients who drink alcohol daily.

b. Daily alcohol intake can increase the activity of metabolic enzymes in this patient.

c. There is no effect on metabolism of opioids as long as the patient has normal liver function.

d. In patients who drink alcohol daily, prodrugs like opioids are activated more slowly.

ANS: B

In patient who drinks alcohol daily, there is an increase in the activity of metabolic enzymes,
requiring higher doses of the opioid to achieve pain control.

DIF: Cognitive Level: Understanding REF: p. 33

20. Which statement concerning absorption of oral drugs is true for neonates?

a. Oral prodrugs are activated most quickly in the neonate.

b. Oral drugs are poorly absorbed for the gastrointestinal tract of the neonate.

c. The overall low body metabolism results in increased toxicity in the neonate.

d. The high percentage of total body water rapidly dilutes oral drugs in the neonate.

ANS: B

In neonates oral drugs are absorbed poorly from the GI tract because no gastric acid is present to
help break down drugs, no intestinal bacteria or enzyme function is present to metabolize a drug,
and the time it takes for a drug to move through the stomach and intestines is slow.

DIF: Cognitive Level: Remembering REF: p. 33

21. The nurse


Authentic Nurse receives
Guru:anFor
ordermore
to giveofmethylprednisolone
these materials125 mg/1 mL IM. The nurse knows
:masterclasstutor1@gmail.com
that she should inject this drug into the ----------- to assure the drug is absorbed most rapidly.

a. Vastus lateralis

b. Dorsogluteal muscle

c. Abdomen

d. Ventrogluteal muscle

e. Deltoid muscle

ANS: E

The deltoid muscle has faster absorption than other muscles. The abdomen is used for
subcutaneous injections for fast absorption.

DIF: Cognitive Level: Remembering REF: p. 28

22. You are preparing to give an oral drug to a patient. You are aware that in order for the drug to
be used by the body, the drug needs--------------- to be absorbed through the processes of -------- , -
------------and to move drug molecules.Using into the circulation.

a. active transport; excretion; pharmacodynamics; distribution

b. metabolism; receptor binding; receptor site; solubility

c. diffusion; filtration; osmosis; passive transport

d. diffusion; excretion; pharmacodynamics; passive transport

ANS: C

Absorption involves the way a drug enters the body and passes into the body and tissues. Drug
absorption takes place through the processes of diffusion, filtration, and osmosis. Many drugs
enter the circulation using passive transport.

DIF: Cognitive Level: Understanding REF: p. 27

MULTIPLE RESPONSE

1. WhichNurse
Authentic of the following
Guru: Forfactors
moreare known to influence
of these drug:masterclasstutor1@gmail.com
materials distribution? (Select all that apply.)
a. Drug binding

b. Tissue type

c. Receptor site binding

d. Electron displacement

e. Cell membrane penetration

f. Tissue perfusion

ANS: A, B, E, F

Distribution of the drug depends upon tissue (perfusion), types of tissue (bone, fat, and muscle)
and how easy it is for the drug to penetrate the cell membranes. Some drugs will also bind
together with many blood substances and proteins such as albumin. This binding allows only
“free” drug (that which is not bound) to penetrate the tissues.

DIF: Cognitive Level: Remembering REF: p. 29

2. Which of the following routes are sites of excretion of drug metabolites from the body? (Select
all that apply.)

a. Feces

b. Urine

c. Breast milk

d. Bloodstream

e. Lymph nodes

f. Salivary glands

ANS: A, B, C

All inactive chemicals, chemical by-products, and waste (metabolites) finally break down through
metabolism and are removed from the body through the process of excretion in feces, urine, and
breast milk.
Authentic Nurse Guru: For more of these materials :masterclasstutor1@gmail.com
DIF: Cognitive Level: Remembering REF: p. 29

3. What are the possible responses a patient can have to a drug’s actions? (Select all that apply.)

a. Desired action

b. Bioequivalence

c. Adverse effects

d. Side effects

e. Half-life

f. Onset

ANS: A, C, D

When a drug is given to a patient, the expected response of the drug is called the desired action.
Other responses to drugs include side effects and adverse effects.

DIF: Cognitive Level: Applying REF: p. 30

4. A nurse working at an assisted living facility knows that there are certain pharmacologic
challenges in older adults. Which of the following issues related to drug therapy in older patients
are true? (Select all that apply.)

a. Drug interactions are far less prevalent in older patients.

b. Drugs are secreted more slowly in older patients.

c. Adverse drug reactions are more common in older patients.

d. Fat-soluble drugs are eliminated and leave the body quickly in older patients.

e. Dehydration can cause the blood level of drug to increase in older patients.

f. Loss of electrolytes due to diuretics can result in loss of electrolytes in older patients.

ANS: B, C, E, F

Drugs are absorbed, metabolized, and excreted more slowly and less completely in older adults.
Authentic Nurse Guru: For more of these materials :masterclasstutor1@gmail.com
Adverse drug reactions (ADRs) are common in older adults. Issues such as falls, hypotension,
delirium, kidney failure, and bleeding are common clinical manifestations. Older adult patients
may become dehydrated easily, thus allowing the amount of drug in the blood to increase.

Diuretics lead to an increase in urination and this can lead to loss of electrolytes.

DIF: Cognitive Level: Understanding REF: p. 34

Authentic Nurse Guru: For more of these materials :masterclasstutor1@gmail.com


Chapter 04: Drug Calculation: Preparing and Giving Drugs Visovsky:
Introduction to Clinical Pharmacology, 10th Edition

MULTIPLE CHOICE

1. When calculating the dose of a drug to be given to a child, Clark’s rule is used. What measure
is Clark’s rule based upon?

a. The age of the child

b. The weight of the child

c. One-half of the adult dose

d. The international standardized ratio

ANS: B

The Joint Commission now recommends that all dosages for children be weight-based. Clark’s
rule: Pediatric child dose = (weight of child/150 lb adult dose).

DIF: Cognitive Level: Remembering REF: p. 43

2. An elderly patient is scheduled to take six drugs each morning. What action should you take
when giving these drugs?

a. Allow extra time to give all the drugs.

b. Crush all the drugs before giving them.

c. Allow the patient to take only the drugs she can swallow.

d. Leave the drugs at the bedside so the patient can take them slowly.

ANS: A

Allow extra time to give drug to the elderly. These individuals often are slower at swallowing
drugs and water. The nurse must be present for the administration of every drug.

DIF: Cognitive Level: Applying REF: p. 47

Authentic Nurse Guru: For more of these materials :masterclasstutor1@gmail.com


3. You have just finished giving a 72-year-old male patient with a nasogastric (NG) tube
connected to suction his morning drugs.

What action is appropriate to take next?

a. The NG tube is clamped for 10 minutes and then reconnected to suction.

b. The NG tube is clamped for 30 minutes and then reconnected to suction.

c. The NG tube is immediately reconnected to the suction.

d. The NG tube is disconnected for 4 hours, and then reconnected to suction.

ANS: B

When the drug has passed through the tube, reclamp the tube for 30 minutes before reattaching
the suction.

DIF: Cognitive Level: Remembering REF: p. 47

4. A patient in your care is suddenly experiencing a life-threatening emergency. Which route


would you expect that emergency drugs may be given during this situation?

a. IV route

b. IM route

c. Rectal route

d. Subcutaneous route

ANS: A

IV injections or infusions may be needed when drug must go directly into the bloodstream,
because the action of this method is rapid.

DIF: Cognitive Level: Understanding REF: p. 48

5. The LPN/LVN is about to prepare an intramuscular injection for a patient containing a


nonsteroidal anti-inflammatory drug for pain. Which is the correct needle gauge for giving an
intramuscular (IM) injection?
Authentic Nurse Guru: For more of these materials :masterclasstutor1@gmail.com
a. 18 G, 1-inch needle

b. 20 G, 1

/2-inch needle

c. 25 G, 2-inch needle

d. 21 G, 11

/2-inch needle

ANS: D

An IM injection is usually given with a 1- to 2-inch, 20- to 22-gauge needle.

DIF: Cognitive Level: Understanding REF: p. 49

6. Which principle of drug administration will you include in the teaching plan of a patient who
will be giving his own subcutaneous injections?

a. Rotate sites among the upper arm, abdomen, and anterior thigh.

b. Avoid injecting within 3 inches of a previous injection site.

c. Insert the needle at a 30-degree angle to the skin.

d. Use a 22-gauge, 5/8-inch needle.

ANS: A

The patient should be taught the principles of injection site rotation and be given a diagram to take
home at discharge.

DIF: Cognitive Level: Applying REF: p. 53

7. What physical assessment findings would you observe when an IV becomes infiltrated?

a. Pallor and pain

b. Pallor, warmth

Authentic Nurse and


c. Pain, warmth, Guru: For more of these materials :masterclasstutor1@gmail.com
burning
d. Pain, swelling, and redness

ANS: D

Infiltration produces pain, swelling of the area, and redness. Pain with warmth and burning are
signs of infection.

DIF: Cognitive Level: Understanding REF: p. 59

8. Which correct step should you take when giving a Z-track injection?

a. Draw up the exact amount of the drug adding 0.5 mL of air.

b. Do not massage the injection site after giving the drug.

c. Use the deltoid site for this injection.

d. Pull the tissue upward and away.

ANS: B

The “Z-track technique” of IM injection uses the skin itself as a “door” to seal in the drug and
prevent it from leaking back out from muscle tissue. 0.1 to 0.2 mL of air is added to the drug after
it is drawn up. The tissue is pulled down and away, and the site is not massaged after the injection
is given.

DIF: Cognitive Level: Applying REF: p. 55

9. What step should be taken when applying a topical nitroglycerin ointment to a patient?

a. Squeeze the nitroglycerine ointment onto the applicator paper and place it on the skin.

b. Apply the nitroglycerine ointment to the medial aspect of the thigh.

c. Massage the nitroglycerine ointment thoroughly into the skin.

d. Shave the skin before application.

ANS: A

The correct number of inches of drug is squeezed onto the applicator paper as a small ribbon. The
applicator
Authentic paper isGuru:
Nurse then laid
Foronmore
top of of
the these
skin where the drug :masterclasstutor1@gmail.com
materials is to be applied. A nonhairy area on
the chest, upper arm, or flank area should be selected for application. Apply it to the paper
because the nurse can receive some of the drug if it comes in contact with his or her fingers and is
absorbed into the skin.

DIF: Cognitive Level: Applying REF: p. 61

10. You are preparing to give a drug by the transdermal route to a patient. Which safety
precaution should you use when giving this type of drug?

a. Always wear gloves when giving transdermal drugs.

b. To maintain blood levels, do not clean the skin.

c. Remove all patches before showering.

d. Keep all prior patches on the skin.

ANS: A

Always wear gloves to avoid drug absorption onto your own skin.

DIF: Cognitive Level: Applying REF: p. 60

11. You are preparing to teach a teenage patient with asthma about using a metered-dose inhaler.
What instruction would be appropriate in teaching this patient?

a. Never shake the inhaler drug before use.

b. Hold the head back while inhaling the drug.

c. Exhale while squeezing the canister to deliver the drug.

d. Sit upright, exhale, then activate the inhaler as the next inhalation begins.

ANS: D

The patient should exhale and then activate the inhaler as the next inspiration begins. This will
carry drug down into the lungs.

DIF: Cognitive Level: Applying REF: p. 63

12. You Nurse


Authentic are instructing
Guru:a For
clinicmore
patientof
onthese
how to materials
use an inhaler during an asthma attack. Which
:masterclasstutor1@gmail.com
statement is most important about the use of an inhaler during an asthma attack?

a. The drug must be delivered into the back of the nose.

b. The drug must be delivered directly into the throat.

c. The drug must be delivered into the lungs.

d. The drug must be delivered onto the tongue.

ANS: C

If using inhalers, patients must be carefully instructed so the drug goes all the way into the lungs,
not just to the back of the nose or throat. Take a deep breath first, exhale, and then inhale the drug.

DIF: Cognitive Level: Understanding REF: p. 63

13. A patient with pneumonia has been ordered to receive an antibiotic to be given by the
intravenous (IV) route. The nurse uses her understanding of drug preparation and administration
to complete this task.

The nurse first before preparing the drug. Once the drug is prepared, the nurse---------

a. checks the drug order and the patient’s allergy history; identifies the patient per institution
policy and explains what drug is being given

b. returns expired drugs to the pharmacy and the patient’s allergy history; uses betadine to prepare
the IV site for injection and explains what drug is being given

c. reports adverse reactions and the patient’s allergy history; obtains a 25-gauge needle and
prepares the skin for injection

d. gives the drug and disposes of the glass vial in the trash; reports adverse reactions and checks
the patient allergy history

ANS: A

The nurse first checks the drug order and the patient’s allergy history before preparing the drug.
Once the drug is prepared, the nurse identifies the patient per institution policy and explains what
drug is being given.
Authentic Nurse Guru: For more of these materials :masterclasstutor1@gmail.com
DIF: Cognitive Level: Applying REF: p. 45

14. You are caring for a patient with Parkinson’s disease who has been ordered to receive an oral
capsule of the drug rivastigmine.

Which actions represent the proper procedure for giving an oral capsule?

a. Crush and dilute the capsule contents in warm water.

b. Tell the patient to swallow the capsule whole.

c. Open the capsule and mix the drug with soft foods. Tell the patient to chew the capsule
completely.

d. Pierce the capsule with a needle and squeeze the contents into the mouth.

e. Remain at the patient’s bedside until the drug is swallowed.

ANS: B

Remain at the patient’s bedside until the drug is swallowed. Do not crush tablets or break capsules
without checking with the pharmacist. Many drugs have special coatings that are essential for
proper absorption. Capsules should not be opened or mixed with foods without checking with the
pharmacist for potential interactions.

DIF: Cognitive Level: Applying REF: p. 46

MULTIPLE RESPONSE

1. The nurse is taking care of a patient receiving an IV infusion. During her rounds, the nurse
discovers that the IV infusion is failing to flow properly. List the nursing action for the patient’s
care at this time. (Select all that apply.)

a. Change the IV tubing.

b. Lower the IV pole height.

c. Check the IV tubing for kinks.

d. Check the position of the needle.

Authentic Nurse Guru: For more of these materials :masterclasstutor1@gmail.com


e. Examine IV site for infiltration.

f. Check the IV solution type.

g. Reposition the patient’s wrist and elbow.

ANS: B, C, D, E, G

Failure of an IV to infuse properly warrants the following nursing actions: check for bent or
kinked tubing, reposition the patient’s wrist and elbow, check the needle position—it should be
against a vein wall, the IV pole may be too low, or the needle may be out of the vein and
infiltrated.

DIF: Cognitive Level: Applying REF: p. 59

2. You are educating a nursing student about the different ways drugs can be absorbed. Which of
the following drugs are absorbed through the mucosal membranes? (Select all that apply.)

a. Metered-dose inhaler

b. Oral tablets

c. Transdermal drugs

d. Rectal suppositories

e. Intravenous drugs

f. Liquid drugs

ANS: A, C, D

Percutaneous drugs are given through mucous membranes. Transdermal drugs, rectal
suppositories, and inhalers are examples of percutaneous drugs.

DIF: Cognitive Level: Remembering REF: p. 62

3. Which of the following are examples of drugs given in units instead of milligrams? (Select all
that apply.)

a. Insulin
Authentic Nurse Guru: For more of these materials :masterclasstutor1@gmail.com
b. Furosemide

c. Normal saline

d. Heparin

e. Nitroglycerine

f. Intramuscular penicillin

ANS: A, D, F

Insulin and heparin are parenteral drugs given in units, not milligrams. Intramuscular or
intravenous penicillin is also given in units.

DIF: Cognitive Level: Remembering REF: p. 42

4. A 68-year-old woman who was admitted with a stroke has had a nasogastric (NG) tube placed
and is receiving enteral feedings through the NG tube. You are about to give her the prescribed
drugs through the NG tube. List the actions you will take to prevent harm.

a. Take the vital signs.

b. Check the NG tube placement.

c. Ask the patient if the NG tube is causing pain.

d. Flush the NG tube with 30 mL of water.

e. Check the stomach content residual.

f. Crush all drugs to a fine powder and give them together.

g. Clamp the NG tube for 30 minutes after giving the drugs.

h. Allow the drug to run into the NG tube by gravity.

ANS: B, D, E, G, H

Make certain that the NG tube is in the stomach. Aspirate (take out) stomach contents with a
syringe and test the pH of the stomach contents. If the pH is 0 to 5, the NG tube is most likely in
the stomach.
Authentic NurseOnce this is For
Guru: done,more
flush the
of NG tubematerials
these with water, :masterclasstutor1@gmail.com
allow the drugs to run into the NG
tube via gravity, and finally, clamp the NG tube off for 30 minutes after giving the drugs.

DIF: Cognitive Level: Understanding REF: p. 47

5. You are preparing to give both a subcutaneous injection of insulin and an influenza vaccine to a
patient. What precautions should you take to prevent injury from a needlestick? (Select all that
apply.)

a. Recap both needles before disposal.

b. Remove the needle from the syringe, placing both in the designated sharps container.

c. Carry the needles and syringes with you until it can be disposed of.

d. Immediately discard each syringes in the sharps container.

e. Ask a coworker to hold the first syringe until you are finished giving both injections.

f. Ensure there is a sharps container within easy reach of your work area.

ANS: D, E

Standard precautions recommend the use of puncture-resistant (sharps) containers for disposal of
all needles and sharps. Check if one is available in the area where you are giving injections. Never
recap a needle; most sticks happen this way. Never remove the needle from the syringe after
injecting into a patient. Never ask anyone to hold used syringes, they should be discarded
immediately after use.

DIF: Cognitive Level: Understanding REF: p. 48

Authentic Nurse Guru: For more of these materials :masterclasstutor1@gmail.com


Chapter 05: Drugs for Bacterial Infections

Visovsky: Introduction to Clinical Pharmacology, 10th Edition

MULTIPLE CHOICE

1. A patient receiving antibiotics for the treatment of a bacterial wound infection develops a
secondary fungal infection. The patient asks you to explain why the fungal infection developed.
What is your best response?

a. “Antibiotics can upset the balance of normal flora and allow yeast or fungus to grow.”

b. “Antibiotic resistance has allowed the bacteria in your wound to become a fungus.”

c. “Genetic changes induced by antibiotics lead to this fungal infection.”

d. “There is no known cause for this secondary fungal infection.”

ANS: A

Antibiotic use can upset the normal flora balance in the body and cause yeast or fungal infections
to occur. Candida is a common body yeast and often overgrows to cause a fungal infection. When
a person is given antibiotics to kill infectious bacteria, the normal flora is killed off as well.

DIF: Cognitive Level: Applying REF: p. 69

2. A breast-feeding mother has been prescribed antimicrobial therapy for an infection. What
information should be included in her teaching plan?

a. Breast-feeding is permitted during antimicrobial therapy and should be continued.

b. Breast-feeding can continue if the breast milk is pumped and given by bottle.

c. Breast-feeding should be avoided during antimicrobial therapy.

d. Breast-feeding during antimicrobial therapy can lead to mastitis.

ANS: C

Breast-feeding should be avoided during antimicrobial therapy because most of these drugs are
excreted into breast milk and the infant (who may not have an infection) will be exposed to the
Authentic Nurse Guru: For more of these materials :masterclasstutor1@gmail.com
actions, side effects, and adverse effects.

DIF: Cognitive Level: Applying REF: p. 70

3. Which term refers to an antibiotic with a high degree of activity against many different
organisms?

a. Antimycotic

b. Bacteriostatic

c. Antimicrobial

d. Broad-spectrum

ANS: D

Drugs that are effective against a variety of organisms are called broad-spectrum antibiotics.

DIF: Cognitive Level: Remembering REF: p. 70

4. You are caring for an elderly patient who is newly admitted with a suspected urinary tract
infection requiring antimicrobial therapy.

Which of the following tests would you expect the healthcare provider to order before initiating
treatment?

a. Abdominal x-ray

b. Blood chemistry

c. Laparoscopic examination

d. Culture and sensitivity studies

ANS: D

The organisms present in the urine must be carefully cultured and tested to see which drugs are
effective against them (drug sensitivity).

DIF: Cognitive Level: Remembering REF: p. 69

Authentic Nurse Guru: For more of these materials :masterclasstutor1@gmail.com


5. You are caring for a patient with an upper respiratory infection from group A beta-hemolytic
streptococci. The patient asks you what antimicrobial drug is effective for this infection. What is
your best response?

a. Cefazolin

b. Penicillin

c. Vancomycin

d. Sulfamethoxazole/trimethoprim

ANS: B

Penicillin is effective in the treatment of the following susceptible organisms: group A beta-
hemolytic streptococci and other less common organisms.

DIF: Cognitive Level: Applying REF: p. 73

6. You are teaching a patient about possible side effects when taking penicillin, a broad-spectrum
antibiotic. What is the most common side effect you would stress in teaching the patient about this
drug?

a. Edema of the lower extremities

b. Metallic taste in the mouth

c. Ringing in the ears

d. Loose stools

ANS: D

The most common side effect of penicillin (and many other antibiotics) is simple diarrhea of two
to four loose stools daily.

DIF: Cognitive Level: Applying REF: p. 73

7. Which statement is true of any antibiotic used to treat an infection caused by a penicillinase-
producing organism?

Authentic Nurse Guru: For more of these materials :masterclasstutor1@gmail.com


a. The antibiotic should be acid stable.

b. The antibiotic should be penicillinase resistant.

c. The antibiotic should have an extended-release formulation.

d. The antibiotic should be a narrow-spectrum preparation.

ANS: B

Penicillins come in different forms; one class is natural penicillinase-resistant penicillin.

DIF: Cognitive Level: Remembering REF: p. 73

8. Which effect or drug interaction of ampicillin would be important to teach to female patient of
childbearing age?

a. “Ampicillin can reduce the effect of oral contraceptives.”

b. “Ampicillin is given only to patients allergic to penicillin.”

c. “Taking ampicillin can upset your menstrual cycle for the next month.”

d. “NSAIDS taken for menstrual pain can decrease serum levels of ampicillin.”

ANS: A

Ampicillin reduces the effectiveness of oral contraceptives, which can lead to an unplanned
pregnancy. Teach women of childbearing age to use two reliable methods of birth control while
taking any penicillin to prevent unplanned pregnancy.

DIF: Cognitive Level: Applying REF: p. 73

9. You are caring for an elderly patient in a long-term care facility who has been prescribed a
cephalosporin. Which adverse effect should you monitor the patient for while taking this drug?

a. Lethargy

b. Nephrotoxicity

c. Peripheral edema

Authentic Nurse Guru: For more of these materials :masterclasstutor1@gmail.com


d. Sleep disturbance

ANS: B

Nephrotoxicity (kidney toxic effects) has been reported with some cephalosporins, and the
incidence is greater in older adult patients and in patients with poor renal function.

DIF: Cognitive Level: Applying REF: p. 75

10. A patient who was prescribed a cephalosporin for the treatment of an infection states he has an
allergy to penicillin. What action should you take before giving this drug to the patient?

a. Inform the healthcare provider immediately.

b. Inform the patient that he can take the prescribed drug safely.

c. Instruct the patient to call the healthcare provider only if a rash appears.

d. Instruct the patient to take a dose in your presence to observe for adverse effects.

ANS: A

Patients who are allergic to penicillin are often allergic to the cephalosporins because the
chemical structures are similar. Inform the prescriber about a penicillin allergy.

DIF: Cognitive Level: Applying REF: p. 75

11. A patient receiving antibiotics for 3 days reports a skin rash over the chest, back, and arms.
What is your first action?

a. Ask the patient whether he or she has ever developed a rash while taking another drug.

b. Reassure the patient that many people have this expected reaction to antibiotic therapy.

c. Ask the patient whether the rash itches, burns, or causes other types of discomfort.

d. Document the report as the only action.

ANS: A

A rash is an indication that the patient is allergic to the drug; however, at this time it is not an
emergency.
Authentic First Guru:
Nurse explore For
the patient’s
more of response
thesefurther and hold
materials the dose; then notify the
:masterclasstutor1@gmail.com
prescriber of this problem.

DIF: Cognitive Level: Applying REF: p. 75

12. Which of the following methods is the correct procedure for giving IM penicillin?

a. Inject the penicillin into the dorsal gluteal site by deep IM injection; do not aspirate.

b. Inject the penicillin using the Z-track method; aspirate before injecting the drug.

c. Inject subcutaneously as a bleb; if no reaction, then proceed to inject IM.

d. Penicillin is not permitted to be given by IM injection, but by IV only.

ANS: B

Penicillin is given by the Z-track method and aspiration is performed before injecting the drug. If
blood appears in the syringe, remove the syringe, dispose of the drug, and prepare another dose.

DIF: Cognitive Level: Remembering REF: p. 74

13. Which cell-wall synthesis inhibitor drug is given in an oral form to treat Clostridium difficile?

a. Gabapentin

b. Vancomycin

c. Doxycycline

d. Clindamycin

ANS: B

Vancomycin has an oral form of a cell-wall synthesis inhibitor that is used to combat the
pseudomembranous colitis caused by

Clostridium difficile.

DIF: Cognitive Level: Remembering REF: p. 76

14. A patient is to receive penicillin G benzathine (Bicillin LA) 2,400,000 units intramuscularly.
The drug on hand is penicillin G benzathine 600,000 units/mL. How many milliliters should you
Authentic Nurse Guru: For more of these materials :masterclasstutor1@gmail.com
prepare for the correct dose?

a. 0.25

b. 0.5

c. 2

d. 4

ANS: D

Want 2,400,000 units/have 600,000 units/mL. 2,400,000/600,000 = 24/6 = 4 mL.

DIF: Cognitive Level: Applying REF: p. 74

15. A patient prescribed vancomycin (Vancocin) has developed redness on the face, neck, chest,
back, and arms. The family asks the nurse if the drug should be stopped because of this response.
What is your best answer?

a. “Yes, these problems indicate an allergic reaction.”

b. “Yes, these side effects eventually lead to difficulty breathing.”

c. “No, these uncomfortable problems are an expected drug side effect.”

d. “No, the problems are caused by the presence of the infection and are not related to the drug.”

ANS: C

Vancomycin is a powerful antibacterial drug that has many side effects, including “red man
syndrome.” This problem is caused by a histamine release that dilates blood vessels, giving a red
appearance to the face, neck, chest, back, and arms. Sometimes this reaction can be reduced by
slowing the infusion rate; however, it is not an indication to stop the drug.

DIF: Cognitive Level: Applying REF: p. 76

16. You are counseling a patient who is to begin a course of tetracycline for the treatment of
Lyme disease. What instructions would be important to provide to this patient?

a. “Tetracycline can cause sun sensitivity, so protect yourself from sun exposure.”
Authentic Nurse Guru: For more of these materials :masterclasstutor1@gmail.com
b. “Tetracycline alters blood clotting; avoid injury while taking this drug.”

c. “Tetracycline should be taken with food to avoid stomach upset.”

d. “Tetracycline may cause irritation to the vein when it is injected.”

ANS: A

The tetracyclines increase the sensitivity of the skin to the sun and severe sunburns are possible,
even among people with dark complexions.

DIF: Cognitive Level: Applying REF: p. 77

17. The mother of a 6-year-old child with a skin infection asks why her child is not being treated
with tetracycline as she was for a similar infection. What is your best response?

a. “Tetracycline can impair the development of teeth and bones in young children.”

b. “Tetracycline is a large pill and difficult for a young child to swallow.”

c. “Tetracycline can cause severe headaches in young children.”

d. “Tetracycline can cause seizures in young children.”

ANS: A

Tetracycline interferes with bone development and the development of tooth enamel. Exposure to
tetracycline can cause permanent tooth staining.

DIF: Cognitive Level: Applying REF: p. 77

18. Which statement is true regarding macrolides?

a. Macrolides are the first treatment for MRSA infection.

b. Macrolides always require a longer course of treatment.

c. Macrolides cannot be used for patients with a penicillin allergy.

d. Macrolides are effective against the same organisms as penicillin.

ANS: D
Authentic Nurse Guru: For more of these materials :masterclasstutor1@gmail.com
Macrolides are effective against the same infectious organisms that are sensitive to penicillin and
are used for patients who have a penicillin allergy. These drugs are not effective against MRSA.

DIF: Cognitive Level: Remembering REF: p. 77

19. You are caring for a hospitalized patient receiving gentamycin IV, an aminoglycoside, for the
treatment of Pseudomonas pneumonia. Which laboratory values would be important to monitor to
avoid adverse effects from drugs of this classification?

a. Complete blood count

b. BUN and creatinine

c. Troponin levels

d. Liver enzymes

ANS: B

Patients who are given aminoglycosides must be monitored for nephrotoxicity (kidney damage) as
well as ototoxicity. The nurse should monitor BUN and creatinine during treatment, and report
elevations to the healthcare provider.

DIF: Cognitive Level: Applying REF: p. 79

20. A sulfonamide, Bactrim, is ordered for a patient with a urinary tract infection. What should
you include in the teaching plan for the patient who is taking this drug?

a. Take the drug on a full stomach.

b. Drink milk when taking the drug.

c. Take the drug at night before going to bed.

d. Drink large amounts of water with this drug.

ANS: D

To prevent crystals in the urine, the patient should be told to drink large amounts of water while
taking this drug.

Authentic Nurse Guru: For more of these materials :masterclasstutor1@gmail.com


DIF: Cognitive Level: Applying REF: p. 80

21. You are caring for a patient with type 2 diabetes who has been prescribed a course of a
sulfonamide (Bactrim) for a urinary tract infection. Which statement by the patient reflects
understanding of potential drug interactions?

a. “I will need to watch for symptoms of low blood glucose levels.”

b. “I will need to restrict the amount of fluids I drink to prevent edema.”

c. “I will need to take the Bactrim on an empty stomach to increase drug absorption.”

d. “I will need to add additional salt to my food to replace sodium excreted with this drug.”

ANS: A

Sulfonamides can interact with some type 2 diabetic drugs, such as sulfonylureas that can cause
symptoms of hypoglycemia (fatigue, shakiness, anxiety, and irritability).

DIF: Cognitive Level: Applying REF: p. 80

22. In reviewing a patient’s reported drug allergies, which drug class sensitivity would suggest the
patient should not be given a sulfonamide?

a. Beta blockers

b. ACE inhibitors

c. Thiazide diuretics

d. Calcium channel blockers

ANS: C

Patients with an allergy or sensitivity to thiazide diuretics, oral sulfonylureas, or carbonic


anhydrase inhibitors may exhibit the same allergy or sensitivity to sulfonamides.

DIF: Cognitive Level: Remembering REF: p. 80

23. You are providing education for a patient prescribed a fluoroquinolone. Which response made
by the patient indicates that teaching has been effective?
Authentic Nurse Guru: For more of these materials :masterclasstutor1@gmail.com
a. “There are no drug interactions associated with fluoroquinolones.”

b. “I should take this drug with food to decrease GI upset.”

c. “I can take this drug with caffeine such as a cup of coffee.”

d. “I can discontinue this drug once I feel better.”

ANS: B

Fluoroquinolones should be taken with food to decrease adverse GI effects.

DIF: Cognitive Level: Applying REF: p. 81

24. A patient who is a marathon runner requires treatment with the fluoroquinolone levofloxacin
(Levaquin) for the treatment of a severe infection. What statement by the patient ensures
understanding of the potential adverse effects of this drug?

a. “I will report the development of pain, redness, or swelling around any joint.”

b. “I will call my healthcare provider if I develop abnormal bruising or bleeding.”

c. “I will call my healthcare provider if I develop lower leg swelling.”

d. “I will report the development of a chronic, irritating cough.”

ANS: A

Fluoroquinolones can cause rupture of tendons. Teach patients to notify the healthcare provider
for tendonitis symptoms that might occur (ache, pain, redness, and swelling in a joint or area
where a tendon attaches to a bone).

DIF: Cognitive Level: Applying REF: p. 81

25. A 25-year-old patient presents to the clinic with symptoms of a sinus infection and is sent
home with a prescription for an antibiotic. The LPN/LVN provides the patient with important
information about side effects, adverse effects, and allergic reactions related to antibiotics.

The most common side effects of antibacterial drugs affect the-------------and ------------ include.
The LPN/LVN knows that the patient understands the difference between a side effect and an
adverse effect when she states that she should--------------
Authentic Nurse Guru: For more of these materials :masterclasstutor1@gmail.com
A. digestive system; digestive system; “I should notify my prescriber immediately if

I vomit.” and “It is normal to have changes in skin color while taking this drug.”

B. digestive system; GI upset, vomiting, and diarrhea; “I should call 911 if I experience any
difficulty breathing.” and “I should stop taking the drug if I develop hives or a rash.”

c. GI upset, vomiting, and diarrhea; respiratory system; “I should stop taking the drug if I develop
hives or a rash.” and “I should stop taking the drug if I develop diarrhea.”

d. Shortness of breath, cough, and wheezing; Tachycardia, palpitations, and chest pain; “I should
call 911 if I experience any difficulty breathing.” and “I should stop taking the drug if I develop
diarrhea.”

ANS: B

GI upset, vomiting, and diarrhea are common side effects of antibacterial drugs and are not signs
of allergic reactions. When a patient experiences an allergic reaction, he or she should be taught to
stop taking the drug if rash or hives develop and to call the prescriber immediately. A patient
should also be taught to call 911 immediately for difficulty breathing or a feeling of a lump in the
throat because these are signs of a serious allergic reaction.

DIF: Cognitive Level: Understanding REF: p. 72

26. The LPN/LVN is working in a primary care clinic. A 50-year-old male patient presents with
symptoms of a complicated urinary tract infection (UTI). The provider prescribes the patient
Ciprofloxacin, a fluoroquinolone.

The LPN/LVN knows that fluoroquinolones are ---------------, which means they -------- --. When
instructing the patient, the LPN/LVN knows to stress the importance of taking the drug with------
and to notify the provider immediately if he Experiences --------- --.

A. bactericidal; kill the bacteria; joint pain or swelling

B. limit or slow the growth of the bacteria; bacteribiotic; GI upset

C. kill the bacteria; bacteriostatic; headache

D. restore the normal flora in the body; bacteriostatic; joint pain or swelling
Authentic Nurse Guru: For more of these materials :masterclasstutor1@gmail.com
ANS: A

Ciprofloxacin is an antibiotic in the class of antibiotics called Fluoroquinolones. Fluoroquinolones


are bactericidal, which indicates that they kill bacteria. Fluoroquinolones should be given with
food to ensure absorption. However, they should not be given with dairy, which decreases their
absorption. Some of the most common side effects of fluoroquinolones are nausea, vomiting,
diarrhea, and headache. Tendon rupture is a possible adverse effect of fluoroquinolones, so the
patient should notify the provider immediately if they experience any symptoms of tendonitis,
such as any ache, pain, redness, and/or swelling in a joint or area where a tendon attaches to a
bone.

DIF: Cognitive Level: Understanding REF: p. 81

MULTIPLE RESPONSE

1. A LPN/LVN is teaching a patient about normal flora in the body. The LPN/LVN explains that
the normal flora of humans is extremely complex and that there are more than 200 species of
bacteria throughout the body, including but not limited to the skin, oral cavity, upper and lower
respiratory tract, gastrointestinal tract, and urogenital tract. The patient asks under which
circumstances may the body’s normal flora cause infection? List the correct response by the
LPN/LVN. (Select all that apply.)

a. When the normal flora are within immunocompromised persons

b. When normal flora undergo genetic mutations

c. When the normal flora are exposed to probiotics

d. When normal flora are present in greater amounts than normal

e. When the normal flora are exposed to antibiotics

f. When normal flora present in an unusual place in the body

ANS: A, D, E, F

Conditions in which normal flora cause infection include: when a person has very little immunity,
if the organisms are present in excessive amounts and overwhelm the body, or they are located in
the wrong place. Antibiotic use can upset the normal flora balance in the body and cause yeast or
Authentic Nurse Guru: For more of these materials :masterclasstutor1@gmail.com
fungal infections to occur.

DIF: Cognitive Level: Understanding REF: p. 69

2. Which adverse reactions are associated with the use of broad-spectrum antibiotics? (Select all
that apply.)

a. Bacterial changes in the bowel

b. Overgrowth of yeast

c. Loss of appetite

d. Superinfection

e. Nausea

f. Edema

ANS: A, B, D

Several types of adverse reactions are seen with broad-spectrum antibiotics. Adverse reactions
include: changes in the normal bacteria in the bowel, an overgrowth of yeast, and superinfections.

DIF: Cognitive Level: Remembering REF: p. 70

3. A patient has been taking amoxicillin for a sinus infection for 4 days. She calls the clinic and
states that her symptoms have progressively gotten worse. List the nursing action for the patient’s
care at this time. (Select all that apply.)

a. Ask the patient if she has a fever.

b. Ask the patient if she has any allergies.

c. Ask the patient if she is taking the drug exactly as directed.

d. Notify the healthcare provider.

e. Tell the patient she needs to go to the E.D.

f. Tell the patient to double the dose of her antibiotic.

Authentic Nurse Guru: For more of these materials :masterclasstutor1@gmail.com


ANS: A, C, D

Appropriate nursing actions for this scenario include asking the patient if she is taking the drug
exactly as directed and if the patient confirms this, the healthcare provider should then be notified.
Most of the time it takes 3 to 4 days for patients to feel better. The nurse should also ask the
patient if she has a fever, which indicates worsening infection. If this is the case, the antibiotic
may need to be changed or the healthcare provider may want to see the patient for further
diagnosis.

DIF: Cognitive Level: Applying REF: p. 71

4. The LPN/LVN knows that there are many adverse effects and drug interactions of
fluoroquinolones. There are also certain diseases in which a patient should not receive this type of
drug. The LPN/LVD is educating a patient about possible adverse effects and when to notify the
provider. List the statements that are correct regarding fluoroquinolones. (Select all that apply.)

a. Fluoroquinolones are the antibiotic of choice for patients taking an antidysrhythmic drug.

b. Dairy products and enteral tube feedings reduce the absorption of fluoroquinolones.

c. Fluoroquinolones interact with antidiabetic drugs.

d. Fluoroquinolones may cause photosensitivity.

e. Fluoroquinolones increase the anticoagulant action of warfarin.

f. Fluoroquinolones should be taken 2 hours before or 4 hours after multivitamins, minerals,


antacids, or iron.

ANS: B, C, D, E, F

Nausea, vomiting, diarrhea, abdominal pain, and headache are the most common side effects.
Fluoroquinolones are generally contraindicated in patients on antidysrhythmics because if used
together it can cause life-threatening heart rhythm changes.

Fluoroquinolones should be taken 2 hours before or 4 hours after multivitamins, minerals,


antacids and iron because these agents reduce the absorption of the antibiotic by as much as 90%.
When taken with warfarin, fluoroquinolones increase warfarin’s anticoagulant effects. Dairy
products and enteral tube feedings reduce the absorption of fluoroquinolones. In patients taking
Authentic Nurse Guru: For more of these materials :masterclasstutor1@gmail.com
antidiabetic drugs, hyper or hypoglycemia may occur.

DIF: Cognitive Level: Remembering REF: p. 81

Authentic Nurse Guru: For more of these materials :masterclasstutor1@gmail.com


Chapter 06: Drugs for Tuberculosis, Fungal, and Parasitic Infections
Visovsky: Introduction to Clinical Pharmacology, 10th Edition

MULTIPLE CHOICE

1. A patient newly diagnosed with tuberculosis (TB) asks how this infection is transmitted. What
is your best response?

a. “TB can be transmitted by ingesting contaminated foods.”

b. “TB is transmitted by living in unsanitary conditions.”

c. “TB is transmitted by inhaling droplets from coughing or sneezing.”

d. “TB is transmitted from one person to another by infected blood.”

ANS: C

TB is transmitted by infected humans, cows (bovine), and birds (avian). Droplets ejected during
coughing or sneezing are inhaled by an uninfected host. Once the bacterium is inhaled it rapidly
multiplies in the oxygen-rich lung tissue.

DIF: Cognitive Level: Applying REF: p. 84

2. A patient with a confirmed active tuberculosis (TB) exposure asks how the first-line drugs such
as isoniazid (INH) work to prevent infection. What response demonstrates your understanding of
the action of isoniazid?

a. “INH disrupts blood flow in TB cells depriving them of oxygen.”

b. “INH inhibits enzymes needed for reproduction and growth of TB.”

c. “INH is a bacteriostatic drug that works directly in the lung when inhaled.”

d. “INH works by increasing the elimination of tubercular bacteria through the urine.”

ANS: B

Isoniazid (INH) is a bactericidal drug that inhibits the enzymes of the TB organisms needed for
reproduction and growth. INH can inhibit the enzymes of the TB bacteria that are in an infectious
Authentic Nurse Guru: For more of these materials :masterclasstutor1@gmail.com
as well as a dormant state.

DIF: Cognitive Level: Understanding REF: p. 87

3. Which type of treatment would you expect to be used for a patient with active tuberculosis?

a. Long-term treatment with several antitubercular drugs

b. Prophylactic treatment with isoniazid (INH) only

c. Rifampin for 10 days or until the cough is resolved

d. Ethambutol plus a course of broad-spectrum antibiotics

ANS: A

Antitubercular drugs are classified as primary or secondary agents to describe the way they are
used in treating tuberculosis. The combination of drugs helps to slow the development of bacterial
resistance in active TB.

DIF: Cognitive Level: Remembering REF: p. 85

4. You are teaching a patient prescribed rifampin for the treatment of tuberculosis. What
instruction should be included in the treatment plan?

a. “Avoid taking acetaminophen and alcohol while taking this drug.”

b. “Take this drug with milk or an antacid to avoid stomach upset.”

c. “Your stools may be darker than normal when taking this drug.”

d. “Take this drug with food.”

ANS: A

Rifampin can cause liver damage that is potentiated by acetaminophen and alcohol.

DIF: Cognitive Level: Applying REF: p. 86

5. A patient receiving antibiotics for the treatment of a bacterial wound infection develops a
secondary fungal infection. The patient asks you to explain why the fungal infection developed.
What is your
Authentic NursebestGuru:
response?
For more of these materials :masterclasstutor1@gmail.com
a. “Antibiotics can upset the balance of normal flora and allow yeast or fungus to grow.”

b. “Antibiotic resistance has allowed the bacteria in your wound to become a fungus.”

c. “Genetic changes induced by antibiotics lead to this fungal infection.”

d. “There is no known cause for this secondary fungal infection.”

ANS: A

Antibiotic use can upset the normal flora balance in the body and cause yeast or fungal infections
to occur. Candida is a common body yeast and often overgrows to cause a fungal infection. When
a person is given antibiotics to kill infectious bacteria, the normal flora is killed off as well.

DIF: Cognitive Level: Applying REF: p. 90

6. Which class of antifungal drugs works by altering the cell wall of the fungus?

a. Azoles

b. Polyenes

c. Allylamines

d. Antimetabolites

ANS: A

Azoles work by altering the cellular membrane of the fungus by depleting a lipid-like substance
(ergosterol), which damages the fungus and will not allow it to reproduce.

DIF: Cognitive Level: Remembering REF: p. 91

7. You are providing instructions regarding drug therapy with oral antifungal lozenges to a patient
with an oral Candida infection.

Which statement should be included as part of this patient’s teaching plan?

a. “Monitor your skin for the appearance of rashes or blistering.”

b. “Be sure to use an alcohol-based mouthwash between meals.”

Authentic Nurse Guru: For more of these materials :masterclasstutor1@gmail.com


c. “Take this lozenge with an acidic beverage to increase absorption.”

d. “Brush your teeth and tongue before using the lozenge.”

ANS: D

The patient should be instructed to brush the teeth and tongue to remove as much of the infectious
organism as possible form the area so that the drug is in direct contact with the infected area.

DIF: Cognitive Level: Applying REF: p. 92

8. A female patient with a vaginal fungal infection is reviewing the teaching plan for using a
vaginal antifungal cream. Which statement made by the patient indicates an understanding of the
teaching?

a. “I will place the cream into the vagina before bedtime.”

b. “I will use condoms as birth control during my treatment.”

c. “I will only use the cream until my symptoms resolve.”

d. “I will only have sexual relations in between doses.”

ANS: A

Placing the cream into the vagina before bedtime keeps the drug in the vagina longer. Women
should refrain from sexual intercourse during the treatment. The drug should be used as ordered
for the length of time directed by the healthcare provider.

DIF: Cognitive Level: Applying REF: p. 92

9. A 43-year-old patient with a fungal infection is prescribed griseofulvin. What symptoms should
you monitor that would indicate an adverse effect of the drug?

a. Increased glucose levels

b. Skin redness and blistering

c. Jaundice and dark-colored urine

d. Visual changes
Authentic Nurse Guru: For more of these materials :masterclasstutor1@gmail.com
ANS: C

Griseofulvin can cause liver toxicity. Jaundice, yellow sclera, dark urine, and light-colored stools
should be assessed and reported to the healthcare provider.

DIF: Cognitive Level: Understanding REF: p. 91

10. Your patient has been prescribed a course of fluconazole for the treatment of a fungal
infection. Which symptoms would indicate an adverse reaction to the drug resulting in a medical
emergency?

a. Hypoglycemia and dizziness

b. White plaques coating the tongue

c. Painful red or purple rash, and blisters

d. Decreased hemoglobin and hematocrit

ANS: C

Painful red or purple rash, and blisters with flu-like symptoms are signs of Stevens-Johnso
Syndrome, a medical emergency.

DIF: Cognitive Level: Applying REF: p. 91

11. For which condition would a patient receive the drug primaquine as a treatment?

a. Malaria

b. Pneumonia

c. Tuberculosis

d. Strep infection

ANS: A

Primaquine is a drug used in the treatment of malaria (antimalarial).

DIF: Cognitive Level: Remembering REF: p. 95

Authentic Nurse Guru: For more of these materials :masterclasstutor1@gmail.com


12. A patient is being treated for pinworms with anthelmintics. You have completed teaching this
patient about the drug therapy.

Which statement made by the patient indicates an understanding of the side effects of this
treatment?

a. “I will need to wash the toilet seat weekly.”

b. “I will need to follow a high fiber diet while taking this drug.”

c. “All of my immediate family members will require laboratory testing for this infection.”

d. “I may have diarrhea and abdominal pain while taking this drug.”

ANS: D

Some people have diarrhea and abdominal discomfort while taking the drug. During the initial
period of illness, patients must remember that they are contagious. Every effort must be made to
protect those nearby.

DIF: Cognitive Level: Applying REF: p. 97

13. A patient who is planning to visit a tropical country asks you about taking drugs for malaria.
What is your best response?

a. “Antimalarials are taken only after symptoms of malaria resolve.”

b. “Antimalarials are taken prophylactically when traveling to areas where malaria is common.”

c. “Antimalarials are taken prophylactically when people return from malaria-infested areas.”

d. “Antimalarials are taken for treatment of malaria once the acute illness is over.”

ANS: B

People in the military or those traveling to or living in areas where malaria is endemic can use
antimalarials to prevent malaria and to treat the symptoms.

DIF: Cognitive Level: Applying REF: p. 94

14. A 5-year-old child who is seen at the homeless clinic has a case of active tuberculosis. The
Authentic Nurse Guru: For more of these materials :masterclasstutor1@gmail.com
healthcare provider has prescribed rifampin, pyrazinamide, and ethambutol. Which of the
following side effects of ethambutol would prompt the nurse to discuss the prescription with the
healthcare provider?

a. The drug must be taken with food.

b. The drug is slow-acting.

c. Patient needs to report vision changes.

d. Precipitation of uric acid.

ANS: C

Ethambutol should not be used in infants or young children because they may not be able to report
vision changes that could lead to permanent blindness.

DIF: Cognitive Level: Understanding REF: p. 88

MULTIPLE RESPONSE

1. A homeless 34-year-old male with suspected tuberculosis presents to the local free clinic with a
fever, chills, night sweats, and a cough with bloody sputum. A chest x-ray confirms this diagnosis
and he is prescribed with the first-line therapy of isoniazid (INH), rifampin (Rifadin),
pyrazinamide, and ethambutol (Myambutol). Which instructions should be part of the teaching
plan for all patients receiving treatment with antitubercular drugs? (Select all that apply.)

a. Report weight loss to your healthcare provider.

b. Sleep with your head elevated using two pillows.

c. Avoid alcohol for the duration of this treatment.

d. Report the appearance of dark colored urine and light-colored stools.

e. You will need to have weekly blood draws to assess for neutropenia.

f. Take each drug at a different time throughout the day.

ANS: A, C, D, E

Authentic Nurse Guru: For more of these materials :masterclasstutor1@gmail.com


Weight loss is an indication of worsening disease and should be reported to the healthcare
provider. These drugs can cause liver damage as evidenced by clay-colored stools, dark urine, and
jaundice. As these drugs affect the liver, all alcohols should be avoided.

DIF: Cognitive Level: Applying REF: p. 89

2. The LPN/LVN is taking care of a diabetic patient with a candida infection in the mouth. The
patient has been prescribed nystatin oral suspension. The LPN/LVN is preparing the drug for
administration. Which of the following actions should the LPN/LVN remember when giving this
drug? (Select all that apply.)

a. The LPN/LVN can prepare multiple doses at the same time because the drug contains
preservatives and will stay fresh for a few days.

b. The LPN/LVN should not shake the suspension before pouring it.

c. The LPN/LVN should instruct the patient to keep the suspension in her mouth for several
minutes before swallowing.

d. The LPN/LVN should give this drug to the patient only after the patient consumes a fatty meal,
which increases absorption.

e. The LPN/LVN should check the patient’s pulse before taking this drug and report a heart rate
of 100 or greater.

f. The LPN/LVN should instruct the patient to take half the dose in each side of the mouth so that
the drug reaches all of the oral mucosa.

ANS: C, F

The LPN/LVN should only prepare a single dose of nystatin powder at a time because it does not
contain preservatives and will not stay fresh. Nystatin should be shaken before administration
because the drug separates quickly. The LPN/LVN should instruct the patient to take half the dose
in each side of the mouth so that the drug reaches all of the oral mucosa and covers all oral
tissues.

Patients should be taught to retain the suspension in the mouth for several minutes before
swallowing to ensure the drug comes into contact with the oral fungus.

Authentic Nurse Guru: For more of these materials :masterclasstutor1@gmail.com


DIF: Cognitive Level: Applying REF: p. 91

3. A patient is being prescribed with metronidazole for an amebiasis infection after recent travel
to South America. Before being discharged, the LPN/LVN will instruct the patient about how to
properly take this drug. List the correct nursing instructions. (Select all that apply.)

a. “If the pill is too big to swallow, you can crush, break, or chew the drug.”

b. “Your urine may be darker than usual, which is expected.”

c. “This drug should be taken 1 hour before or 2 hours after meals to ensure best absorption.”

d. “It’s ok to drink alcohol while taking this drug.”

e. “You should take this drug with a full glass of water to support excretion by the kidneys.”

f. “Once you’re feeling better, you can stop taking the drug even if you have pills leftover.”

ANS: B, C, E

Patients should swallow metronidazole extended-release tablets whole and should not be crushed,
broken, or chewed because it could cause the drug to be released too rapidly, which can lead to
adverse effects. It can be shocking, but when taking this drug, it is expected that the patient’s
urine may appear darker than usual or appear reddish-brown in color; however, this pigment
change is nothing to worry about and due to a metabolite in the drug that occurs when the kidneys
break it down for elimination.

Metronidazole should be taken 1 hour before or 2 hours after meals to ensure best absorption.
Metronidazole should be taken with a full glass of water to support excretion by the kidneys. The
patient should not take metronidazole with alcohol or products containing alcohol because severe
side effects can occur. To help clear up the infection, the patient should keep using this drug until
the last pill is taken.

DIF: Cognitive Level: Applying REF: p. 95

Authentic Nurse Guru: For more of these materials :masterclasstutor1@gmail.com


Chapter 07: Drugs for Viral and Retroviral Infections Visovsky: Introduction
to Clinical Pharmacology, 10th Edition

MULTIPLE CHOICE

1. Which statement about antiviral drugs is true?

a. Antiviral drugs kill viruses by altering viral DNA.

b. Antiviral drugs work by stopping viral replication.

c. Antiviral drugs alter the host immune system.

d. Antiviral drugs are considered virucidal.

ANS: B

Antiviral drugs must enter the infected cell and act at the site of infection to be effective.
Antivirals do not kill the virus but rather stop viral reproduction. This action means that all
antivirals are only virustatic, not virucidal.

DIF: Cognitive Level: Remembering REF: p. 101

2. Which antiviral drug is only given by oral inhalation?

a. Acyclovir (Zovirax)

b. Zanamivir (Relenza)

c. Oseltamivir (Tamiflu)

d. Amantadine (Symmetrel)

ANS: B

Zanamivir is given only by oral inhalation. Acyclovir is given intravenously or as an orally


swallowed drug. Oseltamivir and amantadine are swallowed orally.

DIF: Cognitive Level: Remembering REF: p. 102


Authentic Nurse Guru: For more of these materials :masterclasstutor1@gmail.com
3. Which antiviral drug is most likely to have more severe CNS side effects, including worsening
of glaucoma?

a. Acyclovir

b. Amantadine

c. Valacyclovir

d. Zanamivir

ANS: B

Amantadine crosses the blood–brain barrier and concentrates in the brain tissues. None of the
other drugs concentrate in brain tissues.

DIF: Cognitive Level: Remembering REF: p. 102

4. A patient who is asymptomatic comes into the clinic complaining of being exposed to a
coworker who has flu. What drug would be most appropriate for this patient?

a. Flumadine (rimantadine)

b. Valtrex (valacyclovir)

c. Crixivan (indinavir)

d. Ziagen (abacavir)

ANS: A

Flumadine (rimantadine) is used for prophylaxis and treatment of illness caused by various strains
of influenza virus.

DIF: Cognitive Level: Applying REF: p. 101

5. Which drug is the only approved treatment for RSV?

a. Ribavirin (Virazole)

b. Cidofovir (Vistide)

Authentic Nurse Guru: For more of these materials :masterclasstutor1@gmail.com


c. Foscarnet (Foscavir)

d. Lamivudine (Epivir)

ANS: A

The only drug approved for RSV treatment is ribavirin (Virazole).

DIF: Cognitive Level: Remembering REF: p. 103

6. A 2-year-old patient with respiratory syncytial virus is prescribed aerosolized ribavirin


(Virazole). Which visitor should you ensure is not in the patient’s room during the aerosol
treatments?

a. 16-year-old brother

b. 81-year-old grandmother

c. 32-year-old pregnant mother

d. 36-year-old father who has diabetes

ANS: C

A major adverse effect of ribavirin is that it is a teratogen, an agent that has a very high likelihood
of increasing the risks for birth defects and fetal damage. It should not be given to pregnant or
breast-feeding women, and it should not be handled or inhaled by anyone who is pregnant.

DIF: Cognitive Level: Applying REF: p. 103

7. The oral antiviral drug acyclovir (Zovirax) has been ordered for a patient with genital herpes.
What is your best action before giving this drug?

a. Ask the patient if he/she is allergic to milk.

b. Wear a gown, mask, and gloves to give this drug.

c. Assess the patient for the presence of irregular pulse.

d. Inform the patient that this drug can cause excessive thirst.

ANS: A Nurse Guru: For more of these materials :masterclasstutor1@gmail.com


Authentic
Before giving acyclovir ask patients whether they have a true milk allergy because this is a
contraindication to acyclovir therapy as there is cross-reactivity of the antibodies.

DIF: Cognitive Level: Applying REF: p. 102

8. Which of the following is a sign or symptom of an allergic or anaphylactic response to an


antiviral drug?

a. Nausea

b. Insomnia

c. Intolerance to fatty foods

d. Swelling of the face or throat

ANS: D

Allergic and anaphylactic symptoms include swelling of the face or throat, hives, itching, redness,
low blood pressure, feeling a lump in the throat, an irregular heartbeat, a sense that something bad
is happening, and lightheadedness.

DIF: Cognitive Level: Remembering REF: p. 105

9. You are giving a patient who has hepatitis B an oral dose of the antiviral drug adefovir
(Hepsera). What assessment should you perform before giving this drug to the patient?

a. Assess the respiratory rate and pulse.

b. Assess for signs of abnormal bleeding.

c. Assess for yellowing of the skin and sclera.

d. Assess for signs of lower extremity edema.

ANS: C

Assess patients for yellowing of the skin and sclera, and elevated liver enzymes because DNA
polymerase inhibitors are liver toxic.

Liver enzyme levels should also be monitored.


Authentic Nurse Guru: For more of these materials :masterclasstutor1@gmail.com
DIF: Cognitive Level: Applying REF: p. 104

10. A patient who has been taking efavirenz (Sustiva) reports a sore throat, fever, and blisters.
What is your best action?

a. Hold the dose and notify the prescriber.

b. Document the report as the only action.

c. Remind the patient that these are symptoms of opportunistic infection.

d. Reassure the patient that these are common and expected side effects of the drug.

ANS: A

Efavirenz is a non-nucleotide analog reverse transcriptase inhibitor (NNRTI). A sore throat, fever,
different types of rashes, blisters, or multiple bruises are all signs of serious adverse effects of
drugs from this class. The drug should be stopped and the prescriber contacted.

DIF: Cognitive Level: Applying REF: p. 108

11. In what way does a retrovirus differ from other common viruses?

a. Retroviruses lack specialized enzymes.

b. Retroviruses use DNA instead of RNA in their genes.

c. Retroviruses cannot leave the cell to infect new cells.

d. Retroviruses transmit their own information into the cell’s DNA.

ANS: D

Retroviruses are organisms that differ from viruses in that instead of merely hijacking a cell’s
DNA or RNA to reproduce, they transmit their own information into the cell’s DNA.

DIF: Cognitive Level: Understanding REF: p. 106

12. Which statement when made by a HIV positive patient informs you that the patient needs
additional teaching?

a. “I mayNurse
Authentic be moreGuru:
susceptible
For to opportunistic
more of these infections.”
materials :masterclasstutor1@gmail.com
b. “By taking the antiretrovirals over time, I will be cured of HIV.”

c. “I understand that I may take several different drugs for my HIV.”

d. “Antiretrovirals interfere with the ability of the retrovirus to reproduce.”

ANS: B

Once a person has become infected with HIV he or she will have the virus for life. All
antiretroviral drugs are virustatic rather than virucidal. None of these drugs kill the virus.

DIF: Cognitive Level: Applying REF: p. 106

13. Which drug classification works to limit the progression of HIV?

a. Antiretrovirals

b. Antivirals

c. Antibiotics

d. Antifungals

ANS: A

Antiretrovirals are an important group of drugs that slow the growth or prevent the duplication of
retroviruses; they are used to limit the advance of HIV and AIDS.

DIF: Cognitive Level: Remembering REF: p. 107

14. A patient diagnosed with HIV asks you about opportunistic infections. What is your best
response?

a. “This infection is a toxic effect of the virus.”

b. “This infection is from drug-related toxicities.”

c. “This infection is a first-generation metabolic infection.”

d. “This infection is from damage to the body’s immune system.”

ANS: D
Authentic Nurse Guru: For more of these materials :masterclasstutor1@gmail.com
An opportunistic infection develops because of the damage to the body’s immune system, leaving
the body unable to protect itself from certain other infections.

DIF: Cognitive Level: Applying REF: p. 106

15. How do protease inhibitor (PI) drugs prevent viral replication?

a. They are counterfeit bases that prevent reverse transcriptase from synthesizing the

DNA needed for viral replication.

b. They inactivate the enzyme that allows the viral genetic material to be integrated into the
human host’s cellular DNA.

c. They prevent the production of proteins needed for viral particles to leave the cell and infect
other cells.

d. They prevent initial infection by blocking the receptor the virus uses to enter target cells.

ANS: C

Protease inhibitors prevent viral replication and release of viral particles. Human
immunodeficiency virus produces its proteins, including those needed to move viral particles out
of the host cell, in one long (HIV) strand. For the proteins to be active, this large protein must be
broken down into separate smaller proteins through the action of the viral enzyme HIV protease.
Proteaseinhibitors, when taken into an HIV-infected cell, make the protease enzyme work on the
drug rather than on the initial large protein. Thus active proteins are not produced and viral
particles cannot leave the cell to infect other cells.

DIF: Cognitive Level: Understanding REF: p. 110

16. You are teaching a patient about the action of a newly prescribed antiretroviral drug. Which
statement explains the action of antiretrovirals?

a. “These drugs act by boosting the body’s T-cell production.”

b. “These drugs act by interfering with replication of the retrovirus.”

c. “These drugs act by using the DNA of the host cell kill the virus.”

Authentic Nurse Guru: For more of these materials :masterclasstutor1@gmail.com


d. “These drugs act by producing artificially acquired active immunity.”

ANS: B

Antiretroviral agents act to stop more retroviruses from being made by interfering with the ability
of a retrovirus to reproduce, or replicate.

DIF: Cognitive Level: Applying REF: p. 107

17. A patient beginning antiretroviral therapy for HIV infection asks you why it is important to
report all other drugs that the patient is taking. What is your best response?

a. “The antiretroviral and other drugs must be taken together.”

b. “The antiretroviral and other drugs must be taken with milk.”

c. “The antiretroviral should be taken with water 30 minutes before eating.”

d. “The antiretroviral should be taken alone and not with other drugs.”

ANS: D

Most antiretrovirals react with other drugs. Antiretroviral drugs inhibit the cytochrome P-450
enzyme system involved in the metabolism of drugs in the liver. For this reason, they should not
be taken at the same time as other drugs.

DIF: Cognitive Level: Applying REF: p. 112

18. A female with HIV who is taking a combination of a nucleoside reverse transcriptase
inhibitor, non-nucleoside reverse transcriptase inhibitor, and protease inhibitor tells you she is
now pregnant. What advice regarding her drug therapy would be appropriate?

a. “Keep taking the drug regimen exactly as ordered throughout your pregnancy.”

b. “Stop the drug regimen during the first trimester, as this is when the fetus forms.”

c. “This drug regimen is only given the last trimester to protect the baby during birth.”

d. “Keep taking the drug regimen as prescribed until delavirdine or efavirenz is added.”

ANS: A
Authentic Nurse Guru: For more of these materials :masterclasstutor1@gmail.com
Antiretroviral drugs (except delavirdine or efavirenz) are recommended to be taken by pregnant
women who are known to be HIV positive because the virus can cross the placenta and infect the
fetus. These drugs, when taken as prescribed, can reduce the chances of fetal infection from about
30% to about 8%.

DIF: Cognitive Level: Applying REF: p. 110

19. You suspect that a patient on antiretroviral therapy may have drug-related toxicity. Which of
the following laboratory tests would alert you to a drug toxicity?

a. Elevated glucose level

b. Decreased hemoglobin

c. Elevated liver enzymes

d. Decreased amylase and lipase

ANS: C

Most antiretrovirals can also cause damage to the liver or kidneys (hepatotoxic or nephrotoxic).

DIF: Cognitive Level: Understanding REF: p. 111

20. A patient taking ART therapy for HIV infection develops severe upper abdominal pain that
radiates to the back. What complication of this therapy may this patient be experiencing?

a. Pancreatitis

b. Kidney failure

c. Vertebral fracture

d. Acute gallbladder infection

ANS: A

Upper abdominal pain and/or pain that radiates to your back, pain that worsens after eating, fever,
rapid pulse, increased nausea, and vomiting may indicate a pancreatitis which is a medical
emergency.

Authentic Nurse Guru: For more of these materials :masterclasstutor1@gmail.com


DIF: Cognitive Level: Understanding REF: p. 111

21. A patient with HIV infection tells you that she has been only partially compliant with ART
drugs because she is tired of taking so many pills. What is your best response?

a. “I will let your healthcare provider know so your regimen can be changed.”

b. “You need only take the drug regimen 50% of the time for it to be effective.”

c. “You can stop the drugs for a short 2-week ‘drug holiday’ then resume them.”

d. “You should always take your drug therapy as prescribed or it may not be effective.”

ANS: D

These drugs must be taken exactly as ordered every day to ensure the drugs work properly and to
avoid drug resistance. Taking too little of the drugs or skipping doses leads to drug resistance and
disease advancement. It is imperative not to skip doses or decrease the dosage. To be most
effective in preventing HIV infection and slowing HIV reproduction, ART drugs must be taken
correctly and on time at least 90% of the time.

DIF: Cognitive Level: Applying REF: p. 111

22. A patient who has been taking subcutaneous enfuvirtide (Fuzeon) for 6 months reports
reduced sensation in the fingers and toes.

What is your best action?

a. Document the report as the only action.

b. Hold the dose and notify the prescriber.

c. Remind the patient to take a multiple vitamin daily.

d. Reassure the patient that this is an expected drug side effect.

ANS: D

Peripheral neuropathy with loss of sensation in the extremities is a common and expected side
effect of therapy. Drug therapy is not stopped for this effect. The patient needs to implement
precautions to prevent injury from not having full sensation for touch, temperature, and pressure.
Authentic Nurse Guru: For more of these materials :masterclasstutor1@gmail.com
DIF: Cognitive Level: Applying REF: p. 111

23. You are teaching a patient who has been prescribed atazanavir (Reyataz) about this drug.
What should be included in this patient’s teaching plan?

a. “Take this drug only with fruit juice.”

b. “Be sure to chew this drug thoroughly.”

c. “Take your pulse daily before taking this drug.”

d. “Change positions carefully as this drug can lower your blood pressure.”

ANS: C

Teach patients taking atazanavir and ritonavir to check their pulse daily and report low heart rate
to the prescriber because these two drugs can impair electric conduction and lead to heart block.

DIF: Cognitive Level: Applying REF: p. 109

24. A diabetic patient who is receiving treatment for HIV with raltegravir (Isentress) asks you
how this drug may affect his diabetes.

What is your best response?

a. “This drug will not interact with your diabetic drugs.”

b. “You will need to increase your carbohydrate intake while on this drug.”

c. “Monitor your glucose levels, as this drug can increase blood glucose levels.”

d. “Keep hard candy with you at all times, as this drug can cause hypoglycemia.”

ANS: C

Teach patients with diabetes to closely monitor blood glucose levels because these drugs increase
hyperglycemia.

DIF: Cognitive Level: Applying REF: p. 109

25. A patient presents to the clinic with flu symptoms that began 3 days ago. The patient is
wondering
Authentic why the
Nurse flu cannot
Guru: be treated
For more of with
thesean antiviral.
materials Which are the LPN/LVN’s best
:masterclasstutor1@gmail.com
response?

a. “Antiviral drugs are not useful in combating influenza.”

b. “Influenza is treated with antibiotics, not antivirals.”

c. “Antiviral drugs are only used for prevention of influenza.”

d. “Antiviral drugs work best when given within 48 hours of the start of symptoms.”

e. “Antivirals can only be used on drug-resistant influenza.”

f. “Antiviral drugs can only be given to people who are immunocompromised.”

ANS: D

All the antiviral drugs for influenza are used to either prevent an infection in a patient who has
been exposed to the virus or to reduce the symptoms of an existing influenza infection. They work
best when given after exposure and before symptoms start or within 48 hours of the onset of
symptoms.

DIF: Cognitive Level: Applying REF: p. 101

MULTIPLE RESPONSE

1. Which drug classes are used in the treatment of hepatitis B virus? (Select all that apply.)

a. Interferon

b. Uncoating inhibitors

c. Neuraminidase inhibitors

d. DNA polymerase inhibitors

e. Nucleoside reverse transcriptase inhibitors

f. Non-nucleoside reverse transcriptase inhibitors

ANS: A, D, E

The major drugs used to treat HBV fall into three classes, the nucleoside reverse transcriptase
Authentic Nurse Guru: For more of these materials :masterclasstutor1@gmail.com
inhibitors (NRTIs), the DNA polymerase inhibitors, and interferon.

DIF: Cognitive Level: Remembering REF: p. 104

2. A patient diagnosed with HIV infection taking antiretroviral therapy appears less depressed and
more talkative. The patient tells you the drugs seem to be working, and she is certain she is now
cured. What are your best responses? (Select all that apply.)

a. “You started the drug early enough to be cured.”

b. “Taking St. John’s Wort will help you feel even better.”

c. “Taking the drug prevents more virus from being produced.”

d. “You are not cured of HIV; however, you will not be able to spread the virus.”

e. “You are having a good response to the drugs, so you can stop them for a while.”

f. “HIV is not currently curable, but drug therapy slows the advance of the disease.”

ANS: C, F

Antiretrovirals do not cure HIV infection, but taking the drug regimen as prescribed prevents
resistant strains of HIV and slows the progression of the disease.

DIF: Cognitive Level: Applying REF: p. 106

Authentic Nurse Guru: For more of these materials :masterclasstutor1@gmail.com


Chapter 08: Drugs for Allergy and Respiratory Problems Visovsky:
Introduction to Clinical Pharmacology, 10th Edition

MULTIPLE CHOICE

1. A patient with allergies has been prescribed an antihistamine. Which response by the patient
demonstrates understanding of the action of antihistamine drugs?

a. “Antihistamines act by blocking histamine receptors in tissues.”

b. “Antihistamines act by increasing dilation of blood vessels.”

c. “Antihistamines act by blocking the release of histamine.”

d. “Antihistamines act by increasing the production of acetylcholine.”

ANS: A

Antihistamines do not block the release of histamine, but rather, block histamine receptors in
target tissues which in turn limit the blood vessel vasodilation, capillary leak, swelling, and
bronchoconstriction.

DIF: Cognitive Level: Applying REF: p. 115

2. An older adult patient with glaucoma reports taking an antihistamine for allergy symptoms.
What adverse effect of antihistamines would you alert this patient about?

a. Ototoxicity

b. Ocular “floaters”

c. Excessive tearing

d. Increased intraocular pressure

ANS: D

A rising intraocular pressure (pressure inside the eye) in patients with glaucoma can worsen the
disease and could cause blindness.

DIF: Cognitive Level: Applying REF: p. 118


Authentic Nurse Guru: For more of these materials :masterclasstutor1@gmail.com
3. A 43-year-old male patient with hypertension has stated that he is taking an over-the-counter
antihistamine every day for seasonally allergies. In reviewing all the drugs this patient is taking,
what statement should be included in his teaching plan?

a. “Take your antihistamine 2 hours before your other prescription drugs.”

b. “Antihistamines can cause your blood pressure to rise.”

c. “Antihistamines can cause a slowing of the pulse rate.”

d. “Antihistamines must be taken at the same time as antihypertensives.”

ANS: B

A rise in blood pressure is an adverse action of antihistamines. Patients should be instructed to


report all OTC drugs to their healthcare providers, as adverse interactions can occur.

DIF: Cognitive Level: Applying REF: p. 118

4. A patient taking a decongestant for seasonal allergies asks you to explain how a decongestant
works. What is your best response?

a. “Decongestants act by decreasing inflammation within the respiratory tract.”

b. “Decongestants act in the respiratory tract to decrease mast cell activity.”

c. “Decongestants reduce allergy symptoms, but to not reduce inflammation.”

d. “Decongestants act by reducing the immune response of lymphocytes.”

ANS: C

Decongestants are a class of drugs that are used to control the symptoms associated with allergy.
They have no anti-inflammatory action.

DIF: Cognitive Level: Applying REF: p. 115

5. An adolescent with asthma is taking an inhaled anti-inflammatory drug and asks you what side
effects can be expected with this inhaled drug. What is your best response?

a. “You may experience some mild anemia with this drug.”


Authentic Nurse Guru: For more of these materials :masterclasstutor1@gmail.com
b. “You may experience some bronchoconstriction after the first dose of this drug.”

c. “You may experience mild itching of the mucous membranes with this drug.”

d. “You may experience dryness of the mouth with this drug.”

ANS: D

Anti-inflammatory drugs reduce inflammation and dry all mucous membranes including those in
the mouth and throat. They would reduce any cutaneous (skin) itching. They do not work on
bronchial smooth muscles or the bone marrow (which produces red blood cells).

DIF: Cognitive Level: Applying REF: p. 124

6. Which patient would not be a candidate for a leukotriene inhibitor as part of the treatment plan
for mild asthma?

a. An adult patient with a history of liver disease

b. An older patient with cognitive impairment

c. An adolescent with allergic rhinitis

d. A patient with depression

ANS: A

Liver dysfunction is possible with long-term use. The leukotriene inhibitors interact with drugs
that stimulate liver metabolism such as phenytoin, phenobarbital, and carbamazepine, and
rifampin.

DIF: Cognitive Level: Understanding REF: p. 117

7. An adolescent with a history of asthma will need a drug to prevent exercise-induced asthma
while playing high school sports.

Which drug would be appropriate for this use?

a. Cromolyn sodium

b. Theochron
Authentic Nurse Guru: For more of these materials :masterclasstutor1@gmail.com
c. Sus-Phrine

d. Proventil

ANS: A

Cromolyn sodium is used for prophylaxis in treating asthma. It works by slowing down the
destruction of sensitized mast cells and inhaled specific antigens. This drug is taken daily as it
does not act rapidly.

DIF: Cognitive Level: Remembering REF: p. 119

8. You are teaching a patient with newly diagnosed asthma about using the prescribed drugs to
manage acute asthma symptoms.

Which of the following statements made by the patient would indicate your teaching was
effective?

a. “I will keep my short-acting bronchodilating inhaler with me at all times.”

b. “I will take my cromolyn sodium 5 minutes after asthma symptoms begin.”

c. “I will take montelukast (Singulair) at the first sign of an asthma attack.”

d. “I will take my decongestant and corticosteroid nasal spray each day.”

ANS: A

Teach patients with asthma to always have their short-acting beta2-adrenergic agonists (SABAs)
reliever drug with them at all times because an attack can occur anywhere and only a SABA can
work fast enough to prevent a severe attack and death.

DIF: Cognitive Level: Applying REF: p. 124

9. A patient has been prescribed fluticasone (Flonase) to control swelling of the nasal mucosa due
to allergies. Which statement made by the patient alerts you to a possible adverse effect of this
drug?

a. “My nasal passages appear to be dry.”

b. “I have a white patch in my throat that is sore.”


Authentic Nurse Guru: For more of these materials :masterclasstutor1@gmail.com
c. “I have been experiencing a fast heartbeat after taking this drug.”

d. “I feel sleepy most of the day.”

ANS: B

Fluticasone is a nasal corticosteroid that can reduce the local immune response of the patient, and
a secondary fungal infection may occur.

DIF: Cognitive Level: Applying REF: p. 120

10. A patient with asthma who has been prescribed a long-acting beta-adrenergic agonist asks
why this drug is not useful during an actual acute asthma attack. What is your best response?

a. “Long-acting beta agonists reduce inflammation rather than relax bronchial smooth.”

b. “Long-acting beta agonists can worsen an asthma attack.”

c. “Long-acting beta agonists need time to build up an effect.”

d. “Long-acting beta agonists take too long to be absorbed by the intestinal tract.”

ANS: C

Long-acting beta2-adrenergic agonists (LABAs) work in the same way as SABAs but need time
to build up an effect. Therefore,

LABAs are used to prevent an asthma attack because their effects last longer but have no value
during an acute attack.

DIF: Cognitive Level: Applying REF: p. 124

11. Which of the following patients is most at risk for overdose from the oral mucolytic drug
guaifenesin?

a. A 38-year-old patient who had surgery 4 days ago

b. A 60-year-old with a history of COPD

c. An adolescent with a history of asthma

d. A 1-year-old
Authentic Nursechild withFor
Guru: a cold and cough
more of these materials :masterclasstutor1@gmail.com
ANS: D

Because of cases of overdosage, the FDA recommends that these products not be given to
children under 2 years old and some products not to children under 6 years old.

DIF: Cognitive Level: Applying REF: p. 128

12. Which statement made by the patient demonstrates understanding of the use of oral
mucolytics?

a. “I will not use a humidifier while taking this drug.”

b. “This drug will stop me from coughing.”

c. “I will drink at least 2 quarts of water while taking this drug.”

d. “This drug will cause my urine to change color.”

ANS: C

Patient instructions should include the use of a humidifier and instruct the patient to drink at least
2 quarts of water daily while taking a mucolytic unless there is a medical reason for fluid
restriction. These actions will help get the mucus out.

DIF: Cognitive Level: Applying REF: p. 128

13. What information should be included in the teaching plan for a patient who is prescribed an
antitussive agent?

a. “This drug contains codeine but is not habit forming.”

b. “This drug can be taken for long periods of time.”

c. “This drug can cause drowsiness.”

d. “Do not give this drug to adults, as this drug is for children’s use only.”

ANS: C

Side effects of antitussives include drowsiness and dizziness. Antitussives should be used only for
short periods of time because they can be addictive. They are prescribed for adults.
Authentic Nurse Guru: For more of these materials :masterclasstutor1@gmail.com
DIF: Cognitive Level: Applying REF: p. 130

14. A patient with a severe cough has been prescribed a codeine-based antitussive. What
statement made by this patient would alert you that the patient is experiencing a side effect of the
drug?

a. “My appetite has been greatly increased lately.”

b. “I have been having difficulty sleeping.”

c. “My fingertips feel a bit numb.”

d. “I have been experiencing constipation lately.”

ANS: D

Codeine-containing antitussives may cause severe constipation.

DIF: Cognitive Level: Applying REF: p. 129

15. You are teaching a patient with severe asthma prescribed both a short acting beta-adrenergic
agonist (SABA) and an inhaled corticosteroid. Which statement made by the patient demonstrates
understanding of how to use these drugs?

a. “Never take these two inhalers together.”

b. “Take the SABA before the inhaled corticosteroid.”

c. “Take the inhaled corticosteroid before the SABA.”

d. “Take the SABA on alternate days only.”

ANS: B

When both a SABA and an inhaled corticosteroid are ordered, instruct the patient to take the
SABA first to relax the smooth muscle, opening the airway so the corticosteroid can reach the
respiratory tract.

DIF: Cognitive Level: Applying REF: p. 124

MULTIPLE RESPONSE
Authentic Nurse Guru: For more of these materials :masterclasstutor1@gmail.com
1. You are teaching an older adult patient about the antihistamine that was prescribed for the
treatment of allergies. Which instructions would be appropriate to include? (Select all that apply.)

a. Antihistamines can cause drowsiness.

b. Avoid alcohol while taking antihistamines.

c. Drink additional fluids when taking antihistamines.

d. Antihistamines can lower blood pressure in older patients.

e. Take antihistamines around the clock to stabilize drug blood levels.

f. Older patients can develop memory problems when using antihistamines.

ANS: A, B, C, F

Most antihistamines cause drowsiness, so additional sedatives and alcohol should be avoided. If
not contraindicated, additional fluids counteract the dryness of mucous membranes experienced
with antihistamines. These drugs should only be taken when needed, and can raise the blood
pressure in older adults. Older adults can develop memory problems due to antihistamine use.

DIF: Cognitive Level: Applying REF: p. 118

2. You are planning a teaching session for a patient with allergies prescribed a mast cell stabilizer.
List the indications for this drug’s use. (Select all that apply.)

a. Asthma prophylaxis

b. Management of allergies

c. Pneumonia prophylaxis

d. Management of allergic rhinitis

e. Management of postexercise bronchospasm

f. Management of acute asthma exacerbation

g. Management of cough

h. Decrease
Authentic respiratory
Nurse Guru:secretions
For more of these materials :masterclasstutor1@gmail.com
ANS: A, B, D, E

A mast cell stabilizer is effective for the management of allergies, allergic rhinitis, and for asthma
prophylaxis. It is also useful in patients with asthma who have post-exercise bronchospasm. This
drug should not be used to manage acute attacks of asthma, or pneumonia.

DIF: Cognitive Level: Applying REF: p. 119

3. A 32-year old male patient diagnosed with asthma has received teaching regarding the expected
side effects from his newly prescribed inhaled beta2-adrenergic agonist. List the patient responses
that demonstrates understanding of the teaching session. (Select all that apply.)

a. “I may have a bad taste in my mouth.”

b. “I can expect to have increased salivation.”

c. “I may experience difficulty sleeping.”

d. “I may have a rapid heart rate after taking this drug.”

e. “I may get dizzy if I get up too quickly.”

f. “I may experience occasional diarrhea.”

g. “I may develop urinary retention.”

h. “I may experience some tremors.”

ANS: A, C, D, H

Beta2-adrenergic agonists have actions similar to the sympathetic division of the autonomic
nervous system. When these drugs are used heavily, they can have systemic effects, which include
rapid heart rate, tremors, increased blood pressure, a feeling of nervousness, and difficulty
sleeping. The inhaled drugs can dry the mouth and throat and also may leave a bad taste in the
mouth.

DIF: Cognitive Level: Applying REF: p. 124

4. Which side effects should you monitor for after a patient receives ipratropium (Atrovent)?
(Select all that apply.)
Authentic Nurse Guru: For more of these materials :masterclasstutor1@gmail.com
a. Eye pain

b. Sore throat

c. Stuffy nose

d. Constipation

e. Decreased heart rate

f. Increased urine output

ANS: B, C, D

Ipratropium (Atrovent) is a cholinergic antagonist drug. Cholinergic antagonists are associated


with sore throat, stuffy nose, and constipation. Patients should be educated to expect these
common side-effects.

DIF: Cognitive Level: Understanding REF: p. 125

Authentic Nurse Guru: For more of these materials :masterclasstutor1@gmail.com


Chapter 09: Drugs Affecting the Renal/Urinary and Cardiovascular Systems
Visovsky: Introduction to Clinical Pharmacology, 10th Edition

MULTIPLE CHOICE

1. Which body system works with the renal/urinary system to maintain fluid balance and remove
waste products from the body?

a. Biliary system

b. Cardiovascular system

c. CNS

d. Gastrointestinal system

ANS: B

The renal/urinary system works together with the cardiovascular system to maintain adequate
circulation to all parts of the body. The interactions between these two systems help maintain
fluid balance, delivery of nutrients, and removal of waste products from cells, tissues, and organs.
Most drugs that affect one system have an effect on the functioning of the other system.

DIF: Cognitive Level: Remembering REF: p. 134

2. A patient with newly diagnosed hypertension is prescribed a thiazide diuretic. The patient asks
how thiazide diuretic drugs reducehigh blood pressure. What is your best response?

a. “Thiazide diuretics reduce blood volume and relax vascular smooth muscle in the arterioles,
reducing blood pressure.”

b. “Thiazide diuretics increase potassium excretion and slow heart contractions, reducing blood
pressure.”

c. “Thiazide diuretics release sodium from cellular storage sites to reduce blood pressure.”

d. “Thiazide diuretics increase the reabsorption of sodium, potassium and chlorides, reducing
blood pressure.”

ANS: A
Authentic Nurse Guru: For more of these materials :masterclasstutor1@gmail.com
Blood pressure is maintained by fluid volume in circulation, the force of heart contractions, and
the constriction of blood vessels that provide resistance. All diuretics help lower blood pressure
by reducing the amount of fluid (volume) in the blood. Thiazide and thiazide-like diuretics also
relax blood vessel smooth muscle, which decrease vascular resistance to blood flow. The reduced
volume and decreased resistance result in lower blood pressure.

DIF: Cognitive Level: Applying REF: p. 134

3. You are teaching a patient about the side effects of diuretics. Which health problem is a
potential side effect or adverse reaction of any class of diuretic drug?

a. Dehydration

b. Heart failure

c. Blurred vision

d. Urinary tract infection

ANS: A

Diuretics work by increasing the excretion of water through urination. When water loss is
excessive, dehydration of the circulatory system or even the whole body can occur. Diuretic
therapy helps improve the symptoms of heart failure and do not cause blurred vision. Although a
urinary tract infection can occur in a patient taking a diuretic, the drug is not the direct cause.

DIF: Cognitive Level: Understanding REF: p. 134

4. A patient has been prescribed furosemide (Lasix) for the treatment of heart failure. Which
statement made by the patient indicates the need for more teaching?

a. “Since I work at night, I take my drug when I first wake up rather than in the morning.”

b. “If my hearing decreases, I will notify my healthcare provider.”

c. “I make sure to space my fluid intake evenly throughout the day.”

d. “When I travel long distances, I will plan to skip my diuretic that day.”

ANS: D
Authentic Nurse Guru: For more of these materials :masterclasstutor1@gmail.com
Although taking the time to urinate while traveling is inconvenient, the patient needs to
understand that the drug must be taken every day. Often, patients are told to take the drug in the
morning; it should be taken at the time furthest away from when the patient usually goes to bed so
there is less interference with his or her normal sleep patterns. Spacing fluid intake evenly
throughout the day helps prevent dehydration. Loop diuretics can cause hearing loss and the
patient should notify the healthcare provider if this develops even though his or her health
problem may require that the drug be continued.

DIF: Cognitive Level: Applying REF: p. 138

5. The LPN is caring for a patient who has recently been prescribed nitrate drug. The patient
reports a throbbing headache. Which response is the most appropriate?

a. “Throbbing headache is a symptom of an allergic response to the drug.”

b. “Throbbing headache is a symptom of developing tolerance to the drug.”

c. “Throbbing headache is a temporary symptom that should disappear within a few weeks.”

d. “Throbbing headache is a symptom of interaction with your other drugs.”

ANS: C

Throbbing headache is a transient side effect of nitrate drugs that should disappear. This symptom
does not indicate an allergic response, development of tolerance to the drug, or interaction with
the patient’s other drugs.

DIF: Cognitive Level: Applying REF: p. 155

6. Which of the following side effects is seen in patients taking angiotensin-converting enzyme
inhibitors but rarely seen in patients taking angiotensin II receptor blockers?

a. Orthostatic hypotension

b. Cough

c. Dizziness

d. Hypotension

Authentic Nurse Guru: For more of these materials :masterclasstutor1@gmail.com


ANS: B

A dry, hacking cough is often associated with angiotensin-converting enzymes. Patients who
experience cough can be switched to angiotensin II receptor blockers.

DIF: Cognitive Level: Remembering REF: p. 150

7. Why is it important to warn a patient prescribed to take finasteride (Proscar) for benign
prostatic hyperplasia to wear a condom when having sex with a pregnant woman or one who
could become pregnant?

a. The drug contains a hormone that can greatly increase the risk for early birth when absorbed by
a pregnant woman.

b. The drug can be absorbed by the pregnant woman and cause birth defects in the fetus.

c. The drug reduces local immunity and increases the risk for transmitting HIV.

d. The drug increases the risk for vaginal infections in pregnant women.

ANS: B

The drug is a synthetic hormone that can be absorbed through the skin and mucous membranes.
When the fetus is exposed, birth defects are possible. The drug does not increase the risk for any
type of infection development or transmission. The drug does not increase the risk for having
twins.

DIF: Cognitive Level: Understanding REF: p. 139

8. The LPN is obtaining a patient history on a patient who is scheduled to begin treatment with a
nitrate drug for the treatment of angina. Which information is most important for the LPN to
obtain?

a. The patient’s future plans to become pregnant

b. The patient’s past use of recreational drugs

c. The patient’s sexual history

d. The patient’s description of anginal pain


Authentic Nurse Guru: For more of these materials :masterclasstutor1@gmail.com
ANS: D

The patient’s description of anginal pain will affect which drug the patient should be prescribed.
While future plans of pregnancy, past use of recreational drugs, and sexual history are important
to obtain, the description of the pain helps determine which nitrate is best for this patient.

DIF: Cognitive Level: Remembering REF: p. 156

9. A pregnant nurse is scheduled to care for a patient benign prostatic hypertrophy. Which of the
following drugs listed here should not be handled by a pregnant nurse?

a. Finasteride (Proscar) B

b. Oxybutynin (Ditropan)

c. Tamsulosin (Flomax)

d. Tolterodine (Detrol)

ANS: A

Finasteride is a DHT inhibitor used to treat benign prostatic hypertrophy. Pregnant women should
not handle or touch these drugs because they can cause birth defects when absorbed through the
skin. Tamsulosin is a selective alpha-1 blocker that also treats benign prostatic hyperplasia.
Oxybutynin and tolterodine are both urinary antispasmodics.

DIF: Cognitive Level: Understanding REF: p. 139

10. You are teaching a 75-year-old male patient who has angina about nitroglycerine sublingual
tablets. Which patient statement indicates an accurate understanding of the correct way to store
nitroglycerin?

a. “I will need to keep my nitroglycerine in the refrigerator.”

b. “I can keep my nitroglycerin together with my other pills in my pillbox.”

c. “I should keep my nitroglycerine in the original container.”

d. “I should keep my nitroglycerin stored on the kitchen countertop.”

ANS: C
Authentic Nurse Guru: For more of these materials :masterclasstutor1@gmail.com
Nitroglycerin should be stored in the original dark glass container. Storing nitroglycerin with
other pills allows the nitrate to escape. Nitroglycerin should not be exposed to light.

DIF: Cognitive Level: Applying REF: p. 157

11. You are caring for a patient with angina in the home who develops chest pain. The patient has
a recently filled prescription for sublingual nitroglycerin that has never been taken before. You
give the patient the nitroglycerine sublingually to relieve the angina pain. What action should you
take until an ambulance arrives?

a. Take the patient’s blood pressure every 5 minutes.

b. Assess the patient’s lung sounds.

c. Raise the patient’s legs above the level of the heart.

d. Place the patient in supine position.

ANS: A

The nurse should carefully evaluate the patient’s blood pressure every 5 minutes after giving
nitrates to assess for hypotension as the patient’s response to nitrates is unknown.

DIF: Cognitive Level: Applying REF: p. 156

12. The LPN/VN is teaching a patient who has had a myocardial infarction (MI) about using
sublingual NTG tablets for the onset of chest pain. Which of the following statements by the
patient indicates that he or she understands the teaching?

a. “I need to chew the NTG tablet thoroughly.”

b. “I will swallow the pill like all of my other heart pills.”

c. “I will place the NTG under my tongue when I have chest pain.”

d. “I can take my NTG with applesauce to disguise the metallic taste.”

ANS: C

The sublingual form of NTG is only effective if placed under the tongue for absorption.
Swallowing the pill results in the drug being destroyed by first-pass metabolism so no drug would
Authentic Nurse Guru: For more of these materials :masterclasstutor1@gmail.com
reach the circulation. The tablet should not be chewed or mixed with food.

DIF: Cognitive Level: Applying REF: p. 156

13. The nurse is instructing the patient about a statin drug. Which statement indicates a need for
further instruction?

a. “I can drink beer only while taking this drug since it has a lower alcohol content.”

b. “I will need to have my cholesterol checked in 4 to 12 weeks after starting this drug.”

c. “I will report severe muscle aches or changes in my urine.”

d. “I will avoid grapefruit juice while taking this drug.”

ANS: A

Patients need to avoid alcohol because drinking alcohol puts stress on the liver and adds to the
stress of these drugs. Patients need to avoid grapefruit juice while taking statins because grapefruit
juice increases the concentration of statins, and increases the risk of toxicity. Patients need to be
instructed to report severe muscle aches, changes in urine color, or decreased urine output because
statins can cause rhabdomyolysis, a disorder resulting from broken muscle cells that damage the
kidneys. Patients need to be reminded to follow up with their healthcare provider to check
cholesterol levels 4 to 12 weeks after starting the drug and after dose changes.

DIF: Cognitive Level: Understanding REF: p. 143

14. The nurse is caring for a patient who is receiving a loop diuretic for the treatment of heart
failure. Which statement indicates that the patient has an accurate understanding of nutritional
requirements associated with loop diuretics?

a. “I should increase the amount of protein in my diet by eating peanut butter.”

b. “I should increase the amount of calcium in my diet by eating dairy products.”

c. “I should increase the amount of fiber in my diet by eating raw vegetables.”

d. “I should increase the amount of potassium in my diet by eating foods like bananas.”

ANS: D
Authentic Nurse Guru: For more of these materials :masterclasstutor1@gmail.com
Visovsky: Introduction to Clinical Pharmacology, 10th Edition 10

Loop diuretics cause a loss of potassium. Patients should be advised to eat potassium-rich foods
including bananas, citrus fruits, dried beans and lentils, and all-bran cereal. Cardiotonic drugs do
not require the patient to alter intake of protein, calcium, or fiber.

DIF: Cognitive Level: Applying REF: p. 135

15. A patient contacts the LPN at the healthcare provider’s office and tells the nurse that she
forgot to take her daily dose of antihypertensive drug that was due 4 hours ago. How should the
nurse advise the patient?

a. “Take the next dose of drug at the regularly scheduled time.”

b. “Double the next regularly scheduled dose of drug.”

c. “Take the drug with a full glass of orange juice.”

d. “Take a dose of drug now to avoid a missed dose.”

ANS: D

If the patient missed her daily dose of an antihypertensive and does not remember until 4 hours
later, she should go ahead and take the missed dose. She should not double the next dose or
increase the dose in any way. Taking the drug with orange juice does not address what the patient
should do about missing a dose of drug. She should only take the missed dose if she remembers
within 8 hours of the next scheduled administration time.

DIF: Cognitive Level: Applying REF: p. 155

16. The LPN is obtaining a health history on a 52-year-old male patient who has been receiving
treatment with an antihypertensive drug who decided to stop taking the drug. Which question is
most important for the nurse to ask the patient?

a. “Do you wear a MedicAlert bracelet?”

b. “Have you experienced any sexual side effects?”

c. “Do you store your drug away from children?”

d. “Do you remember to only salt your food after it is cooked?”

Authentic Nurse Guru: For more of these materials :masterclasstutor1@gmail.com


Visovsky: Introduction to Clinical Pharmacology, 10th Edition 10

ANS: B

The nurse should ask the patient first about side effects. Patients may not report new or
uncomfortable sexual side effects, and these effects may cause patients to struggle with drug
compliance. Wearing a MedicAlert bracelet and proper storage are important considerations when
taking antihypertensives but are lower in priority than side effects. The patient should avoid high-
sodium foods and should not salt food during or after cooking.

DIF: Cognitive Level: Applying REF: p. 155

17. A patient has been prescribed phenazopyridine as part of treatment for a urinary tract
infection. Which manifestation warrants the nurse’s immediate intervention?

a. Reddish-orange urine

b. Vertigo

c. Nausea and vomiting

d. Yellow sclera

ANS: D

Yellow sclera can indicate jaundice. Jaundice is not an expected side effect of phenazopyridine
and should be reported immediately to the healthcare provider. Reddish-orange urine, vertigo, and
GI upset are expected side effects of phenazopyridine.

DIF: Cognitive Level: Understanding REF: p. 139

18. A nurse is caring for a patient with a history of coronary artery disease who reports chest pain
6/10. After obtaining the patient’s vital signs and giving nitroglycerin, the nurse carefully
monitors the patient for which adverse reaction to the drug?

a. Tachypnea

b. Bradycardia

c. Hypotension

d. Hypothermia
Authentic Nurse Guru: For more of these materials :masterclasstutor1@gmail.com
Visovsky: Introduction to Clinical Pharmacology, 10th Edition 10

ANS: C

Nitroglycerin acts as a vasodilator which lowers blood pressure. The nurse should monitor the
patient for a hypotensive reaction. Nitroglycerin should not increase respirations or lower heart
rate or temperature.

DIF: Cognitive Level: Understanding REF: p. 156

19. The nurse is caring for a patient who presents to the emergency department with atrial
fibrillation. Once stabilized, she is discharged home with a prescription for an oral calcium
channel blocker. Which drug does the nurse anticipate that the healthcare provider will prescribe?

a. Metoprolol (Lopressor)

b. Verapamil (Calan)

c. Quinidine (Quinaglute)

d. Disopyramide (Norpace)

ANS: B

Verapamil is an oral calcium-channel blocker indicated in the treatment of supraventricular


tachycardia including atrial fibrillation. Metoprolol, a selective beta blocker is indicated in the
treatment of cardiac dysrhythmias, angina, and myocardial infarction. Disopyramide and
quinidine, sodium channel blockers, are used to treat supraventricular and ventricular
dysrhythmias.

DIF: Cognitive Level: Understanding REF: p. 149

20. The home health nurse makes a visit to a patient with chronic heart failure who is receiving
digoxin therapy. The nurse obtains the following vital signs: blood pressure 142/92 mm Hg,
respirations 26 breaths/min, apical pulse 54 beats/min, and temperature

98.4 F. The patient reports dizziness and weakness. What action should the nurse take next?

a. Verify that the patient received the flu vaccine.

b. Reassure the patient that these symptoms are expected with digoxin therapy.

Authentic Nurse Guru: For more of these materials :masterclasstutor1@gmail.com


Visovsky: Introduction to Clinical Pharmacology, 10th Edition 10

c. Encourage the patient to get more rest.

d. Call the primary healthcare provider and request an order for blood work.

ANS: D

The nurse should obtain an order from the healthcare provider for lab work to rule out digoxin
toxicity. Toxic effects of digoxin can include anorexia, nausea, vomiting, diarrhea, weakness,
fatigue, vision changes, confusion, dizziness, and bradycardia.

DIF: Cognitive Level: Understanding REF: p. 162

MULTIPLE RESPONSE

1. A 62-year-old male patient is in the hospital for an exacerbation related to congestive heart
failure (CHF). The patient is taking furosemide (Lasix), a loop diuretic. The nurse is reviewing a
nursing care plan for this patient. List the actions the nurse will include for a patient on diuretics.
(Select all that apply.)

a. Monitor intake and output.

b. Weigh every day at the same time.

c. Remind to stand up slowly.

d. Encourage to eat a diet high in sodium.

e. Monitor serum potassium levels.

ANS: A, B, C, E

The patient should have a diet high in potassium unless it is a potassium-sparing diuretic. Monitor
intake and output to make sure that the patient achieves fluid balance. Monitor potassium levels to
assess that they are within normal levels because these drugs reduce blood potassium levels.
Report potassium levels below normal to the healthcare provider because low potassium levels can
have serious effects on muscles and breathing. Remind the patient to stand up slowly to avoid
Authentic Nurse Guru: For more of these materials :masterclasstutor1@gmail.com
Visovsky: Introduction to Clinical Pharmacology, 10th Edition 10

orthostatic hypotension. Weigh the patient daily while they are taking diuretics to monitor trends
and prevent dehydration.

DIF: Cognitive Level: Applying REF: p. 136

2. A patient is to begin antihypertensive therapy with a thiazide diuretic. Which of the following
adverse effects are common with this type of diuretic? (Select all that apply.)

a. Hyperuricemia

b. Hypokalemia

c. Hypotension

d. Hypoglycemia

e. Hypercalcemia

ANS: A, B, C, E

Thiazide diuretics can cause an increase in uric acid which leads to the risk of gout in patients
with a history of gout. It can also cause hypokalemia; hypotension and hypercalcemia are also
possible. Hyperglycemia is a risk rather than hypoglycemia so diabetes should be monitored
carefully.

DIF: Cognitive Level: Understanding REF: p. 136

3. A 65-year-old female admitted for supraventricular tachycardia is transferred to your floor and
has been prescribed sotalol 80 mg orally every 12 hours to begin today. What adverse effects
should you be alert for? (Select all that apply.)

a. Decreased heart rate

b. Increased respiratory rate

c. Symptoms of depression

d. Increased blood glucose levels

Authentic Nurse Guru: For more of these materials :masterclasstutor1@gmail.com


Visovsky: Introduction to Clinical Pharmacology, 10th Edition 10

e. Symptoms of immune suppression

f. Decreased thyroid gland function

ANS: A, C, D

Sotalol is a beta blocker. Beta blockers can cause decreased heart rate and blood pressure,
increased blood glucose in diabetics, and can result in symptoms of depression.

DIF: Cognitive Level: Remembering REF: p. 151

Authentic Nurse Guru: For more of these materials :masterclasstutor1@gmail.com


Visovsky: Introduction to Clinical Pharmacology, 10th Edition 10

Chapter 10: Drugs for Central Nervous System Problems Visovsky:


Introduction to Clinical Pharmacology, 10th Edition

MULTIPLE CHOICE

1. Which neurotransmitter has an inhibitory action within the CNS?

a. Epinephrine

b. Acetylcholine

c. Norepinephrine

d. Gamma-aminobutyric acid

ANS: D

Neurotransmitters can be excitatory or inhibitory. Excitatory neurotransmitters include


acetylcholine (Ach), epinephrine, and norepinephrine. Inhibitory neurotransmitters include
dopamine, some types of serotonin, and GABA (gamma-aminobutyric acid).

DIF: Cognitive Level: Remembering REF: p. 167

2. A 73-year-old male patient newly diagnosed with Parkinson’s disease (PD) is prescribed a
dopamine agonist. The patient’s wife asks you to explain how this class of drugs will work to help
her husband. What is your best response?

a. “Dopamine agonists reduce levels of acetylcholine to restore gait and balance.”

b. “Dopamine agonists use an enzyme to break down dopamine to improve PD symptoms.”

c. “Dopamine agonists prevent the buildup of beta amyloid plaques to improve coordination.”

d. “Dopamine agonists reduce muscle rigidity and tremors by increasing dopamine levels.”

ANS: D

Dopamine agonists work to increase dopamine levels in the brain to restore the balance between
the actions of acetylcholine and dopamine. This action reduces muscle rigidity and tremors, to
improve mobility.
Authentic Nurse Guru: For more of these materials :masterclasstutor1@gmail.com
Visovsky: Introduction to Clinical Pharmacology, 10th Edition 10

DIF: Cognitive Level: Applying REF: p. 169

3. A patient placed on the drug tolcapone (Tasmar) for the relief of symptoms of Parkinson’s
disease asks you how this drug works for the disease. What is your best response?

a. “Tolcapone acts as a dopamine agonist, replacing the activity of dopamine.”

b. “Tolcapone forces the substantia nigra to increase production of natural dopamine.”

c. “Tolcapone suppresses the enzyme that breaks down naturally occurring dopamine.”

d. “Tolcapone increases the number of dopamine receptors sites.”

ANS: C

Catechol-O-methyltransferase (COMT) is an enzyme that breaks down (metabolizes) naturally


occurring catecholamine-based neurotransmitters, including dopamine. It also breaks down
dopamine agonist drugs. Catechol-O-methyltransferase (COMT) inhibitors are drugs that suppress
the activity of the COMT enzyme so both naturally occurring dopamine and dopamine agonist
drugs remain active in the body longer, helping to restore the acetylcholine–dopamine balance in
the brain. Entacapone (Comtan) and tolcapone (Tasmar) are two COMT inhibitors that are used
currently for PD.

DIF: Cognitive Level: Applying REF: p. 173

4. Why are carbidopa and levodopa usually given together?

a. Carbidopa is a dopamine agonist and levodopa is a dopamine antagonist.

b. Levodopa is a dopamine agonist and carbidopa is a dopamine antagonist.

c. Carbidopa enhances the action of levodopa, and less drug is needed.

d. Levodopa reverses or prevents the side effects of carbidopa.

ANS: C

Carbidopa is usually given in combination with levodopa because it enhances the levodopa so
lower doses of levodopa can be used, thus preventing the nausea and vomiting that accompanies
the continual increasing of the levodopa dose to control disease symptoms.
Authentic Nurse Guru: For more of these materials :masterclasstutor1@gmail.com
Visovsky: Introduction to Clinical Pharmacology, 10th Edition 10

DIF: Cognitive Level: Understanding REF: p. 171

5. When assessing a patient with Parkinson’s disease who takes a carbidopa/levodopa


combination drug, you find that he now has almost constant muscle movements that look like
uncoordinated dancing. What is your best action?

a. Document the finding as the only action.

b. Hold the next drug dose and report the finding to the healthcare provider.

c. Give the next dose earlier than scheduled because the drugs are wearing off.

d. Request that the healthcare provider prescribe a one-time dose of a muscle relaxant.

ANS: B

The most common adverse reaction to carbidopa/levodopa is dyskinesia, involuntary muscle


movements that look like uncoordinated dance movements. Dyskinesia is common in patients on
long-term carbidopa/levodopa therapy (longer than 3 years).

Usually, this adverse effect requires the healthcare provider to adjust the drug therapy for
Parkinson’s disease.

DIF: Cognitive Level: Applying REF: p. 171

6. A patient newly diagnosed with Parkinson’s disease is prescribed an oral dopamine agonist.
Which precaution is most important to teach the patient and family about the timing for taking
this drug?

a. “Take the drug 30 to 60 minutes before meals on an empty stomach.”

b. “Take the drug 30 to 60 minutes after eating a high protein meal.”

c. “Take the drug first thing in the morning before getting out of bed.”

d. “Take the drug when your symptoms are at their worst.”

ANS: A

Authentic Nurse Guru: For more of these materials :masterclasstutor1@gmail.com


Visovsky: Introduction to Clinical Pharmacology, 10th Edition 10

Dopamine agonists should be taken 30 to 60 minutes before a meal, so patients have an easier
time swallowing. An empty stomach is best to enhance absorption. Patients must be taught to
avoid taking the drug with or shortly after eating protein because protein reduces the effectiveness
of these drugs.

DIF: Cognitive Level: Applying REF: p. 170

7. You are assessing the drug list of a patient who has been started on a dopamine agonist. You
discover the patient is also taking an oral antihypertensive drug. Which of the following
statements should you make to this patient?

a. “Taking an antihypertensive and a dopamine antagonist will increase your blood pressure.”

b. “Taking an antihypertensive and a dopamine antagonist will decrease your blood pressure.”

c. “Taking an antihypertensive and a dopamine antagonist will have no adverse effects.”

d. “Taking an antihypertensive and a dopamine antagonist will reduce your dopamine levels.”

ANS: B

Patients who are taking both a dopamine agonist and an antihypertensive can experience severe
hypotension.

DIF: Cognitive Level: Applying REF: p. 172

8. A patient with diagnosed Parkinson’s disease is prescribed a monoamine oxidase type B


inhibitor reports not adhering to a tyramine-restricted diet. What signs/symptoms would you
expect this patient to present with?

a. Hypertension

b. Severe bradycardia

c. Bone marrow failure

d. Edema of the lower extremities

ANS: A

Tyramine-rich foods should be avoided as they can precipitate a severe hypertensive crisis.
Authentic Nurse Guru: For more of these materials :masterclasstutor1@gmail.com
Visovsky: Introduction to Clinical Pharmacology, 10th Edition 11

DIF: Cognitive Level: Applying REF: p. 174

9. When asking the family of a patient with Alzheimer’s disease who takes the drug donepezil
(Aricept) what other prescribed or over-the-counter drugs the patient takes, they list all the
following drugs. Which one will you tell them to stop giving the patient?

a. Aspirin

b. Buspirone

c. Vitamin C

d. Dextromethorphan

ANS: D

Dextromethorphan, a common over-the-counter cough drug, can cause long Q-T syndrome. When
taken with donepezil, a fatal dysrhythmia known as torsade de pointe, a form of ventricular
tachycardia, can occur as a result of the interaction.

DIF: Cognitive Level: Applying REF: p. 176

10. When admitting a new patient with Alzheimer’s to a memory unit of a long-term care facility,
you note that she is prescribed both memantine and rivastigmine. What is your best action?

a. Give both drugs as prescribed.

b. Ask the patient when she usually takes these drugs.

c. Notify the prescriber that both drugs are for Alzheimer’s disease.

d. Give the memantine one odd-numbered days and rivastigmine on even-numbered days.

ANS: A

Although both drugs are used for Alzheimer’s disease, they have very different actions and both
can be used at the same time. Memantine is usually given with rivastigmine and other cholinergic
agonists because it increases the effectiveness of these other drugs.

DIF: Cognitive Level: Applying REF: p. 176

Authentic Nurse Guru: For more of these materials :masterclasstutor1@gmail.com


Visovsky: Introduction to Clinical Pharmacology, 10th Edition 11

11. A family member of a patient using the rivastigmine patches reports that the patient keeps
taking the patches off his chest. Where will you suggest the family member apply the patches to
avoid this problem?

a. On the forehead

b. On the buttocks

c. On the upper or lower back

d. On the outer aspect of the thigh

ANS: C

Apply the patch to areas that the patient cannot see and would have a hard time reaching. The
recommended area is the upper or lower back to avoid removal by the patient.

DIF: Cognitive Level: Applying REF: p. 176

12. A patient is prescribed to receive memantine extended release (XR) 14 mg orally once daily.
You have on hand memantine XR 7-mg capsule and memantine 28-mg capsule. What is the best
way to ensure the patient gets a 14-mg dose?

a. Give the patient two 7-mg capsules every day.

b. Give the patient one 28-mg capsule every other day.

c. Cut the 28-mg capsule in half and give the patient one of the halves.

d. Open a 28-mg capsule, empty it into a drug cup, divide the contents in half, and give one half to
the patient.

ANS: A

It is suggested that in general, extended release capsules not be opened. However, for some drugs,
such as memantine, a capsule can be opened, and the entire contents sprinkled on food if the
patient has difficulty swallowing the capsule whole. If it is not possible to divide the contents well
enough to ensure an accurate dose, the best action is to give two 7-mg capsules to equal the 14-mg
prescribed dose.

Authentic Nurse Guru: For more of these materials :masterclasstutor1@gmail.com


Visovsky: Introduction to Clinical Pharmacology, 10th Edition 11

DIF: Cognitive Level: Understanding REF: p. 176

13. When assessing a patient before starting the first dose of a newly prescribed antiepileptic drug,
what is the most important nursing action to perform?

a. Determine the type of aura a patient usually has before a seizure.

b. Obtain an accurate weight because most drug dosages are based on weight.

c. Ask the patient about all other prescribed or over-the-counter drugs he or she takes daily.

d. Ensure that oxygen and suction equipment are in the patient’s room and in good working order.

ANS: C

All actions are reasonable and helpful. The most important assessment information is determining
all other drugs the patients take.

Antiepileptic drugs have many drug interactions that can lead to adverse reactions.

DIF: Cognitive Level: Applying REF: p. 179

14. You are about to give a patient hospitalized for seizures a dose of intravenous (IV) phenytoin.
The patient asks why this drug is given by IV instead of by intramuscular (IM) injection. What is
your best response?

a. “The rapid IM absorption can cause tachycardia.”

b. “Giving phenytoin IM is very irritating to tissue”.

c. “Seizure activity is worsened by giving phenytoin by IM injection.”

d. “When phenytoin is given by IM route, drug elimination is impaired.”

ANS: B

Giving phenytoin by the IM route results in pain and discomfort (and sometimes damaged tissue)
because it is a severe tissue irritant. The best routes are oral and intravenous.

DIF: Cognitive Level: Applying REF: p. 180

15. A patient
Authentic Nurse youGuru:
are caring
Forformore
is scheduled for elective
of these surgery.
materials When reviewing the
:masterclasstutor1@gmail.com
Visovsky: Introduction to Clinical Pharmacology, 10th Edition 11

preoperative laboratory results of a patient taking oxcarbazepine for seizure control, you note all
of the following values below. For which one will you notify the surgeon immediately?

a. White blood cell count 8700/mm3

b. Serum sodium level of 128 mEq/L

c. International normalized ratio (INR) 0.9

d. Serum chloride level of 100 mEq/L

ANS: B

The serum sodium level is well below normal (136 to 145 mEq/L), which means the patient has
hyponatremia. This problem is a common side effect of oxcarbazepine and must be corrected
before surgery to prevent serious complications.

DIF: Cognitive Level: Applying REF: p. 182

16. When teaching a parent of a young child prescribed lamotrigine for epilepsy, which statement
is related to an important precaution that should be included in the teaching plan?

a. “Give this drug to your child at night only to prevent dizziness.”

b. “Ensure that your child takes a multi vitamin daily while on this drug.”

c. “Call your healthcare provider immediately if a rash develops.”

d. “Call your healthcare provider for a weight loss of more than 2 lbs in one week.”

ANS: C

Lamotrigine can cause life-threatening rashes (including Stevens-Johnson syndrome and toxic
epidermal necrolysis). It has a black box warning that states to discontinue the drug immediately
if any rash appears during treatment. Although this problem can occur at any age, it is more likely
to occur in children. Although the drug interferes with the formation of folic acid, an important
vitamin, and some patients may become folic acid deficient, a typical multiple vitamin does not
contain enough folic acid to prevent this problem. Weight loss is not associated with this drug.

DIF: Cognitive Level: Applying REF: p. 183


Authentic Nurse Guru: For more of these materials :masterclasstutor1@gmail.com
Visovsky: Introduction to Clinical Pharmacology, 10th Edition 11

17. Which drug to manage multiple sclerosis should be avoided by patients who also have
epilepsy?

a. Beta-interferon

b. Daclizumab

c. Dalfampridine

d. Tecfidera

ANS: C

Dalfampridine lowers the seizure threshold and increases the risk for seizure activity.

DIF: Cognitive Level: Knowing REF: p. 188

18. What is the most important precaution to teach patients taking any monoclonal antibody or
neurologic drug to manage multiple sclerosis?

a. “Avoid crowds and people who are sick.”

b. “Always wear protective clothing or sunscreen when outdoors.”

c. “Report any weight loss immediately to your healthcare provider.”

d. “Rest as much as possible and avoid any weight-bearing activities.”

ANS: A

The monoclonal antibodies and neurologic drugs currently prescribed to help manage multiple
sclerosis all reduce immunity and inflammation, increasing the risk for infection.

DIF: Cognitive Level: Applying REF: p. 188

19. Which of the following drugs for multiple sclerosis is specifically prescribed to improve
walking?

a. Beta-interferon

b. Daclizumab

Authentic Nurse Guru: For more of these materials :masterclasstutor1@gmail.com


Visovsky: Introduction to Clinical Pharmacology, 10th Edition 11

c. Dalfampridine

d. Tecfidera

ANS: C

Dalfampridine is a drug that is specifically used to improve walking in patients with multiple
sclerosis. Multiple sclerosis is a disease that attacks the CNS and causes nerves to not work as
well resulting in communication problems between the brain and the rest of the body.
Dalfampridine strengthens the signals between the brain and the rest of the body and the nerves
that were damaged from the disease.

DIF: Cognitive Level: Remembering REF: p. 188

MULTIPLE RESPONSE

1. The LPN/LVN is taking care of a patient with Parkinson’s disease who is taking levodopa. The
LPN/LVN knows that there are certain symptoms to watch out for that an indicate that the drug is
“wearing off?” (Select all that apply.)

a. Decreased muscle tone

b. Dry mouth

c. Slower gait

d. Difficulty swallowing

e. Increased appetite

f. Increased tremors

ANS: C, D, F

Levodopa is a dopamine agonist that restores balance between excitatory and inhibitory input to
motor responses. When it wears off, the symptoms of Parkinson’s disease return or become
worse. These include muscle movements that are hard to control and jerky with rigidity because
they fail to relax sufficiently. The gait becomes slow and shuffling with short steps. Other
common symptoms of PD include tremors, stooped posture, difficulty stopping motion once it has
started, difficulty chewing and swallowing, and drooling.
Authentic Nurse Guru: For more of these materials :masterclasstutor1@gmail.com
Visovsky: Introduction to Clinical Pharmacology, 10th Edition 11

DIF: Cognitive Level: Applying or higher REF: p. 163

2. Which changes are most important to assess for in a patient who is taking topiramate for
seizure control? (Select all that apply.)

a. Weight loss

b. Decreased seizure activity

c. Warm flushed skin

d. Insomnia

e. Pain at the IV site

f. Decreased heart rate

ANS: B, C, F

Patients taking any antiepileptic drug should be assessed for changes in seizure activity to
determine drug effectiveness. Topiramate can cause metabolic acidosis and the patient should be
assessed for its symptoms on a regular basis. These symptoms include slow heart rate,
hypotension, muscle weakness, and warm, flushed skin.

DIF: Cognitive Level: Applying REF: p. 183

3. The LPN/LVN is working at an Assisted Living Facility and taking care of a patient with
Parkinson’s disease. The patient is currently taking carbidopa/levodopa. The LPN/LVN is
teaching a nursing student about this drug. Which of the following statements by the LPN/LVN
are true regarding this drug. (Select all that apply.)

a. “This drug may be crushed or chewed if the patient has difficulty swallowing the pill whole.”

b. “This drug should be taken with 6 to 8 ounces of water at least 30 to 60 minutes before meals to
maximize absorption and decrease the risk of aspiration.”

c. “After using for several years, this drug can have a ‘wearing off’ effect which causes rapid
swings of symptoms.”

d. “When starting this drug, it is given at the highest dose to get to the maximum therapeutic
Authentic Nurse Guru: For more of these materials :masterclasstutor1@gmail.com
Visovsky: Introduction to Clinical Pharmacology, 10th Edition 11

effect as quickly as possible.”

e. “This drug should be given with a high-protein snack to increase absorption.”

f. “It’s important for the patient to change positions slowly while taking this drug because it can
cause hypotension.”

ANS: B, C, F

Carbidopa/levodopa should not be crushed or chewed and should be swallowed intact. This drug
should be given with 6 to 8 ounces of water at least 30 to 60 minutes before eating to maximize
absorption, as well as to have maximum effect so the patient has less difficulty chewing and
swallowing, which lessens the risk for aspiration. It can be given with a no protein snack to avoid
nausea. This drug should not be given with high-protein foods because they will decrease
absorption. All dopamine agonists have the risk of postural hypotension, so it is important to
instruct patients to get up and change positions slowly. These agents are usually started at the
lowest possible dose and increased gradually until the maximum therapeutic effect is reached.

DIF: Cognitive Level: Applying REF: p. 170

Authentic Nurse Guru: For more of these materials :masterclasstutor1@gmail.com


Visovsky: Introduction to Clinical Pharmacology, 10th Edition 11

Chapter 11: Drugs for Mental Health Visovsky: Introduction to Clinical


Pharmacology, 10th Edition

MULTIPLE CHOICE

1. The healthcare provider is considering prescribing a drug for a patient with insomnia. Which
drug category is helpful in promoting sleep among patients with insomnia?

a. Sedatives

b. ACE inhibitors

c. Atypical antipsychotics

d. Selective serotonin reuptake inhibitors

ANS: A

Sedatives are drugs that have the main purpose of promoting sleep by changing signals in the
CNS and reducing responses to stimulation.

DIF: Cognitive Level: Remembering REF: p. 195

2. You are preparing to give the benzodiazepine Valium to a patient as a premedication before
surgery. The patient asks you how this drug works. What is your best response?

a. “This drug works by binding to receptors acting with GABA to induce sleep.”

b. “This drug acts by preventing binding to the benzodiazepine receptors.”

c. “This drug acts by inhibiting GABA responses to relax muscles.”

d. “This drug acts by binding to opioid receptors.”

ANS: A

Benzodiazepines are sedating hypnotic drugs that depress the CNS by binding to benzodiazepine
(BNZ) receptors that act with gamma-aminobutyric acid (GABA) receptors to enhance GABA
effects, resulting sleep and muscle relaxation.

DIF: Cognitive Level: Applying REF: p. 196


Authentic Nurse Guru: For more of these materials :masterclasstutor1@gmail.com
Visovsky: Introduction to Clinical Pharmacology, 10th Edition 11

3. You are caring for a patient in the clinic setting who has been taking a benzodiazepine
alprazolam (Xanax) for anxiety. What potential problem should you observe this patient for?

a. Seizure activity

b. Dependence

c. Insomnia

d. Anorexia

ANS: B

The benzodiazepines have a higher risk for physical dependence as compared to


nonbenzodiazepines. They also carry a black box warning for CNS depression. Overdoses are
possible and serious.

DIF: Cognitive Level: Understanding REF: p. 196

4. The family of a patient who has been brought to the emergency room suspects the patient took
an accidental overdose of Which drug would you anticipate will be given to a patient who
experienced an overdose of a benzodiazepine?

a. Rifampin

b. Naloxone

c. Flumazenil

d. Epinephrine

ANS: C

The drug flumazenil (Romazicon) is a benzodiazepine receptor antagonist and is an antidote used
to reverse an overdose of either a benzodiazepine sedative or a nonbenzodiazepine sedative. For
adults, it is given intravenously with an initial dose of 0.2 mg. If there is no response after 45
seconds, the dose can be repeated. After that, it can be repeated every minute for a total of 4
doses.

DIF: Cognitive Level: Remembering REF: p. 198


Authentic Nurse Guru: For more of these materials :masterclasstutor1@gmail.com
Visovsky: Introduction to Clinical Pharmacology, 10th Edition 12

5. For which patient would benzodiazepines be contraindicated?

a. A female with breast cancer

b. A woman who is 20 weeks pregnant

c. A male who has a history of smoking

d. A male who is about to undergo surgery

ANS: B

Pregnancy is an absolute contraindication for the benzodiazepines because they have a high risk
for causing birth defects. Although the chemical structure of the non-benzodiazepines
(benzodiazepine agonists) is different, these drugs bind to the same receptors and have similar
actions. As a result, they are not recommended during pregnancy.

DIF: Cognitive Level: Applying REF: p. 198

6. You are planning an educational session with a patient who is taking a benzodiazepine agonist.
What specific information should you include as part of the teaching plan?

a. “These drugs can interact with many foods and beverages.”

b. “These drugs can cause you to be physically active at night.”

c. “These drugs must be takes for at least 6 weeks to see any effects.”

d. “Excess caffeine intake can work to increase the effectiveness of these drugs.”

ANS: B

When taking a non-benzodiazepine (benzodiazepine agonist), be aware that drugs from this class
can cause you to be physically active at night, even going for a drive, without your knowledge or
memory of the event. It is best to have a family member or friend watch out for these effects when
you first start taking the drug.

DIF: Cognitive Level: Applying REF: p. 197

7. You are preparing to teach a patient with anxiety disorder about the actions the benzodiazepine
agonist buspirone. What would youinform the patient about how this drug works?
Authentic Nurse Guru: For more of these materials :masterclasstutor1@gmail.com
Visovsky: Introduction to Clinical Pharmacology, 10th Edition 12

a. “Buspirone acts by inhibiting GABA pathways to decrease anxiety.”

b. “Buspirone acts by affecting the serotonin and dopamine neurotransmitters.”

c. “Buspirone acts by affecting the monoamine oxidase pathway.”

d. “Buspirone acts by inhibiting the action of dopamine receptors.”

ANS: B

A newer drug from the non-benzodiazepine class, buspirone, reduces anxiety through a variety of
actions affecting the serotonin and dopamine neurotransmitters.

DIF: Cognitive Level: Applying REF: p. 199

8. A patient you are interviewing tells you that the healthcare provider will be prescribing a
benzodiazepine. Which of the following drugs is classified as a benzodiazepine?

a. Lorazepam (Ativan)

b. Buspirone (BuSpar)

c. Benztropine (Cogentin)

d. Chlorpromazine (Thorazine)

ANS: A

Lorazepam (Ativan) is a benzodiazepine.

DIF: Cognitive Level: Remembering REF: p. 197

9. A patient taking an antidepressant tells you the drug is not working and has decided to stop
taking it. What is your best response?

a. “If you are not receiving the benefits, you can stop the antidepressant right away.”

b. “You should not stop taking the antidepressant abruptly.”

c. “You can stop the antidepressant if you have reached a satisfactory drug blood level.”

d. “You should consider doubling the dose to obtain the desired effect.”
Authentic Nurse Guru: For more of these materials :masterclasstutor1@gmail.com
Visovsky: Introduction to Clinical Pharmacology, 10th Edition 12

ANS: B

Antidepressants should not be stopped abruptly as the patient may experience withdrawal
symptoms or recurrence of depression. These drugs require tapering by the healthcare provider.

DIF: Cognitive Level: Applying REF: p. 201

10. You are teaching a patient about the adverse effects associated with selective serotonin
reuptake inhibitors (SSRIs). Which adverse effect would you inform your patient about?

a. Metallic taste

b. Sexual dysfunction

c. Cardiovascular disease

d. Extrapyramidal symptoms

ANS: B

Expected side effects of SSRI’s include sexual side effects in men and women that include
decreased sex drive and decreased ability to orgasm and erectile dysfunction. Other adverse
effects include nausea (during the first 2 weeks), drowsiness, insomnia, dry mouth, decreased
appetite, increased sweating, and constipation.

DIF: Cognitive Level: Applying REF: p. 202

11. A patient who has been prescribed a serotonin-norepinephrine reuptake inhibitor (SNRI) one
week ago reports that she has not noticed any difference in her depression level since beginning
this drug. What is your best response?

a. “Begin taking two tablets daily to increase blood serum levels.”

b. “Add a tablet of St, John’s Wort to increase the drug’s effects.”

c. “You should call your healthcare provider immediately.”

d. “It may take several weeks for this drug to take effect.”

ANS: D

Authentic Nurse Guru: For more of these materials :masterclasstutor1@gmail.com


Visovsky: Introduction to Clinical Pharmacology, 10th Edition 12

The patient should see effects within a few weeks of taking the drug, with maximum effects at 6
to 8 weeks.

DIF: Cognitive Level: Applying REF: p. 203

12. A young adult patient taking an SSRI for the past 2 weeks comes in for an evaluation and
follow-up appointment. Which potential side effect should you check for in this patient?

a. Suicidal thoughts

b. Psychotic episodes

c. Delusional thinking

d. Schizo-affective disorders

ANS: A

SSRIs may cause thoughts of suicide, most likely in children and young adults. Remind the
patients and their families that this is just a side effect of the drug and should be reported to the
healthcare providers immediately.

DIF: Cognitive Level: Applying REF: p. 204

13. Which of the following drug classifications for the treatment of mental health problems is
contraindicated for patients with vision problems?

a. Antipsychotics

b. Tricyclic antidepressants

c. Selective serotonin reuptake inhibitors

d. Serotonin-norepinephrine reuptake inhibitors

ANS: B

TCAs should
Authentic Nursenot Guru:
be usedFor
in patients
morewith glaucoma
of these because these
materials drugs can increase intraocular
:masterclasstutor1@gmail.com
Visovsky: Introduction to Clinical Pharmacology, 10th Edition 12

pressure.

DIF: Cognitive Level: Remembering REF: p. 206

14. A patient who has been taking imipramine comes to the clinic for a routine follow-up
appointment. Which of the following assessments would be appropriate to perform?

a. Skin assessment

b. Hearing assessment

c. Elimination assessment

d. Vital signs and weight assessment

ANS: D

Assess vital signs including baseline weight. TCAs can cause hypotension and weight gain.

DIF: Cognitive Level: Applying REF: p. 207

15. You are caring for a patient who will begin treatment for anxiety and depression that has been
unresponsive to other drugs with isocarboxazid. Which of the following statements made by the
patient would indicate understanding of the teaching related to this drug?

a. “I will take both my isocarboxazid and my SSRI drugs at the same time each day.”

b. “I will avoid situations that cause me to lose fluids and become dehydrated.”

c. “I will avoid eating foods containing tyramine that can increase blood pressure.”

d. “I will take my isocarboxazid weekly, as this drug is long-acting.”

ANS: C

Patients taking MAO-Is risk of hypertensive crisis from taking foods or drinks high in tyramine.
Tyramine is an amino acid that is involved in the release of norepinephrine. Normally, tyramine is
broken down by monoamine oxidases (enzymes). When the patient is taking an inhibitor of the
enzyme (MAO-Is), there is an increase in norepinephrine which can then significantly increase
blood pressure. This can cause sudden and severe hypertension.

Authentic Nurse Guru: For more of these materials :masterclasstutor1@gmail.com


Visovsky: Introduction to Clinical Pharmacology, 10th Edition 12

DIF: Cognitive Level: Applying REF: p. 208

16. Which of the following drugs would be appropriate for the treatment of bipolar illness?

a. Lithium

b. Sertoline

c. Alprazolam

d. Chlorpromazine

ANS: A

Lithium is specifically used for patients with bipolar disorder who are in an acute manic phase.

DIF: Cognitive Level: Remembering REF: p. 209

17. You have just completed teaching with a patient who is beginning lithium. Which of the
following statements made by the patient demonstrates understanding of this drug?

a. “I will need to increase my salt intake while taking this drug.”

b. “I will need regular blood testing while taking this drug.”

c. “I may develop mild memory loss while taking this drug.”

d. “Lithium cannot be taken with tyrosine-containing foods.”

ANS: B

Serum lithium levels need to be monitored frequently while the patient is taking the drug. Blood
levels are checked 4 days after the patient starts taking lithium. Desired level for acute mania is
0.8 to 1.2 mEq/L. Any levels over 1.5 mEq are considered toxic.

Levels >3 mEq/L are associated with coma, organ failure, and even death.

DIF: Cognitive Level: Applying REF: p. 210

18. You are taking the history of a patient who is suspected of having lithium toxicity. What
symptoms would you expect the patient to report?

Authentic Nurse Guru: For more of these materials :masterclasstutor1@gmail.com


Visovsky: Introduction to Clinical Pharmacology, 10th Edition 12

a. Increased drowsiness, weakness, and nausea

b. Heart palpitations, seizures, and muscle spasm

c. Decreased urine output, vision loss, and irritability

d. Increased mania, excessive salivation, and tachycardia

ANS: A

Signs and symptoms of lithium toxicity include nausea, vomiting, increased drowsiness, muscle
weakness, severe hand tremor, and incoordination. Report these symptoms to the RN or
healthcare provider immediately.

DIF: Cognitive Level: Remembering REF: p. 210

19. A 42-year-old male patient with bipolar illness taking lithium carbonate tells you that he has
recently taken up running outdoors as a means of stress reduction. What is your best response?

a. “Running is considered a safe and effective adjunct to lithium therapy.”

b. “Running outdoors can increase the risk for lithium toxicity in hot weather.”

c. “Lithium can impair gait and balance, so running is contraindicated.”

d. “Lithium can cause hallucinations, so running may place you in danger.”

ANS: B

Patients taking lithium should refrain from sweating or activities that can cause dehydration,
which then places them at risk for lithium toxicity.

DIF: Cognitive Level: Applying REF: p. 210

20. The family members of a male patient admitted for hallucinations and delusions tell you that
the patient is currently taking chlorpromazine (Thorazine) for the treatment of schizophrenia. In
assessing this patient upon admission, you find the patient has a fever of 103 degrees F and a
blood pressure of 186/102. What is your best action?

a. Report these findings to the healthcare provider immediately.

Authentic Nurse Guru: For more of these materials :masterclasstutor1@gmail.com


Visovsky: Introduction to Clinical Pharmacology, 10th Edition 12

b. Give the patient an antihypertensive and an antipyretic immediately.

c. Place the patient on nasal oxygen and place him in semi-fowlers position.

d. Arrange for the patient to be transferred to a medical unit.

ANS: A

Typical antipsychotics, such as Thorazine, are more commonly used for long-term management
of chronic mental illnesses associated with psychosis. A serious adverse effect of this therapy is
neuroleptic malignant syndrome characterized by changes in blood pressure (hypertension or
hypotension, confusion, and high fever). This is a medical emergency, and the healthcare provider
should be notified immediately.

DIF: Cognitive Level: Applying REF: p. 213

21. The family of a patient who has been taking the antipsychotic fluphenazine (Prolixin) for the
last 2 years. The patient’s family reports the patient has suddenly begun smacking her lips, and
involuntarily sticking out her tongue. What would be your best first action?

a. Call the healthcare provider to report these symptoms.

b. Tell the family to hold the next dose of fluphenazine.

c. Inform the family that these symptoms are expected, and of no concern.

d. Tell the family that these symptoms will disappear over time as the drug dose stabilizes.

ANS: A

The main adverse effects of typical antipsychotics are extrapyramidal symptoms (EPS), related to
the decrease in dopamine, many are severe, and some may be irreversible so it is important to
recognize very early in treatment. The healthcare provider should be called immediately, and
often, the drug is discontinued.

DIF: Cognitive Level: Applying REF: p. 215

22. You are about to begin teaching a patient with a psychosis about taking typical antipsychotic
drugs. Which of the following statements would you include in your teaching plan?

Authentic Nurse Guru: For more of these materials :masterclasstutor1@gmail.com


Visovsky: Introduction to Clinical Pharmacology, 10th Edition 12

a. “Do not abruptly stop taking this drug.”

b. “Take these drugs with a glass of grapefruit juice.”

c. “Take these drugs at night with your sleep drug.”

d. “You will experience the effects of these drugs within 30 minutes of taking them.”

ANS: A

Continue to take the drugs as prescribed. It may take several weeks before significant changes
occur. Do not suddenly stop taking these drugs as this can result in nausea, dizziness, and tremors.
You can take many of these drugs with food to avoid GI upset, but these drugs can interact with
grapefruit juice. Do not drink alcohol or use any sedatives while using these drugs to prevent deep
sedation and other dangerous side effects.

DIF: Cognitive Level: Applying REF: p. 216

23. A patient who has been taking the atypical antipsychotic risperidone (Risperdal) reports
weight gain after taking the drug for 3 months. What would you tell this patient?

a. “Stop this drug immediately.”

b. “Weight gain can be an adverse effect of this drug.”

c. “Weight gain is an expected side-effect of this drug.”

d. “I will notify your healthcare provider, so the drug can be changed.”

ANS: C

For patients taking atypical antipsychotics, weight gain is associated with these drugs.

DIF: Cognitive Level: Applying REF: p. 217

MULTIPLE
Authentic RESPONSE
Nurse Guru: For more of these materials :masterclasstutor1@gmail.com
Visovsky: Introduction to Clinical Pharmacology, 10th Edition 12

1. Which symptoms would you expect a patient who is experiencing mania? (Select all that
apply.)

a. Catatonia

b. CNS depression

c. Increased energy

d. Grandiose notions

e. Poor judgement

f. Excessive sleepiness

ANS: C, D, E

Symptoms of mania include increased energy, grandiose notions, poor judgement, and increased
sexual desire, racing thoughts, irritability, increased energy, inappropriate social behavior, and
increased talking.

DIF: Cognitive Level: Knowing REF: p. 209

2. The LPN/LVN is reviewing the various indications for selective serotonin uptake inhibitors
(SSRI) with a nursing student. Mention in which of the following scenarios an SSRI would be
appropriate. (Select all that apply.)

a. A female experiencing psychosis

b. A 30-year-old with obsessive-compulsive disorder

c. A 20-year-old female diagnosed with premenstrual dysphoric disorder (PMDD)

d. A male patient who has PTSD from his experiences in the military

e. A 28-year-old male who was recently diagnosed with bipolar disorder

f. A female patient experiencing generalized anxiety disorder

ANS: B, C, D, F

SSRIs can
Authentic be used
Nurse in a variety
Guru: of conditions
For more including
of these depression,
materials premenstrual dysphoric
:masterclasstutor1@gmail.com
Visovsky: Introduction to Clinical Pharmacology, 10th Edition 13

disorder (PMDD), and post-traumatic stress disorder, obsessive-compulsive disorder, and general
anxiety disorder.

DIF: Cognitive Level: Knowing REF: p. 201

3. Too much serotonin can lead to the adverse effect known as serotonin syndrome. Which of the
following symptoms are associated with serotonin syndrome? (Select all that apply.)

a. Confusion

b. Restlessness

c. Dilated pupils

d. Increased sweating

e. Severe constipation

f. Extremely high blood pressure

ANS: A, B, C, D, F

Serotonin syndrome is a potentially life-threatening disorder characterized by confusion,


restlessness, extremely high blood pressure, dilated pupils, increased sweating, seizures, and
tremors.

DIF: Cognitive Level: Knowing REF: p. 204

4. You are caring for a patient with schizophrenia who is taking antipsychotic drugs for the
control of hallucinations and delusions. Which symptoms experienced by the patient would alert
you to that the development of neuroleptic malignant syndrome? (Select all that apply.)

a. Pinpoint pupils

b. Muscle rigidity

c. Elevated temperature

d. Unstable blood pressure

e. Increased white blood count


Authentic Nurse Guru: For more of these materials :masterclasstutor1@gmail.com
Visovsky: Introduction to Clinical Pharmacology, 10th Edition 13

f. Extremely low creatinine kinase

ANS: B, C, D, E

Neuroleptic malignant syndrome is a potentially life-threatening condition characterized by


muscle rigidity, elevated temperature, unstable blood pressure, increased white blood cell count,
elevated creatinine kinase, and hyperkalemia.

DIF: Cognitive Level: Understanding REF: p. 215

5. The LPN/LVN is taking care of a patient with severe depression who is prescribed
amitriptyline. The LPN/LVN is preparing to teach the patient about the drug amitriptyline. Which
of the following statements should be included in the LPN/LVN’s teaching plan? (Select all that
apply.)

a. “You can use sugarless gum or candy to relieve dry mouth.”

b. “This drug can affect your sleeping.”

c. “You should avoid ripe cheeses and smoked meats.”

d. “Keep out of the sun while taking this drug.”

e. “This drug can relieve depression quickly.”

f. “Move slowly when changing positions.”

ANS: A, B, D, F

TCAs can cause dry mouth, so using sugarless gum, candy, or ice ships can help. Sun sensitivity
can occur, so staying out of the sun and the use of sunscreen is recommended. Orthostatic blood
pressure changes can occur, so patients need to be told to change positions carefully. This drug
can cause either drowsiness, or difficulty sleeping.

DIF: Cognitive Level: Applying REF: p. 202

Authentic Nurse Guru: For more of these materials :masterclasstutor1@gmail.com


Visovsky: Introduction to Clinical Pharmacology, 10th Edition 13

Chapter 12: Drugs for Pain Management Visovsky: Introduction to Clinical


Pharmacology, 10th Edition

MULTIPLE CHOICE

1. What types of drugs have the main purpose of relieving pain?

a. Anticholinergics

b. Antagonists

c. Analgesics

d. Agonists

ANS: C

Analgesics have the specific purpose of relieving pain either by changing the patient’s perception
of pain or by reducing painful stimulation at its source. Although some analgesics are opioid
agonists, the term agonist is not specific for a pain-relieving drug.

DIF: Cognitive Level: Remembering REF: p. 224

2. An adult patient has returned to the unit from an operative procedure. The healthcare provider
asks you to rate a patient’s postoperative pain. What is your best response?

a. “He is asleep now, so his pain level is acceptable.”

b. “I will ask the patient to rate his own pain.”

c. “His wife says he can still feel the incision when he turns or coughs.”

d. “Since he received 15 mg of morphine an hour ago, his pain is probably minimal.”

ANS: B

Pain is always a subjective experience; that is, pain is a sensation the patient feels and that cannot
be felt or measured by someone else. Accurate pain assessment is best measured by the patient, if
he or she is not unconscious or confused. Being asleep does not mean that pain is relieved.

DIF: Cognitive Level: Applying REF: p. 221


Authentic Nurse Guru: For more of these materials :masterclasstutor1@gmail.com
Visovsky: Introduction to Clinical Pharmacology, 10th Edition 13

3. You are caring for a female patient who is paralyzed from the waist down and has a large open
wound on her right heel. When asked about her pain in that foot, she tells you it is a 0 on a 0 to 10
pain rating scale. What does this response indicate?

a. The patient’s paralysis prevents the patient’s brain from perceiving the pain.

b. The patient has chronic pain that does not trigger the stress response.

c. The patient has demonstrated substance misuse behaviors from taking pain drugs too often.

d. The patient’s acute pain has been effectively managed by the prescribed pain drug.

ANS: A

Pain is felt or perceived in the brain rather than in the body area where it occurs. When a body
part is injured, this injury stimulates pain nerve endings in the thumb that then send (transmit)
electrical nerve impulses as a signal from the injury site along nerves to the spinal cord. At the
spinal cord, the original signal is transferred to special pain nerve tracts up the spinal cord to the
area of the brain where pain in that body site is perceived. Paralysis from the waist down in the
spinal cord prevents the transmission of a pain signal from the foot so that pain is not perceived
no matter how deep or bad the heel wound is.

DIF: Cognitive Level: Understanding REF: p. 222

4. A nursing student assigned to work with you asks you to explain why natural and synthetic
opioids are considered “high-alert drugs?” What is your best response?

a. “Opioids have an increased risk for causing a patient harm if given in error.”

b. “Opioids have a relatively high potential for addiction or abuse.”

c. “There is wide variation in the dosages between opioid types.”

d. “High-alert drugs are those that can be given by a variety of routes.”

ANS: A

Although opioids can cause addiction or abuse, have different dosages ranges, and can be given
by the oral, intravenous, intramuscular, and transdermal routes, these are not the reasons they are
categorized as high-alert drugs. All natural and synthetic opioids are high-alert drugs because they
Authentic Nurse Guru: For more of these materials :masterclasstutor1@gmail.com
Visovsky: Introduction to Clinical Pharmacology, 10th Edition 13

have an increased risk for causing patient harm if given in error.

DIF: Cognitive Level: Applying REF: p. 225

5. The RN has given 2 mg of hydromorphone intravenously to a patient for severe pain. When
you assess the patient’s pain level an hour after receiving the drug you find the patient asleep with
a respiratory rate of 10 breaths/min. What is your best first action?

a. Give naloxone intravenously.

b. Document the finding as the only action.

c. Notify the healthcare provider immediately.

d. Assess the patient’s oxygen saturation with pulse oximetry.

ANS: D

Many patients have a reduced respiratory rate after receiving a dose of a strong morphine agonist.
Although 10 breaths/min may be a little lower than normal, it is not low enough to either notify
the healthcare provider or give naloxone without further assessment.

The best assessment is oxygen saturation by pulse oximetry to determine the effectiveness of the
current respiratory rate. If oxygen saturation is lower than 94% or the patient’s usual percentage,
awaken the patient and assess whether the patient’s respiratory rate increases.

DIF: Cognitive Level: Applying REF: p. 227

6. A patient being discharged to home is prescribed an oxycodone–acetaminophen combination


for pain management. Which statement regarding a precaution is most important to teach this
patient?

a. “If you still need this drug after 48 hours, notify your healthcare provider immediately.”

b. “Always go to bed immediately after taking a dose of this drug.”

c. “Avoid drinking alcohol while taking this drug.”

d. “Be sure to drink plenty of water and eat foods high in fiber to prevent constipation.”

ANS: D
Authentic Nurse Guru: For more of these materials :masterclasstutor1@gmail.com
Visovsky: Introduction to Clinical Pharmacology, 10th Edition 13

Drinking alcohol while on a morphine agonist will make side effects worse and increase the risk
for respiratory depression. Although preventing constipation is important, preventing respiratory
depression is more important.

DIF: Cognitive Level: Applying REF: p. 229

7. You accidentally give a patient 30 mg orally of hydromorphone (Dilaudid) instead of the 30 mg


of oral hydrocodone ordered. What is your first best action?

a. Notify the RN in charge immediately.

b. Induce the patient to vomit immediately.

c. Assess the patient’s vital signs immediately.

d. Follow your agency’s policy for documenting a drug error.

ANS: A

The 30 mg oral dose of hydromorphone is six times the normal dose and could cause dangerous or
even lethal effects. Get the RN to assess this patient immediately to determine the best course of
action (naloxone or transporting the patient to the emergency department). The patient should not
be made to vomit up the drug because he or she may already be so unalert that the risk for
aspiration is increased. Assessing vital signs is important but having the RN perform a full
assessment is more important. You can assess the vital signs after you call the RN. It is important
to document the drug error, but this is not the first priority until the possible results to the patient
have been addressed.

DIF: Cognitive Level: Applying REF: p. 226

8. When assessing the blood pressure of a patient receiving pentazocine (Talwin) 30 mg orally for
pain control two days after surgery, the reading is 166/100, which is much higher than the
patient’s presurgical blood pressure. What is your best action?

a. Assess the patient for other symptoms of addiction or dependency.

b. Give the next dose of the drug as prescribed.

c. Assess the patient for other cardiac changes.


Authentic Nurse Guru: For more of these materials :masterclasstutor1@gmail.com
Visovsky: Introduction to Clinical Pharmacology, 10th Edition 13

d. Document the change as the only action.

ANS: C

Pentazocine is an opioid agonist-antagonist that can have serious cardiac reactions. Assessing the
cardiac status is critical in preventing a heart attack or blood pressure crisis.

DIF: Cognitive Level: Applying REF: p. 229

9. A patient with chronic pain asks you to explain how tramadol works to help control pain. What
is your best response?

a. “Tramadol works by binding to opioid receptors in the brain to reduce pain the perception.”

b. “Tramadol works by reducing inflammation to inhibit the actual cause of the pain.”

c. “Tramadol acts by blocking neurotransmitters in the spinal cord and brain.”

d. “Tramadol acts by inhibiting nerve impulses in peripheral sensory nerves to decrease pain.”

ANS: C

Tramadol only weakly binds to opioid receptors and does not reduce pain in this way. Instead,
tramadol works for pain management by blocking some neurotransmitter in spinal cord and brain,
decreasing the perception of pain.

DIF: Cognitive Level: Applying REF: p. 230

10. A patient with chronic arthritis pain reports taking acetaminophen 4 to 6 g/day for arthritis
pain. Which health problem should you teach this patient that can occur as a result of taking this
acetaminophen therapy?

a. “Chronic acetaminophen use can invoke asthma attacks.”

b. “Diabetes is a complication that can result from long-term acetaminophen use.”

c. “Liver toxicity can result as a result of high acetaminophen doses.”

d. “The use and overuse of acetaminophen can result in excessive bleeding.”

ANS: C
Authentic Nurse Guru: For more of these materials :masterclasstutor1@gmail.com
Visovsky: Introduction to Clinical Pharmacology, 10th Edition 13

When taken at higher doses or for prolonged periods, acetaminophen is toxic to the liver, which
can be damaged or destroyed. The drug has a maximum daily dose to prevent liver toxicity and
should not be taken with alcohol or other liver toxic drugs.

DIF: Cognitive Level: Applying REF: p. 231

11. The postoperative orders for an adult patient who has just arrived on your unit after major
abdominal surgery reads morphine 15mg IM every 4 to 6 hours as needed for pain. What is the
best schedule for pain relief during the first postoperative day for this patient?

a. Ask the patient every 4 to 6 hours whether any drug for pain is needed.

b. Give the drug automatically every 4 hours around the clock for the 24 hours.

c. Give the drug automatically every 6 hours around the clock for the first 24 hours.

d. Wait until the patient rates the pain at an 8 or higher on a 0 to 10 pain rating scale before giving
any dose of the prescribed drug.

ANS: B

Although asking a patient to rate the intensity of the pain is good, the day of abdominal surgery
usually results in intense acute pain. The patient may be groggy from anesthesia. Starting
analgesics on a regular around-the-clock schedule provides the best pain management.

DIF: Cognitive Level: Applying REF: p. 226

12. Which of the following assessments is most important to perform on a patient newly
prescribed to take cyclobenzaprine for severe muscle pain?

a. Respiratory rate

b. Deep tendon reflexes

c. Blood pressure and heart rate

d. Muscle strength and coordination


Authentic Nurse Guru: For more of these materials :masterclasstutor1@gmail.com
Visovsky: Introduction to Clinical Pharmacology, 10th Edition 13

ANS: C

Cyclobenzaprine can cause serious cardiac dysrhythmias, prolonged cardiac conduction, and high
blood pressure.

DIF: Cognitive Level: Applying REF: p. 233

13. Which patient should be assessed closely for complications of pain management with
methocarbamol?

a. 75-year-old man with benign prostatic hyperplasia

b. 55-year-old woman with type 2 diabetes

c. 34-year-old woman with a peanut allergy

d. 17-year-old teenager who is near-sighted

ANS: A

Methocarbamol often causes urinary retention, which could cause serious complications in a
person who has difficulty emptying the bladder.

DIF: Cognitive Level: Understanding REF: p. 233

MULTIPLE RESPONSE

1. Some analgesics are considered to be strong opioid agonists. Indicate which of the following
analgesics are considered strong opioid agonists. (Select all that apply.)

a. Codeine

b. Fentanyl

c. Hydrocodone

d. Hydromorphone

e. Morphine

f. Oxycodone

Authentic Nurse Guru: For more of these materials :masterclasstutor1@gmail.com


Visovsky: Introduction to Clinical Pharmacology, 10th Edition 13

ANS: B, D, E

Morphine is a strong opioid agonist against which all other analgesics are compared. Fentanyl and
hydromorphone are more powerful than morphine. Codeine, hydrocodone, and oxycodone are
weaker opioid agonists.

DIF: Cognitive Level: Knowing REF: p. 225

2. Which of the drugs listed below help manage pain by acting at the tissue where pain starts and
do not change the person’s perception of pain? (Select all that apply.)

a. Acetaminophen

b. Antidepressants

c. Corticosteroids

d. NSAIDs

e. Opioid agonists

f. Skeletal muscle relaxants

ANS: C, D

Acetaminophen, antidepressants, opioid agonists, and skeletal muscle relaxants all work in the
brain and or spinal cord to alter the perception and (occasionally) the transmission of pain signals
in the spinal cord. Only corticosteroids and NSAIDs work to reduce mediators of pain and
inflammation at the site of injury.

DIF: Cognitive Level: Understanding REF: p. 232

3. Which of the following drugs are considered to have a role in pain management? (Select all that
apply.)

a. Antibiotics

Authentic Nurse Guru: For more of these materials :masterclasstutor1@gmail.com


b. Anticholinergics
Visovsky: Introduction to Clinical Pharmacology, 10th Edition 14

c. Anticonvulsants

d. Antidepressants

e. Antihistamines

f. Anti-inflammatories
ANS: C, D, F

Anticonvulsants, antidepressants, and anti-inflammatories have all been found to reduce some
types of pain effectively. Antibiotics, anticholinergics, and antihistamines have not been found to
have a role in pain management.

DIF: Cognitive Level: Knowing REF: p. 232

4. The LPN/LVN is taking care of a 65-year-old patient who just had a major surgery. The patient
is experiencing severe pain that he rates a 9/10. The LPN/LVN plans to give his prescribed
opioid. List the nursing action for the patient. (Select all that apply.)

a. The LPN/LVN should reassess the patient’s level of pain within an hour after giving an opioid to
determine its effectiveness.

b. The patient should be given a stool softener or laxative because all opioids cause constipation.

c. Oral opioids must be taken with food.

d. Since the patient is an older adult, he requires a higher dosage of the opioid drug.

e. The patient can take the prescribed opioid as much as he asks for it until the pain subsides.

f. The LPN/LVN should assess the patient’s respiratory rate and pulse oximetry because opioids can cause
respiratory depression.
ANS: A, B, F

It is necessary for the LPN/LVN should reassess the patient’s level of pain within an hour after
giving an opioid to determine its effectiveness. All opioids can cause constipation, so it is
important to monitor elimination status daily. It might be necessary for the patient to be given a
stool softener or laxative if the patient experiences constipation. If the patient experiences nausea
after taking his opioid drug, the opioids can be taken with food. Older adults are more likely to
have some degree of kidney and/or liver impairment that decreases their ability to metabolize and
excrete opioids, which makes them more sensitive to drugs, so they may require a lower dose of
the opioid drug. The LPN/LVN should encourage the patient to take the pain drug on the
Authentic Nurse Guru: For more of these materials :masterclasstutor1@gmail.com
Visovsky: Introduction to Clinical Pharmacology, 10th Edition 14

prescribed schedule because it is most effective to take the drug before the patient has severe pain.
However, the patient cannot receive pain drug in between the prescribed time intervals. Opioids
have the risk of respiratory depression, so the LPN/LVN should monitor the patient’s respiratory
rate and pulse oximetry.

DIF: Cognitive Level: Applying REF: p. 227

Authentic Nurse Guru: For more of these materials :masterclasstutor1@gmail.com


Visovsky: Introduction to Clinical Pharmacology, 10th Edition 14

Chapter 13: Drugs for Inflammation, Arthritis, and Gout Visovsky:


Introduction to Clinical Pharmacology, 10th Edition

MULTIPLE CHOICE

1. A patient with joint inflammation is prescribed an anti-inflammatory drug. The patient asks you
how this drug works to address this problem. What is your best response?

a. “Anti-inflammatory drugs reduce pain and limit blood vessel responses to joint injury.”

b. “Anti-inflammatory drugs limit the body’s response to cortisol in the body.”

c. “Anti-inflammatory drugs increase the release of TNF and other mediators.”

d. “Anti-inflammatory drugs contain antibacterial and anti-inflammatory properties.”

ANS: A

Anti-inflammatory drug’s primary purpose is to reduce pain and prevent or limit the tissue and
blood vessel responses to injury or invasion.

DIF: Cognitive Level: Applying REF: p. 238

2. A patient with a history of cardiovascular disease has been prescribed aspirin daily. What
specific risk should this patient be informed of because of taking aspirin?

a. Risk of infection

b. Risk of bleeding

c. Risk of liver impairment

d. Risk of cognitive impairment

ANS: B

Patients who take aspirin must be monitored for bleeding due to irreversible platelet inhibition for
the lifespan of the platelet so as long as 7 to 8 days.

DIF: Cognitive Level: Applying REF: p. 240

Authentic Nurse Guru: For more of these materials :masterclasstutor1@gmail.com


Visovsky: Introduction to Clinical Pharmacology, 10th Edition 14

3. A male patient taking daily aspirin for the prevention of cardiovascular complications informs
you that he is planning to have a tooth extraction next week. What correct information should be
provided to this patient?

a. “Aspirin is associated with tooth loss following dental surgery.”

b. “Stop taking the aspirin 7 days before the dental procedure.”

c. “You may require aspirin plus additional pain drug after the dental procedure.”

d. “Taking enteric-coated aspirin ensures you will not be at risk for any complications.”

ANS: B

Patients who take aspirin must be monitored for bleeding due to irreversible platelet inhibition for
the lifespan of the platelet as long as 7 to 8 days. This is very important if a patient is scheduled
for an invasive procedure or surgery so make sure to notify the prescriber if the patient has an
upcoming surgery.

DIF: Cognitive Level: Applying REF: p. 240

4. You are preparing a teaching plan for a patient prescribed the nonselective NSAID ibuprofen
800 mg orally daily. Which of the following instructions should be included in this teaching plan?

a. Avoid eating excessive amounts of protein as this can interfere with the drug’s action.

b. This drug can increase serum blood sugar levels in patients with diabetes.

c. Taking this drug before bed can cause excessive night-time urination.

d. Take this drug with food to prevent GI upset.

ANS: D

If taking NSAIDS causes mild GI upset, the patient may take the drug with a small amount of
food or milk. These drugs decrease blood sugar, making diabetics at high risk for hypoglycemia.
NSAIDs can cause fluid retention.

DIF: Cognitive Level: Applying REF: p. 240

5. A patient who has been taking aspirin several times daily for arthritis pain reports ringing in her
Authentic Nurse Guru: For more of these materials :masterclasstutor1@gmail.com
Visovsky: Introduction to Clinical Pharmacology, 10th Edition 14

ears. What action should you take?

a. Instruct the patient to reduce the dose of aspirin by half until the ringing stops.

b. No action is needed; this is an expected response to this dose of aspirin.

c. Call the healthcare provider, as this is adverse reaction to the drug.

d. Instruct the patient to take the full dose before bed.

ANS: C

Tinnitus (ringing in the ears) is an adverse reaction to anti-inflammatory analgesics.

DIF: Cognitive Level: Applying REF: p. 241

6. A mother brings her febrile infant to the pediatric clinic for evaluation. The infant has a
temperature of 102 degrees. The mother tells you that she wishes she had given the baby a dose of
liquid aspirin before leaving the house to bring down the baby’s temperature. What is your best
response?

a. “We can give the baby a dose now before you leave the clinic.”

b. “Only give aspirin alternating with acetaminophen every 4 hours for fever.”

c. “Aspirin should not be given to infants with acute illness.”

d. “Aspirin will need to be given with a proton-pump inhibitor to prevent bleeding.”

ANS: C

Aspirin should not be given to infants or children who have an acute illness because of its
Authentic Nurse Guru: For more of these materials :masterclasstutor1@gmail.com
Visovsky: Introduction to Clinical Pharmacology, 10th Edition 14

association with development of a very serious problem known as Reye syndrome. This disorder
can lead to mental deficits, coma, or death.

DIF: Cognitive Level: Applying REF: p. 241

7. A patient with a history of alcoholism is taking aspirin on a regular basis for general aches and
pains. The patient reports feeling weak and dizzy, and has developed abdominal pain 7 days ago,
which is increasing in severity. What adverse drug effect might this patient be experiencing?

a. Liver damage due to the combination of aspirin and alcohol

b. Reye syndrome due to the combination of aspirin and alcohol

c. Gastrointestinal bleeding due to the combination of aspirin and alcohol

d. Gastroesophageal reflux disease due to the combination of aspirin and alcohol

ANS: C

Alcohol taken with any of the anti-inflammatory analgesics greatly increases the risk for
gastrointestinal bleeding.

DIF: Cognitive Level: Applying REF: p. 241

8. A patient with diabetes has been prescribed a course of prednisone to treat an acute
exacerbation of asthma. Which of the following instructions should you give to the patient before
he is discharged home?

a. “Monitor your blood sugar regularly.”

b. “Take this drug on an empty stomach.”

c. “You may stop this drug when your symptoms improve.”

d. “You may experience a change in the color of your urine.”

ANS: A

The patient should be instructed to monitor blood sugar regularly because corticosteroids reduce
the sensitivity of insulin receptors and increase blood glucose levels. Adjustments to oral
antidiabetic drug or insulin may be needed while taking corticosteroids.
Authentic Nurse Guru: For more of these materials :masterclasstutor1@gmail.com
Visovsky: Introduction to Clinical Pharmacology, 10th Edition 14

Warn patients to not stop taking the oral drug suddenly (without the guidance of the healthcare
provider) to prevent possible adrenal insufficiency. Corticosteroids should be taken with food or
milk to reduce the risk for gastric ulcers. There is no effect of corticosteroids on the color of urine.

DIF: Cognitive Level: Applying REF: p. 243

9. A patient with a chronic inflammatory condition has been taking corticosteroids for several
months. Which of the following side effects should you monitor this patient for?

a. Weight loss

b. Increased muscle mass

c. Hypotension

d. Fat redistribution

ANS: D

After a month of therapy, patients taking corticosteroids experience fat redistribution (moon face
and “buffalo hump” between the shoulders), weight gain, hypertension (after 1 week), and
decreased muscle mass.

DIF: Cognitive Level: Understanding REF: p. 244

10. A patient on long-term corticosteroid therapy appears to have frequent colds and upper
respiratory infections and is concerned that something may be wrong. What is your best response?

a. “Taking corticosteroids for a long time can reduce your immunity.”

b. “There is no relationship between the treatment for your condition and frequent colds.”

c. “Corticosteroids affect the production of protective mucus.”

d. “Corticosteroids increase adrenal gland function and result in infections.”

ANS: D

The most important problems are associated with long-term use and include adrenal gland
suppression and reduced immunity that can make patients susceptible to infections.

Authentic Nurse Guru: For more of these materials :masterclasstutor1@gmail.com


Visovsky: Introduction to Clinical Pharmacology, 10th Edition 14

DIF: Cognitive Level: Applying REF: p. 244

11. A patient with severe asthma who has been taking systemic corticosteroids for 2 weeks tells
you he is feeling better and would like to stop taking this drug today. What is your best response?

a. “Take the last dose today, and then you can stop the drug.”

b. “The dose of this drug will need to be tapered down over time to prevent complications.”

c. “As long as your asthma symptoms have resolved, you can stop the drug.”

d. “Skip the drug on the days you feel better and take it only if symptoms reoccur.”

ANS: B

To prevent adrenal insufficiency, doses of systemic corticosteroids must be tapered rather than
stopped abruptly. Tapering of the drug allows the atrophied adrenal gland cells to gradually begin
producing cortisol again and prevents acute adrenal insufficiency.

DIF: Cognitive Level: Applying REF: p. 245

12. A patient who has been prescribed a topical corticosteroid cream for the treatment of a rash
tells you that he now has a small, reddened, open wound located on the anterior aspect of the right
foot. He asks you if he can apply this same cream to this new wound to reduce the redness. What
is your best response?

a. “You should not apply topical steroids to open wounds.”

b. “You may use the same topical steroid on this wound but apply it with a gloved hand.”

c. “Dispose of at least 1

/2 inch of the topical steroid before applying it elsewhere.”

d. “Mix the topical steroid with an antibiotic cream for the best effect.”

ANS: A

Topical steroids should not be applied to open wounds. They should be avoided if there are any
signs of infection as they may increase the risk of the infection spreading due to their effect on the
immune/inflammatory response.
Authentic Nurse Guru: For more of these materials :masterclasstutor1@gmail.com
Visovsky: Introduction to Clinical Pharmacology, 10th Edition 14

DIF: Cognitive Level: Applying REF: p. 246

13. What would be an appropriate nursing action to prevent complications in an elderly person
taking long-term corticosteroid therapy?

a. Protect the skin from skin tears.

b. Bathe the patient twice daily.

c. Check the patient’s blood pressure before giving the drug.

d. Ask the patient about the presence of constipation.

ANS: A

Make sure to carefully protect skin of patients who are taking corticosteroids while transferring or
positioning to prevent skin tears.

DIF: Cognitive Level: Applying REF: p. 246

14. A patient is beginning a prescribed disease-modifying antirheumatic drug (DMARD) for the
treatment of rheumatoid arthritis. The patient asks you how these drugs work to help this
condition. What is your best response?

a. “These drugs modify the immune system by decreasing T-cell response.”

b. “These drugs act to increase the body’s immune response to rheumatoid factor.”

c. “These drugs work by inhibiting the inflammatory mediator tumor necrosis factor.”

d. “These drugs reduce the amount of an enzyme that controls the production of purines.”

ANS: C

DMARDS inhibit the inflammatory mediator tumor necrosis factor (TNF). They bind to the TNF
molecules produced by white blood cells (WBCs) and prevent them binding to TNF receptor sites
on inflammatory cells and other cells. This prevents the cells from continuing to produce even
more TNF and other substances that enhance the inflammatory responses and cause direct tissue
destruction.

DIF: Cognitive Level: Applying REF: p. 247


Authentic Nurse Guru: For more of these materials :masterclasstutor1@gmail.com
Visovsky: Introduction to Clinical Pharmacology, 10th Edition 14

15. You are teaching a patient with rheumatoid arthritis about giving subcutaneous adalimumab
(Humira) at home. What instructions should be included in this patient’s teaching plan?

a. Before drawing up drug, shake the vial well to distribute the drug.

b. Rotate the injection site of the thighs and abdomen frequently.

c. Do not discontinue this drug if you have an infection.

d. Rub the injection site well to prevent bleeding.

ANS: B

Instruct the patient to rotate injection sites on the front of the thighs and the abdomen to ensure
best absorption and prevent skin problems. Avoid giving within 2 inches of the umbilicus because
this area has many blood vessels and absorption can be too rapid.

DIF: Cognitive Level: Applying REF: p. 245

16. A patient who is taking a disease-modifying antirheumatic drug (DMARD) has just returned
from surgery. What potential complication would you report to the RN or healthcare provider
directly related to the use of DMARDs before surgery?

a. Redness and drainage at the incision site

b. A low blood pressure

c. A decreased urinary output

d. Excessive sleepiness

ANS: A

Because DMARDs reduce the immune response, patients having surgery are at increased risk for
infection. Redness and drainage at the incisions site is an indication of a wound infection.

DIF: Cognitive Level: Applying REF: p. 247

17. You are preparing to give a subcutaneous injection of etanercept to a patient with rheumatoid
arthritis. Which of the following actions is contraindicated in giving this injection?

Authentic Nurse Guru: For more of these materials :masterclasstutor1@gmail.com


Visovsky: Introduction to Clinical Pharmacology, 10th Edition 15

a. Telling the patient to expect pain at the injection site

b. Rotating the injection sites

c. Making sure not to shake the vial

d. Rubbing the site after the injection

ANS: D

Rubbing the site after giving a DMARD injection is contraindicated because it may cause
bleeding and bruising at the injection site.

DIF: Cognitive Level: Applying REF: p. 247

18. Which laboratory value is most likely to be elevated in a patient presenting with acute gout?

a. Potassium

b. Uric acid

c. Calcium

d. Glucose

ANS: B

High serum uric acid levels lead to the formation of uric acid crystals, usually in the kidneys and
joint spaces. These crystals are long and jagged and cause the swelling, inflammation, and severe
pain of gout.

DIF: Cognitive Level: Understanding REF: p. 249

19. A patient with acute gout asks you why allopurinol cannot be used at this time. What is your
best response?

a. “Allopurinol does not lower uric acid levels.”

b. “Allopurinol is useful in rheumatoid arthritis, but not gout.”

c. “Allopurinol is used only for the prevention of gout attacks.”

Authentic Nurse Guru: For more of these materials :masterclasstutor1@gmail.com


Visovsky: Introduction to Clinical Pharmacology, 10th Edition 15

d. “Allopurinol can cause swelling of the feet, worsening your gout symptoms.”

ANS: C

Allopurinol is used to prevent gout attacks by reducing the amount of an enzyme that converts the
purines in protein into uric acid, and to maintain a lower blood uric acid level.

DIF: Cognitive Level: Applying REF: p. 249

20. A patient with gout requires dietary teaching to help control flare ups of the illness. What
instructions should you give to this patient regarding a diet plan?

a. “Increase your intake of organ meats for protein.”

b. “Avoid foods that are high in purine, such as tuna.”

c. “Avoid drinking fruit juices while taking antigout drugs.”

d. “Increase your intake of vegetables such as cauliflower and asparagus.”

ANS: B

Avoid foods that are high in purines as those foods may precipitate an acute attack. High purine
foods are organ meats (liver, kidneys), and certain vegetables such as cauliflower, asparagus,
mushrooms, and spinach.

DIF: Cognitive Level: Applying REF: p. 249

21. A 62-year-old female presents with intense fatigue, increased joint stiffness, pain, aching,
tenderness, and swelling in more than one joint, limited movement in joints and decreased range
of motion, and pain throughout the body. After thorough exam and diagnostic testing, the patient
is diagnosed with rheumatoid arthritis.

Although there is no cure for RA, patients may be prescribed for treatment of symptoms. If a
patient is on long-term corticosteroids, they may develop a cushingoid appearance, which is
exhibited by ----------- --.

A. antibiotics, Ace inhibitor, or NSAIDs; weight loss, jaundice, and/or truncal obesity

b. NSAIDs, Corticosteroids, or DMARDs; moon face, buffalo hump, and/or truncal obesity
Authentic Nurse Guru: For more of these materials :masterclasstutor1@gmail.com
Visovsky: Introduction to Clinical Pharmacology, 10th Edition 15

c. DMARDs, NSAIDs, or Ace inhibitor; buffalo hump, moon face, and/or weightloss

D. corticosteroids, Ace inhibitor, or DMARDs; jaundice, truncal obesity, and/or moonface

ANS: B

Although there is no cure for rheumatoid arthritis, patients may be prescribed the following drugs
(or a combination): NSAIDs, steroids, DMARDs, or biologics. If a patient is on long-term
corticosteroids, they may develop a “cushingoid appearance” or Cushing syndrome, which occurs
when your body is exposed to high levels of cortisol. A “cushingoid appearance” has certain
distinguishable signs and symptoms including: moon face, buffalo hump, and truncal obesity.
Other signs and symptoms of Cushing syndrome include: weight gain, slow wound healing, easy
bruising, muscle weakness, acne, hair thinning, red stretch marks (striae), and
immunosuppression.

DIF: Cognitive Level: Knowing REF: p. 239

MULTIPLE RESPONSE

1. Inflammation is the result of tissue and blood vessel reactions to white blood cells. What are the
symptoms associated with inflammation? (Select all that apply.)

a. Pain

b. Warmth

c. Redness

d. Swelling

e. Elevated temperature

f. Bacteria in the bloodstream

ANS: A, B, C, D

Inflammation is a predictable set of tissue and blood vessel actions caused by white blood cells
(leukocytes) and their products as a response to injury or infection. These tissue and blood vessel
actions cause the five major symptoms of inflammation: pain, redness, warmth, swelling, and loss
of function.
Authentic Nurse Guru: For more of these materials :masterclasstutor1@gmail.com
Visovsky: Introduction to Clinical Pharmacology, 10th Edition 15

DIF: Cognitive Level: Remembering REF: p. 237

Test bank

Test bank

Test bank

Copyright © 2022, Elsevier Inc. All Rights Reserved. 5

2. When caring for a patient who is taking anti-inflammatory drugs, what should be included in
the patient’s teaching plan? (Select all that apply.)

a. “You should take this drug without food.”

b. “Take this drug with a full glass of water.”

c. “Only take this drug when you have pain.”

d. “This drug may interact with anticoagulants.”

e. “Contact your healthcare provider if you develop dark stools.”

ANS: B, D, E

These drugs should be taken exactly as ordered on a regular schedule to keep blood levels of the
drug stable. Dark stools can be a sign of GI bleeding, and adverse effect of the drug. To decrease
stomach upset, take with a full glass of water. These drugs can be taken with food or milk. Anti-
inflammatory drugs can interact with anticoagulants and cause bleeding.

DIF: Cognitive Level: Applying REF: p. 242

3. A patient is taking an anti-inflammatory drug and calls the clinic to tell the LPN/LVN that he
missed some of the doses last week.

The LPN/LVN instructs the patient about what to do in this situation. List the appropriate
response from the nurse. (Select all that apply.)

a. Skip all missed drug doses.

b. If it is close to the next time the drug is due, skip the missed dose.
Authentic Nurse Guru: For more of these materials :masterclasstutor1@gmail.com
Visovsky: Introduction to Clinical Pharmacology, 10th Edition 15

c. All missed doses of the drug should be returned to the pharmacy.

d. Never take a double dose of this drug.

e. If possible, take the missed dose within an hour of the scheduled time.

f. Once you miss a dose, you should double up on the drug dose at the next time it is due.

ANS: B, D, E

Never take a double dose of this drug. If a drug dose is missed, it may be taken within an hour
when it was scheduled. If the patient remembers the missed dose close to the time when the next
dose is to be taken, he should take the regular dose and miss the skipped dose.

DIF: Cognitive Level: Applying REF: p. 242

4. Anti-inflammatory analgesics should not be used by patients with which of the following
conditions? (Select all that apply.)

a. Stroke

b. Cachexia

c. Liver disease

d. Heart disease

e. Severe migraine

f. Transient ischemic attacks

ANS: A, C, F

Anti-inflammatory analgesics should not be used in patients with hepatic (liver) disease, stroke, or
when patients have symptoms of

TIAs.

DIF: Cognitive Level: Applying REF: p. 241

5. Corticosteroids are a class of drugs that are used to manage inflammation. What are the
properties
Authentic or actions
Nurse associated
Guru: with corticosteroids?
For more (Select all
of these materials that apply.)
:masterclasstutor1@gmail.com
Visovsky: Introduction to Clinical Pharmacology, 10th Edition 15

a. Corticosteroids decrease inflammatory mediators.

b. Corticosteroids act only within the cells that are inflamed.

c. Corticosteroids can be used to manage chronic inflammation.

d. Corticosteroids slow the production of WBCs in the bone marrow.

e. Corticosteroids have few adverse effects due to their localized action.

f. Corticosteroids increase arachidonic acid production of inflammatory mediators.

ANS: A, C, D

Corticosteroids are very useful in managing chronic inflammation. They are very powerful in
decreasing the production of all known mediators that trigger inflammation. Corticosteroids
inhibit enzymes and proteins that start and continue the arachidonic acid production of
inflammatory mediators. They also slow the production of white blood cells (WBCs) in the bone
marrow. The actions of corticosteroids occur in all cells, not just those involved in inflammation.
As a result, their therapeutic effects, side effects, and adverse effects are wide-spread.

DIF: Cognitive Level: Remembering REF: p. 242

6. You are preparing to teach a patient and family member about long-term corticosteroid therapy.
Which instructions should be included in the patient’s teaching plan? (Select all that apply.)

a. You may increase the dose of this drug by one-half if your symptoms worsen.

b. Consult your healthcare provider before getting any vaccinations.

c. Stop this drug immediately if you develop nausea or vomiting.

d. Take your dose of this drug early in the morning.

e. Take this drug with food to prevent GI upset.

f. Eat a diet low in potassium-containing foods.

ANS: B, D, E

Only the healthcare provider should adjust the dose of corticosteroids. Patients with normal
Authentic Nurse Guru: For more of these materials :masterclasstutor1@gmail.com
Visovsky: Introduction to Clinical Pharmacology, 10th Edition 15

adrenal gland function should take corticosteroids early in the morning; this is the time when the
adrenal glands are normally secreting the most cortisol, so the corticosteroid dose more closely
mimics the body’s usual actions. Never stop corticosteroids abruptly. Take these drugs with food
to prevent GI upset. Teach the patient to eat a diet rich in potassium-containing foods and low in
sodium.

DIF: Cognitive Level: Applying REF: p. 243

7. A patient presents to the clinic and is newly diagnosed with gout. The LPN/LVN is teaching the
patient about lifestyle changes that can potentially control gout symptoms. One of these changes
is eliminating or eating less foods that produce uric acid. List the foods that the patient should
avoid. (Select all that apply.)

a. Beer and wine

b. Organ meats

c. Shellfish

d. Fruits and vegetables

e. Red meat

f. Pork

g. Peanut butter

h. Whole grains

ANS: A, C, E, F, H

About 10% of people with gout can control their symptoms with lifestyle changes, such as eating
less purine-rich foods, or foods that produce uric acids. Foods high in purines include: beer and
wine, shellfish and fish such as herring, sardines, salmon, haddock, anchovies, trout, tuna, and
Authentic Nurse Guru: For more of these materials :masterclasstutor1@gmail.com
Visovsky: Introduction to Clinical Pharmacology, 10th Edition 15

mussels, red meat, organ meats, and pork.

DIF: Cognitive Level: Knowing REF: p. 249

Authentic Nurse Guru: For more of these materials :masterclasstutor1@gmail.com


Visovsky: Introduction to Clinical Pharmacology, 10th Edition 15

Chapter 14: Drugs for Gastrointestinal Problems Visovsky: Introduction to


Clinical Pharmacology, 10th Edition

MULTIPLE CHOICE

1. A patient arrives at the health clinic after a week of treating herself at home with magnesium-
based antacids. She complains to the LPN that she has experienced several episodes of
lightheadedness and near fainting over the past 2 days. Which action is most appropriate?

a. Ask the patient how many doses of the drug she has been taking each day.

b. Instruct the patient to maintain a symptom diary to determine aggravating factors.

c. Instruct the patient to bring the bottle of antacid drug to her next appointment.

d. Instruct the patient to switch to an aluminum-based antacid immediately.

ANS: A

For the patient taking a magnesium-based antacid, adverse effects are usually related to
hypermagnesemia such as muscle weakness, low blood pressure, and low heart rate. Hypotension
may develop in cases of extreme hypermagnesemia and could cause lightheadedness especially if
the patient is taking too many doses of the drug. Telling the patient to keep a symptom diary of
aggravating factors or bringing the bottle to the next appointment does not address the patient’s
problem of near-syncopal episodes. Instructing the patient to change the drug is a decision that a
healthcare provider should make.

DIF: Cognitive Level: Applying REF: p. 263

2. The LPN is helping to prepare a teaching plan for a patient who is beginning treatment with an
antacid. The LPN verifies that the patient understands that it is most appropriate to take the
antacid at which time(s)?

a. Immediately on awakening in the morning

b. One hour after each meal and at bedtime

c. Thirty minutes after an H2-receptor agonist

Authentic
d. ThirtyNurse
minutesGuru: For more of these materials :masterclasstutor1@gmail.com
before bedtime
Visovsky: Introduction to Clinical Pharmacology, 10th Edition 15

ANS: B

Antacids neutralize gastric acid. Food also serves as a buffer for gastric acid. Antacids are most
beneficial if given between meals and at bedtime. When taken with H2-receptor agonists, antacids
should be scheduled at least an hour apart, with the antacid taken first.

DIF: Cognitive Level: Understanding REF: p. 264

3. Which patient statement indicates an accurate understanding of taking proton-pump inhibitors?

a. “This drug will cure my gastric ulcer.”

b. “I can stop taking my drug when I feel better.”

c. “I need to avoid excessive exposure to the sun.”

d. “I should decrease my intake of calcium and vitamin D.”

ANS: C

The patient should wear sunscreen and protective clothing because the skin may be more sensitive
to light. The patient should avoid driving or using heavy machinery while using this drug as it
may cause dizziness. The patient should contact the healthcare provider for recommendations
about calcium and vitamin D as this drug may increase the risk for osteoporosis. If the patient is
taking prescription proton-pump inhibitors, make sure to take the full prescription even if feeling
better.

DIF: Cognitive Level: Understanding REF: p. 265

4. The nurse is caring for a patient who is receiving famotidine for an ulcer. Which statement
indicates that the patient understands about taking famotidine?

a. “I need to eat three large meals a day.”

b. “I will eat a bedtime snack.”

c. “I will try to stop smoking cigarettes.”

d. “I will notify my healthcare provider if I am not feeling better in a month.”

ANS: C
Authentic Nurse Guru: For more of these materials :masterclasstutor1@gmail.com
Visovsky: Introduction to Clinical Pharmacology, 10th Edition 16

The patient should avoid cigarette smoking because it increases gastric acid produce and can
decrease the effectiveness of H2 blockers. The patient should avoid eating within 3 hours of
bedtime to reduce the risk of reflux while lying flat in bed. Recommend that the patient eat
smaller portions at mealtimes. Tell patients to notify their healthcare provider if they have taken
H2-receptor blockers for longer than 2 weeks because these drugs can lose their effectiveness
over time and the patient may need different drugs.

DIF: Cognitive Level: Understanding REF: p. 265

5. A patient is admitted with a diagnosis of dehydration as a result of diarrhea. You enter the
patient’s room to give the ordered dicyclomine (Bentyl). The patient states, “I have benign
prostatic hypertrophy and don’t want to drink a lot of water.” What is your best response?

a. “I must check with your healthcare provider before I can give you the pill.”

b. “I will give you as much water as you need to swallow the pill.”

c. “We will monitor how much you drink as well as how much you urinate.”

d. “You need to drink plenty of water because you are dehydrated.”

ANS: A

Anticholinergic drugs should not be given to patients with a history of GI obstruction, benign
prostatic hypertrophy, or glaucoma because these drugs worsen the conditions. The LPN should
notify the healthcare provider about the benign prostatic hypertrophy. The other statements may
be made but are not a priority.

DIF: Cognitive Level: Applying REF: p. 272

6. A 32-year-old male patient returns from visiting another country and has developed diarrhea
and he plans to self-treat the diarrhea at home. What advice will you give to this patient about
treating diarrhea at home?

a. “You should not self-treat diarrhea for longer than 1 day.”

b. “You should not self-treat diarrhea for longer than 2 days.”

c. “You should not self-treat diarrhea for longer than 3 days.”


Authentic Nurse Guru: For more of these materials :masterclasstutor1@gmail.com
Visovsky: Introduction to Clinical Pharmacology, 10th Edition 16

d. “You should not self-treat diarrhea for longer than 4 days.”

ANS: B

Diarrhea that persists for more than 48 hours (2 days) should not be self-treated. The healthcare
provider should be notified for further evaluation and diagnosis.

DIF: Cognitive Level: Applying REF: p. 272

7. You are caring for an older adult who is experiencing frequent constipation. Which statement
indicates that the patient accurately understands dietary changes that may help prevent
constipation?

a. “I should increase my intake of cheese and hard-boiled eggs.”

b. “I should increase my intake of whole grains and fresh vegetables.”

c. “I should increase my intake of cottage cheese and rice.”

d. “I should increase my intake of foods with a high sugar content.”

ANS: B

Laxatives are not a substitute for good bowel habits including regular physical activity and a diet
that includes high fiber foods such as whole grains, fruits, and fresh vegetables. High sugar-
content foods are known to cause constipation so intake of high-sugar foods should be decreased.
Cheese and hard-boiled eggs are known to cause constipation so intake of these foods should be
decreased. Cottage cheese and rice are known to cause constipation so intake of these foods
should be decreased.

DIF: Cognitive Level: Understanding REF: p. 271

8. Laxatives would be contraindicated for which patient?

a. A 29-year-old patient in traction for a fractured femur

b. A 36-year-old patient who is a paraplegic

c. A 43-year-old patient with severe abdominal pain

d. A 55-year-old patient who is having a colonoscopy tomorrow


Authentic Nurse Guru: For more of these materials :masterclasstutor1@gmail.com
Visovsky: Introduction to Clinical Pharmacology, 10th Edition 16

ANS: C

Never take a laxative to treat severe abdominal pain because the drug may make conditions that
cause the pain such as appendicitis to worsen. Laxatives are used for patients with limited
mobility, neurogenic bowel, or preprocedure bowel cleansing.

DIF: Cognitive Level: Applying REF: p. 271

9. A patient is taking a nonsteroidal anti-inflammatory (NSAID) drug regularly for arthritic pain.
Which drug can help protect the patient’s gastrointestinal mucosa from the effects of an NSAID
drug?

a. Aluminum hydroxide antacid

b. Sucralfate

c. Famotidine

d. Omeprazole

ANS: B

Sucralfate, a cytoprotective drug, protects gastrointestinal mucosa from the effects of NSAIDs.
Aluminum hydroxide antacid helps delay stomach emptying time and binds bile salts. Famotidine,
an H2 antagonist, is widely used in the prophylaxis and treatment of ulcers. Omeprazole, a
proton-pump inhibitor, is used in the short-term treatment of active duodenal ulcers.

DIF: Cognitive Level: Remembering REF: p. 266

10. You are preparing a teaching session for a patient with diabetic gastroparesis prescribed
metoclopramide. Which statement will you include in this teaching plan?

a. “Take this drug 1 hour after meals.”

b. “You should include high-iron foods in your diet.”

c. “Report any changes in your mood or feelings of depression.”

d. “Rotate the injection site each time you take this drug.”

ANS: C
Authentic Nurse Guru: For more of these materials :masterclasstutor1@gmail.com
Visovsky: Introduction to Clinical Pharmacology, 10th Edition 16

Patients should be taught to report changes in mood, especially feelings of depressed mood or
suicidal ideation, as this is a potential adverse effect of promotility drugs.

DIF: Cognitive Level: Applying REF: p. 260

11. You are preparing a teaching plan for a patient who is on a treatment regimen that includes
laxatives. Which adverse reaction(s) are common to bulk-forming laxatives and stool softeners?

a. Muscle weakness and pruritus

b. Hypermagnesemia

c. Abdominal cramping and nausea

d. Cathartic colon

ANS: C

Fecal softeners and bulk-forming laxatives may cause abdominal cramping and nausea. Stimulant
or irritant laxatives may produce muscle weakness and pruritus. Hyperosmolar laxatives may
cause hypermagnesemia in patients with chronic renal insufficiency.

Cathartic colon may develop after long-term or excessive use of stimulant laxatives.

DIF: Cognitive Level: Remembering REF: p. 268

12. You are caring for an elderly patient who has been given 5 mL of the antispasmodic drug
atropine. Which of the following action(s) should you perform to detect adverse reactions of this
drug?

a. Monitor the patient’s heart rate.

b. Give the atropine with the prescribed sedative to enhance sleep.

c. Monitor the patient for serotonin syndrome.

d. Place the patient in Trendelenburg position after giving the drug.

ANS: A

Monitor the patient’s heart rate as antispasmodic drugs can cause bradycardia or tachycardia.
Authentic Nurse Guru: For more of these materials :masterclasstutor1@gmail.com
Visovsky: Introduction to Clinical Pharmacology, 10th Edition 16

DIF: Cognitive Level: Applying REF: p. 269

13. You are caring for a patient with peptic ulcer disease who is receiving pantoprazole 40 mg
daily. The patient asks you how the pantoprazole will help the peptic ulcer heal. What is your best
response?

a. “Pantoprazole reduces or neutralizes gastric acidity.”

b. “Pantoprazole reduces gastric motility and lessens the amount of stomach acid.”

c. “Pantoprazole limits the action of histamine on the receptor cells in the stomach.”

d. “Pantoprazole decreases gastric acidity by blocking the acid secretory pump.”

ANS: D

Pantoprazole is a proton-pump inhibitor. Proton-pump inhibitors decrease gastric acidity by


blocking the acid secretory pump that is located within the gastric parietal cell membrane.
Antacids reduce or neutralize gastric acidity. Antispasmodics and anticholinergics reduce gastric
motility and the amount of stomach acid. H2 antagonists reduce gastric acid secretion by limiting
the action of histamine at the receptor cells in the stomach.

DIF: Cognitive Level: Applying REF: p. 265

14. You are caring for a patient who is experiencing chronic constipation. Which drug would
lower the surface tension in the gastrointestinal tract and facilitate the softening of the fecal mass
by intestinal fluids?

a. Psyllium seed (Metamucil)

b. Docusate (Colace)

c. Polyethylene glycol (Miralax)

d. Bisacodyl (Dulcolax)

ANS: B

Docusate is a fecal softener. Fecal softeners lower the surface tension in the gastrointestinal tract
to facilitate the softening of the fecal mass by intestinal fluid. Psyllium seed is a bulk-forming
Authentic Nurse Guru: For more of these materials :masterclasstutor1@gmail.com
Visovsky: Introduction to Clinical Pharmacology, 10th Edition 16

laxative. Bulk-forming laxatives absorb water and expand thereby increasing both the bulk and
moisture content of the stool. The increased bulk stimulates peristalsis, and the absorbed water
softens the stool. Polyethylene glycol is an osmotic laxative. Osmotic laxatives cause increased
absorption of fluid into the stool. As a result, the stool is softer and distends the colon leading to
peristalsis and easier passage of the stool. Bisacodyl is a stimulant laxative. Stimulant laxatives
increase peristalsis by several mechanisms. Bisacodyl stimulates sensory nerves in the intestinal
mucosa.

DIF: Cognitive Level: Remembering REF: p. 269

15. A patient with acute vomiting is given promethazine (Phenergan) 12.5 mg IM. Which of the
following action(s) should you take to prevent adverse effect of this drug?

a. Monitor the patient for a bulbous rash on the trunk.

b. Monitor the patient’s temperature.

c. Assess the patient for changes in mood.

d. Assess the patient for swelling of the extremities.

ANS: B

The patient given promethazine (Phenergan) IM should be monitored for an increase in


temperature, as this is the symptom of neuroleptic malignant syndrome, a rare, but life-threatening
adverse effect of this drug.

DIF: Cognitive Level: Applying REF: p. 256

16. Which patient is not recommended to receive promethazine (Phenergan) for nausea or
vomiting?

a. A 42-year-old male patient with sleep apnea

b. A 15-year-old female patient who just underwent minor surgery

c. An elderly male patient with dementia

d. A female patient with cardiovascular disease

Authentic Nurse Guru: For more of these materials :masterclasstutor1@gmail.com


Visovsky: Introduction to Clinical Pharmacology, 10th Edition 16

ANS: D

Phenothiazine drugs such as promethazine should be avoided in patients with cardiovascular


disease because they can cause them to experience angina, tachycardia, and/or orthostatic
hypotension.

DIF: Cognitive Level: Understanding REF: p. 256

17. A patient with liver failure has been given the drug Lactulose. What is the purpose of this drug
in liver failure?

a. Lactulose decreases constipation in patients with liver failure.

b. Lactulose reduces the ammonia level in patients with liver failure.

c. Lactulose increases fluid volume in the stool to reduce constipation.

d. Lactulose also has a diuretic action that can decrease fluid in ascites.

ANS: B

Lactulose is used in patients with liver failure to decrease ammonia level. It does not have a
diuretic action. While it does have a laxative effect, it is not the primary reason for using lactulose
in these patients.

DIF: Cognitive Level: Understanding REF: p. 267

18. How do histamine (H2) receptor antagonists help prevent or heal gastric ulcers?

a. Increasing prostaglandins production of thick mucus in the stomach

b. Neutralizing the hydrochloric acid secreted only in the stomach

c. Blocking the activity of the proton (hydrogen ion) pumps in the stomach

d. Reducing the stimulation of cells that produce acid in the stomach

ANS: D

Histamine receptor antagonists act by reducing stimulation of the histamine cells in the stomach.
Proton-pump inhibitors block proton pumps reducing the acid secreted in the stomach. Antacids
Authentic Nurse Guru: For more of these materials :masterclasstutor1@gmail.com
Visovsky: Introduction to Clinical Pharmacology, 10th Edition 16

neutralize the acid in the stomach.

DIF: Cognitive Level: Understanding REF: p. 264

19. A patient has been given an osmotic laxative as part of bowel preparation for a colonoscopy.
How long does this drug usually take to have its effect?

A. 12 hours to 3 days

B. 1 to 3 hours

C. 6 to 8 hours

D. 6 to 12 hours

ANS: B

Osmotic laxatives act rapidly to prepare a bowel for visualization during a colonoscopy. Lubricant
laxatives may act in 6 to 8 hours. Bulk forming drugs typically act within 12 hours to 3 days.
Stimulant laxatives may act in 6 to 12 hours.

DIF: Cognitive Level: Remembering REF: p. 268

MULTIPLE RESPONSE

1. What symptoms are common side effects of dronabinol (Marinol)? (Select all that apply.)

a. Difficulty concentrating

b. Dizziness

c. Flushing of face

d. Increased awareness

e. Muscle cramps

ANS: A, B, D
Authentic Nurse Guru: For more of these materials :masterclasstutor1@gmail.com
Visovsky: Introduction to Clinical Pharmacology, 10th Edition 16

The most common side effects are related to cannabinoid’s effect on the CNS. Some patients
experience a dose-related “high” (easy laughing, elation, and increased awareness). Other CNS
effects are dizziness, anxiety, insomnia, difficulty concentrating, and mood changes. Some
patients may experience emotional liability (wide swings in emotion). These side effects may
decrease after 2 weeks of treatment.

DIF: Cognitive Level: Remembering REF: p. 256

2. The LPN is ready to prepare the oral form of aprepitant. Which are the correct steps for
preparing the oral form of aprepitant. (Select all that apply.)

a. There should be no foam or clumps in the solution.

b. Do not open aprepitant until ready to prepare.

c. Gently swirl solution 20 times to mix it then invert cup.

d. Fill the mixing cup with ice cold drinking water.

e. Refrigerate any remaining solution for the next dose.

ANS: A, B, C

The oral form of aprepitant should not be opened until ready to prepare. At that time, the mixing
cup provided in the drug kit should be filled with room temperature drinking water. Once the
precise amount of water has been added to the mixing cup as directed, pour the contents into the
cup and then snap the lid shut of the mixing cup. Gently swirl the solution 20 times to mix it, then
invert the cup. Do not shake the cup or foaming may occur. Be sure there are no clumps or foam
in the solution. At that time, measure the solution to give to the patient. Discard any remaining
solution.

DIF: Cognitive Level: Applying REF: p. 256

Authentic Nurse Guru: For more of these materials :masterclasstutor1@gmail.com


Visovsky: Introduction to Clinical Pharmacology, 10th Edition 16

Chapter 15: Drugs Affecting the Hematologic System Visovsky: Introduction


to Clinical Pharmacology, 10th Edition

MULTIPLE CHOICE

1. An older patient who takes a daily NSAID for arthritis pain has been prescribed aspirin as an
antiplatelet agent. What information regarding risks associated with aspirin use should be
included in this teaching plan?

a. The risk for urinary retention

b. The risk for cognitive decline

c. The risk for gastrointestinal bleeding

d. The risk for peripheral arterial disease

ANS: C

Most drugs that affect the blood clotting system, such as antiplatelet drugs, have the potential to
cause bleeding, especially in older patients.

DIF: Cognitive Level: Applying REF: p. 281

2. You are teaching a patient who has recently undergone a coronary artery stent placement, and
has now been prescribed clopidogrel. Which statement made by this patient informs you that the
patient requires further teaching?

a. “I will need to take this drug every day for 3 months to prevent clogging of the stent.”

b. “I will report any abnormal bleeding.”

c. “I will avoid herbal supplements while taking this drug.”

d. “I will take this drug with food.”

ANS: A

For patients who have had a stent placed into the coronary artery because of severe narrowing or
blockage of the artery, clopidogrel prevents platelets from sticking to the stent mesh. For these
Authentic Nurse Guru: For more of these materials :masterclasstutor1@gmail.com
Visovsky: Introduction to Clinical Pharmacology, 10th Edition 17

patients, clopidogrel must be taken daily for a year or longer to prevent clots from developing and
plugging up the stent.

DIF: Cognitive Level: Applying REF: p. 280

3. A female patient taking aspirin as an antiplatelet drug calls you to report she may be having an
allergic reaction to the drug. Which statement made by the patient alerts you to a possible adverse
reaction to the drug?

a. “I am having nausea after my meals.”

b. “I am experiencing itching and hives on my back and abdomen.”

c. “My visual acuity has decreased.”

d. “I am having increased urination throughout the night.”

ANS: B

Allergic reactions to aspirin and NSAIDs generally occur within a few hours of taking the drug.
Symptoms of allergic reactions include itching, hives, and runny nose, with more severe reactions
causing swelling of the lips, tongue, or face.

DIF: Cognitive Level: Applying REF: p. 281

4. You are taking the drug history of a 47-year-old male patient who is recovering from a
myocardial infarction, and will be starting on an antiplatelet drug. Which of the following
statements made by the patient alerts you to a potential drug interaction with antiplatelet drugs?

a. “I am taking a beta blocker for hypertension.”

b. “I am taking furosemide for fluid retention.”

c. “I take an antacid daily for my reflux.”

d. “I take an inhaled corticosteroid for my asthma.”

ANS: C

Antacids interfere with the action of antiplatelet drugs, and should be taken 1 to2 hours before
taking an antiplatelet drug.
Authentic Nurse Guru: For more of these materials :masterclasstutor1@gmail.com
Visovsky: Introduction to Clinical Pharmacology, 10th Edition 17

DIF: Cognitive Level: Applying REF: p. 281

5. A patient prescribed an anticoagulant asks you about potential drug interactions with this new
drug. Which of the following drugs would you advise this patient to avoid while taking
anticoagulants?

a. Vitamin C

b. Thiazide diuretics

c. Potassium supplements

d. Nonsteroidal anti-inflammatory analgesics

ANS: D

Nonsteroidal anti-inflammatory drugs (NSAIDs) will increase the risk of bleeding and
hemorrhage in a patient receiving anticoagulants.

DIF: Cognitive Level: Applying REF: p. 281

6. You are reviewing the laboratory values of an outpatient who is on anticoagulant therapy. The
laboratory tests show a less than desired level of anticoagulation. The patient states that the drug
has been taken as prescribed. You suspect the issue may be related to the patient’s dietary intake.
Which foods would you advise the patient to avoid while on anticoagulant therapy?

a. Spinach and broccoli

b. Peaches and plums

c. Carrots and celery

d. Sweet potatoes and white rice

ANS: A

The patient should avoid excessive amounts of foods high in vitamin K (spinach, broccoli,
cabbage, kale, dark leafy greens, and asparagus).

DIF: Cognitive Level: Applying REF: p. 281

Authentic Nurse Guru: For more of these materials :masterclasstutor1@gmail.com


Visovsky: Introduction to Clinical Pharmacology, 10th Edition 17

7. You are taking a drug history from a patient who is taking an anticoagulant. The patient
informs you that the herbal supplement St. John’s wort is taken daily for depression. What should
you teach the patient about the use of this and other herbal supplements with anticoagulant
therapy?

a. Herbal drugs may decrease clotting times.

b. Herbal drugs may increase the risk of bleeding.

c. There are no risks associated with this drug combination.

d. They are safe to take together if you take them an hour apart.

ANS: B

St. John’s wort, as well as other herbal drugs, may increase the risk of bleeding in patients on
anticoagulants, and thus, should be avoided.

DIF: Cognitive Level: Applying REF: p. 281

8. A patient with atrial fibrillation has been prescribed the drug dabigatran (Pradaxa). The patient
asks you what the advantage of this drug may be over warfarin (Coumadin). What is your best
response?

a. “This drug is taken only once daily.”

b. “This drug does not carry a risk for excessive bleeding.”

c. “This drug does not require frequent laboratory testing.”

d. “This drug does not interact with other drugs you may be taking.”

ANS: C

The advantage of direct thrombin inhibitors (DTIs) is that they do not require the frequent
laboratory blood testing as part of the monitoring process that is required by warfarin.

DIF: Cognitive Level: Applying REF: p. 282

9. You are caring for a patient who is taking the indirect thrombin inhibitor clopidogrel following
cardiac surgery. What laboratory value should you monitor for possible adverse effects of this
Authentic Nurse Guru: For more of these materials :masterclasstutor1@gmail.com
Visovsky: Introduction to Clinical Pharmacology, 10th Edition 17

therapy?

a. Platelet count

b. Potassium level

c. Serum calcium level

d. White blood cell count

ANS: A

The most common adverse reactions from indirect thrombin inhibitors are excessive bleeding and
thrombocytopenia, so monitoring of the patient’s platelet count is needed.

DIF: Cognitive Level: Applying REF: p. 280

10. A 52-year-old male patient who has been taking a direct thrombin inhibitor for several months
tells you he is going on vacation. He asks what specific advice he should follow related to this
specific class of drugs while he is traveling?

a. “Keep this drug in its original bottle while traveling.”

b. “Stop this drug while on vacation to avoid the risk of bleeding.”

c. “Take this drug every other day to prevent adverse events while on vacation.”

d. “Make sure you eat plenty of green vegetables while on vacation.”

ANS: A

Instruct patients to keep DTIs in the original bottle to protect the drug from moisture and light.
Teach them not to put DTIs in pill boxes or pill organizers, as they are sensitive to light.

DIF: Cognitive Level: Applying REF: p. 283

11. You are caring for a 35-year-old female patient who has been prescribed apixaban (Eliquis).
She reports that she experienced unusually heavy bleeding during her menstrual period. What is
your best response?

a. “This bleeding is expected during the first month of taking the drug.”
Authentic Nurse Guru: For more of these materials :masterclasstutor1@gmail.com
Visovsky: Introduction to Clinical Pharmacology, 10th Edition 17

b. “I will let your healthcare provider know right away.”

c. “There is no need for concern, this side effect is expected.”

d. “I will document this finding in my notes and check with you next month.”

ANS: D

Excessive bleeding from menses is an other signs of internal bleeding include bleeding from the
gums while brushing teeth excessive bleeding or oozing from cuts, and unexplained bruising or
nosebleeds.

DIF: Cognitive Level: Applying REF: p. 279

12. What category of anticoagulant drugs works by increasing the amount of the protein
antithrombin III?

a. Fibrinolytics

b. Vitamin K antagonists

c. Direct thrombin inhibitors

d. Indirect thrombin inhibitors

ANS: D

Indirect thrombin inhibitors (ITIs) are anticoagulant drugs that decrease clot formation by
increasing the amount and action of a protein called antithrombin III. This protein inhibits
thrombin from doing its job in the blood clotting cascade, and clot formation is reduced.

DIF: Cognitive Level: Remembering REF: p. 283

13. A patient with a history of deep vein thrombosis is prescribed subcutaneous heparin before
surgery. The patient asks you if the heparin can be taken orally instead of by injection. What is
your best response?

a. “Oral heparin is quite expensive, and not covered by insurance.”

b. “Heparin is given by injection because it cannot be absorbed orally.”

Authentic Nurse Guru: For more of these materials :masterclasstutor1@gmail.com


Visovsky: Introduction to Clinical Pharmacology, 10th Edition 17

c. “Heparin is given by injection before surgery because it works faster.”

d. “Heparin cannot be given orally because you are fasting for your surgery.”

ANS: B

Heparin cannot be absorbed orally, and so is given only by injection.

DIF: Cognitive Level: Applying REF: p. 284

14. You are reviewing the laboratory values of a patient whose heparin dose has been changed by
the healthcare provider. Which laboratory value will inform you about effect of the dose change
on this patient’s anticoagulant therapy?

a. PT

b. INR

c. aPTT

d. DIC panel

ANS: C

Therapy with heparin sodium must be monitored for its anticoagulation effect by a blood test
known as the activated partial thromboplastin time (aPTT). The prescriber maintains or adjusts
dosages according to this test result.

DIF: Cognitive Level: Remembering REF: p. 284

15. A patient is suspected of having an overdose of heparin. What drug should you prepare for the
healthcare provider to give?

a. Naloxone

b. Warfarin

c. Acetylcysteine

d. Protamine sulfate

ANS: D Nurse Guru: For more of these materials :masterclasstutor1@gmail.com


Authentic
Visovsky: Introduction to Clinical Pharmacology, 10th Edition 17

Protamine sulfate is a strongly basic (alkaline) protein that acts as an antagonist to neutralize
(reverse) the actions of heparin.

DIF: Cognitive Level: Remembering REF: p. 280

16. A female patient prescribed subcutaneous heparin tells you that he has noticed mild bleeding
from the gums when he brushed his teeth. What is your best response?

a. “I will hold the drug and notify your healthcare provider.”

b. “This is a fairly common and expected side effect of the drug.”

c. “Bleeding from the gums indicates the drug is working properly.”

d. “I will have to give you a shot of protamine sulfate to reverse the drug’s action.”

ANS: B

Signs of mild bleeding and bruising can be an expected side effect of heparin. Teach patients to
report heavy, abnormal bleeding, including heavy menses, to the healthcare provider because
these may indicate overdosage.

DIF: Cognitive Level: Applying REF: p. 281

17. You are preparing to draw up a dose of heparin to give subcutaneously to a patient. Which of
the following represents the best procedure in preparing this injection?

a. Roll the vial between your hands to distribute the solution.

b. Insert the needle and then turn the vial upside down to distribute the drug.

c. Shake the vial vigorously to adequately distribute the drug molecules.

d. Carefully draw the solution up from the vial without disturbing the contents.

ANS: A

Do not shake the bottle containing the heparin; only roll it carefully between your hands before
inserting the needle. If the heparin solution is discolored or contains a precipitate or particles at
the bottom of the bottle, do not use it.

Authentic Nurse Guru: For more of these materials :masterclasstutor1@gmail.com


Visovsky: Introduction to Clinical Pharmacology, 10th Edition 17

DIF: Cognitive Level: Remembering REF: p. 284

18. The aPTT of a patient who is prescribed continuous intravenous (IV) heparin is two times the
control value. What is your best action?

a. Increase the IV rate as ordered.

b. Decrease the IV rate as ordered.

c. Leave the rate unchanged.

d. Stop the infusion and notify the prescriber.

ANS: C

The goal of continuous heparin therapy is to keep the aPTT within a therapeutic range of 1.5 to
2.5 times greater than the laboratory-established control value. Two times the control value is
within this range. The prescriber should be notified, but the infusion rate, which is therapeutic,
will not change at this time.

DIF: Cognitive Level: Applying REF: p. 284

19. A patient taking an oral anticoagulant has had an international normalized ratio (INR) drawn
to check the therapeutic drug response. Which range represents the therapeutic INR range for a
patient taking an oral anticoagulant?

a. 1.0 to 2.0

b. 2.0 to 3.0

c. 3.0 to 4.0

d. 4.0 to 5.0

ANS: B

The normal range for the INR is 0.8 to 1.2, with a therapeutic target range of 2.0 to 3.0. For
patients with mechanical heart valves, this therapeutic range is slightly higher at 2.5 to 3.5,
because of the high risk for clots forming within the valve itself.

DIF: Cognitive Level: Remembering REF: p. 285


Authentic Nurse Guru: For more of these materials :masterclasstutor1@gmail.com
Visovsky: Introduction to Clinical Pharmacology, 10th Edition 17

20. A young mother who is breast-feeding her 3-month old baby now requires treatment with
heparin for a deep vein thrombosis. The patient asks you if she should continue breast-feeding
during her heparin treatment. What is your best response?

a. “You will need to stop breast-feeding during treatment with heparin.”

b. “Heparin is not found in breast milk, so you may continue breast-feeding.”

c. “The half-life of heparin is short, so breast-feed before your daily heparin injection.”

d. “Pump your breasts to keep breast milk flowing so you can resume breast-feeding later.”

ANS: B

Breast-feeding is safe during heparin therapy because the drug is not found in breast milk.

DIF: Cognitive Level: Applying REF: p. 285

21. An obese, menopausal woman is beginning warfarin (Coumadin) therapy. Which adverse
effect should you monitor this patient for in the first 10 days of drug therapy?

a. Neutropenia

b. Skin necrosis

c. Severe diarrhea

d. Pulmonary edema

ANS: B

Warfarin can cause skin necrosis (death) that can occur within the first 10 days of therapy and is
associated with larger dosages. Obese, menopausal women are at greatest risk for this rare adverse
reaction.

DIF: Cognitive Level: Knowing REF: p. 285

22. You are assessing a patient who is on long-term warfarin (Coumadin) therapy who has been
diagnosed with a warfarin overdosage. What drug would you anticipate would be needed as the
antidote for warfarin overdose?

Authentic Nurse Guru: For more of these materials :masterclasstutor1@gmail.com


Visovsky: Introduction to Clinical Pharmacology, 10th Edition 17

a. Paroxetine

b. Phytonadione

c. Protamine sulfate

d. Prothrombin factor

ANS: B

In response to some bleeding disorders or warfarin overdosage, vitamin K, phytonadione


(AquaMEPHYTON), may be given either orally or parenterally to help stimulate the liver to
resume manufacture of prothrombin and serve as an anticoagulant antagonist. However, this
clotting activity may not return for 48 to 72 hours.

DIF: Cognitive Level: Knowing REF: p. 286

23. A patient has been prescribed a fibrinolytic drug. What response to the patient best describes
the action of this class of drug?

a. “This drug works to prevent blood clot formation.”

b. “This drug acts to reverse the effects of heparin.”

c. “This drug works to lyse existing blood clots.”

d. “This drug thins out the blood.”

ANS: C

Fibrinolytic drugs convert plasminogen to the enzyme plasmin, which breaks down fibrin clots,
fibrinogen, and other plasma proteins, and thus, they dissolve and break down existing blood
clots.

DIF: Cognitive Level: Knowing REF: p. 287

24. Which statement about fibrinolytic drugs is true?

a. These high-alert drugs are given by IV infusion.


Authentic Nurse Guru: For more of these materials :masterclasstutor1@gmail.com
Visovsky: Introduction to Clinical Pharmacology, 10th Edition 18

b. These drugs can be used in the outpatient setting.

c. These drugs increase cellular damage from clots blocking the arteries.

d. These drugs are contraindicated in patients diagnosed with an acute MI.

ANS: A

All fibrolytic drugs are given IV and are high-alert drugs, and are used for several purposes,
including acute MI. These drugs decrease cellular damage and are used only in a critical care
setting.

DIF: Cognitive Level: Knowing REF: p. 287

25. A patient is brought to the hospital with chest pain and shortness of breath is suspected of
having an acute MI. Which assessment is a priority for this patient with a suspected acute MI who
may be given a fibrinolytic drug as treatment?

a. Ask the patient or family when the symptoms first began.

b. Ask the patient or family for a list of all drugs taken that day.

c. Ask the patient or family when the last meal was eaten.

d. Ask the patient or family to report any history of kidney disease present.

ANS: A

Timing is a critical factor in using these drugs. If fibrinolytics are begun within 12 hours of a heart
attack, or 3 hours of the onset of a stroke, the blood clot blocking the artery can be dissolved, and
blood flow restored.

DIF: Cognitive Level: Applying REF: p. 288

26. You are preparing to give an injection of an erythropoietin stimulating agent to a patient. The
patient asks you to explain the expected side effects of this injection. What is your best response?

a. “Thickening of the blood”

b. “Infection at the injection site”

Authentic Nurse Guru: For more of these materials :masterclasstutor1@gmail.com


Visovsky: Introduction to Clinical Pharmacology, 10th Edition 18

c. “Pain at the site of the injection”

d. “Drowsiness and lack of concentration”

ANS: C

Pain at the injection site is the most common side effect of ESAs. Generalized body aches and
pain, skin rash, redness, or warmth at the injection site can occur. Thickening of the blood leading
to hypertension or stroke is an adverse effect, not a common side effect.

DIF: Cognitive Level: Applying REF: p. 289

27. A patient is about to begin treatment with epoetin alfa. Which patient assessment parameter
should you monitor closely to prevent complications from this drug?

a. The patient’s urine output

b. The patient’s blood pressure

c. The patient’s blood glucose level

d. The patient’s serum potassium level

ANS: B

The use of ESAs is not without significant risks. As RBC production increases, the blood itself
becomes thicker. This can result in a higher risk for hypertension, blood clots, stroke, and MI.

DIF: Cognitive Level: Applying REF: p. 290

28. A patient with a clotting disorder is prescribed an anticoagulant and asks you to explain the
purpose of anticoagulant therapy.

a. Anticoagulants are used to prevent the formation of Vitamin K in the liver.

b. Anticoagulants are used to lyse existing clots.

c. Anticoagulants are used to increase the flow of blood.

d. Anticoagulants are used to thin the viscosity of the blood.

e. Anticoagulants
Authentic are usedFor
Nurse Guru: to prevent
more emboli frommaterials
of these traveling through the bloodstream.
:masterclasstutor1@gmail.com
Visovsky: Introduction to Clinical Pharmacology, 10th Edition 18

f. Anticoagulants are used to prevent new clot formation.

ANS: F

Anticoagulants are drugs that interfere with the clotting process, so they are used to reduce
existing clots or to prevent new clots from forming.

DIF: Cognitive Level: Remembering REF: p. 279

MULTIPLE RESPONSE

1. A 63-year-old male patient with a history of atrial fibrillation who has been taking an
anticoagulant comes into the clinic for a checkup. What signs or symptoms would you expect to
see in a patient who has developed adverse effects related to anticoagulant therapy? List the
correct responses. (Select all that apply.)

a. Hypertension

b. Shortness of breath

c. Elevated potassium level

d. Bleeding gums

e. Night sweats

f. Tarry colored stools

g. Bruising

h. Bloody urine

ANS: B, D, F, G, H

For any patient who is taking anticoagulants, watch for early signs of bleeding: easy bruising of
knuckles, elbows, or any body part experiencing pressure (e.g., under watch band), new or
excessive bleeding of gums when brushing teeth, blood in the urine or stool, or tarry-colored
stool, tachycardia, hypotension, shortness of breath, and gastrointestinal pain.

DIF: Cognitive Level: Remembering REF: p. 281

Authentic Nurse Guru: For more of these materials :masterclasstutor1@gmail.com


Visovsky: Introduction to Clinical Pharmacology, 10th Edition 18

2. A 63-year-old male patient is being discharged from the hospital after being treated for rapid
atrial fibrillation. The healthcare provider treating him mentioned that he would be discharged
today and begin treatment with a direct thrombin inhibitor. List the drugs belonging to the direct
thrombin inhibitor class of anticoagulants. (Select all that apply.)

a. Acetylsalicylic acid

b. Apixaban

c. Clopidogrel

d. Dabigatran

e. Heparin

f. Protamine sulfate

g. Rivaroxaban

h. Warfarin

ANS: B, D, G

Apixaban, Dabigatran, and Rivaroxaban are agents considered to be direct thrombin inhibitors.

DIF: Cognitive Level: Remembering REF: p. 282

3. You are preparing to give an injection of heparin to a patient. Which of the following steps
represent the correct procedure? (Select all that apply.)

a. Insert the needle, pull back the plunger, and aspirate before injecting.

b. Give subcutaneous heparin in the same general area each time.

c. Do not massage the injection site after injecting the heparin.

d. Draw up the heparin and check the dose with another nurse.

e. Use a 25-gauge needle to give heparin subcutaneously.

f. Give the heparin IM ventrogluteal muscle.

Authentic Nurse Guru: For more of these materials :masterclasstutor1@gmail.com


Visovsky: Introduction to Clinical Pharmacology, 10th Edition 18

ANS: C, D, E

When injecting subcutaneous heparin, do not pull back on the syringe to aspirate for blood or
move the needle in the tissue during the injection. Do not massage the injection site. All of these
actions increase the risk for bleeding, bruising, and tissue damage at the injection site. Heparin is
not given IM.

DIF: Cognitive Level: Knowing REF: p. 284

4. You suspect a patient taking warfarin (Coumadin) has developed internal bleeding. What
symptoms of this condition would you expect the patient to exhibit? (Select all that apply.)

a. Abdominal pain

b. Dizziness

c. Tachycardia

d. Warm, flushed skin

e. Hypertension

ANS: A, B, C

Signs that suggest internal bleeding include: abdominal pain or swelling, back pain, or
constipation (resulting from paralytic ileus or intestinal obstruction); bloody or tarry stools,
bloody or dark-colored urine, coughing up or vomiting blood or “coffee-ground” substance;
dizziness or cold, clammy skin; severe or continuous headache; and tachycardia (fast pulse),
hypotension (low blood pressure), and tachypnea (rapid breathing).

DIF: Cognitive Level: Knowing REF: p. 281

5. For which conditions would the use of fibrinolytic drugs be appropriate? (Select all that apply.)

a. Ischemic stroke

b. Hypotensive crisis

c. Acute arterial occlusion

d. Acute pulmonary embolism


Authentic Nurse Guru: For more of these materials :masterclasstutor1@gmail.com
Visovsky: Introduction to Clinical Pharmacology, 10th Edition 18

e. Cellular damage from an MI

f. Treatment of warfarin overdose

ANS: A, C, D, E

Fibrinolytic drugs are used in acute myocardial infarction for lysis of thrombi that block coronary
arteries, in acute pulmonary embolism for clot lysis when the patient is hemodynamically
unstable, in acute ischemic stroke, and in acute arterial occlusion. These drugs reduce the extent
of cellular damage from blockage.

DIF: Cognitive Level: Remembering REF: p. 287

Authentic Nurse Guru: For more of these materials :masterclasstutor1@gmail.com


Visovsky: Introduction to Clinical Pharmacology, 10th Edition 18

Chapter 16: Drugs for Immunization and Immunomodulation Visovsky:


Introduction to Clinical Pharmacology, 10th Edition

MULTIPLE CHOICE

1. Which type of immunity serves to protect against day-to-day pathogen exposure?

a. Innate immunity

b. Natural immunity

c. Acquired immunity

d. Supplemental immunity

ANS: A

Innate immunity helps protect you from smaller day-to-day exposures to pathogenic organism but
cannot provide long-term immunity to any single specific disease-causing microorganism.

DIF: Cognitive Level: Remembering REF: p. 295

2. In acquired immunity, what is the body’s response to antigen exposure?

a. The body initiates the clotting cascade.

b. The body produces additional antigens.

c. The body produces an antibody response.

d. The body increases antigen response through lymphocyte action.

ANS: C

Exposure to antigens is the trigger for lymphocytes to begin producing antibodies. These
antibodies can be made in such high amounts that, when you are reinfected by the same
microorganism, they attack and destroy it or rid the body of it before it can make you sick again.

DIF: Cognitive Level: Remembering REF: p. 295

3. Which of the following is an example of acquired passive immunity?


Authentic Nurse Guru: For more of these materials :masterclasstutor1@gmail.com
Visovsky: Introduction to Clinical Pharmacology, 10th Edition 18

a. Immunity developed from microorganism exposure through vaccination.

b. Premade antibodies are transferred to you by another person or an animal.

c. Microorganism exposure that makes you sick, and then develop antibodies.

d. A pregnant woman who transfers antibodies to her baby during breast-feeding.

ANS: D

Natural acquired passive immunity is composed of the antibodies that a woman transfers to her
fetus during pregnancy and to her infant during breast-feeding. This immunity is short-term but
critically important in preventing young infants from many illnesses during the first 6 months
after birth.

DIF: Cognitive Level: Remembering REF: p. 296

4. Which type of vaccine contains antigens that are killed by heat, radiation, or chemicals?

a. A toxoid

b. An antiserum

c. Inactivated vaccine

d. Attenuated vaccine

ANS: C

Laboratories can produce inactivated vaccines that contain dead antigens so that people can be
immunized to prevent them from getting the disease.

DIF: Cognitive Level: Remembering REF: p. 297

5. The parent of an infant you are caring for asks you for the correct schedule for the diphtheria,
tetanus, and pertussis (DTaP) vaccine schedule. What is your best response?

a. “DTaP is given at 2, 4, and 6 months of age.”

b. “DTaP is given at 1, 3, and 5 months of age.”

c. “DTaPNurse
Authentic is givenGuru:
at 6, 12,For
andmore
18 months of age.”materials :masterclasstutor1@gmail.com
of these
Visovsky: Introduction to Clinical Pharmacology, 10th Edition 18

d. “DTaP is given to children when they are ready to begin school.”

ANS: A

The correct schedule for the DTaP is at 2, 4, and 6 months of age, with a booster at 15 to 18
months.

DIF: Cognitive Level: Applying REF: p. 298

6. Which statement made by an adult patient demonstrates understanding of the recommended


Tdap vaccine booster schedule?

a. “I will need a booster every 5 years”

b. “I will need a booster every 7 years.”

c. “I will need a booster every 10 years.”

d. “I will need a booster every 15 years.”

ANS: C

For adults aged 19 and older, Tdap boosters are recommended every 10 years.

DIF: Cognitive Level: Applying REF: p. 299

7. An adult patient scheduled for a seasonal influenza vaccine presents to the clinic with
complaints of an upper respiratory infection. He is also there for a flu shot. What is your best
action?

a. Give a “test dose” of the vaccine to observe if a reaction is present.

b. Give the influenza vaccine to avoid more severe infection.

c. Have the patient sign a consent for the vaccine.

d. Hold the vaccine until the infection resolves.

ANS: D

Vaccines should not be given to patients with active infection, severe febrile illness, or history of
a seriousNurse
Authentic side effect from For
Guru: previous
more vaccinations.
of these materials :masterclasstutor1@gmail.com
Visovsky: Introduction to Clinical Pharmacology, 10th Edition 18

DIF: Cognitive Level: Applying REF: p. 300

8. A pregnant woman presents for her regular obstetric appointment and requests to have the
seasonal influenza vaccine. What is your best response?

a. “This vaccine contains a live virus, so cannot be given during pregnancy.”

b. “This vaccine can be given safely anytime during your pregnancy.”

c. “You may only receive this vaccine during your last trimester.”

d. “You may only receive this vaccine during your first trimester.”

ANS: B

Vaccinations that are recommended during pregnancy include seasonal influenza and Tdap.

DIF: Cognitive Level: Applying REF: p. 301

9. You are preparing to give an 18-month-old child the measles, mumps, and rubella (MMR)
vaccine. The child’s mother informs you that a relative who is HIV-positive has just moved into
the home with this family. What is your best action?

a. Withhold the vaccine, as it contains live microorganisms.

b. Give the vaccine as ordered; there is no effect on the HIV-positive person.

c. Give one-half of the vaccine dose now and schedule the remaining dose at a later date.

d. Have the HIV-positive person contact his/her healthcare provider before giving the vaccine.

ANS: D

People who are immunocompromised are at increased risk for an adverse reaction after
administration of live attenuated vaccines because they have less of an ability to build up an
effective immune response. Before receiving a vaccination with a live vaccine, the household
member’s healthcare provider should be consulted because the patient with reduced immunity
may be at increased risk for contracting the virus the vaccine is designed to prevent.

DIF: Cognitive Level: Applying REF: p. 301

Authentic Nurse Guru: For more of these materials :masterclasstutor1@gmail.com


Visovsky: Introduction to Clinical Pharmacology, 10th Edition 19

10. An adult patient who has received the series of vaccines against varicella zoster asks you if
immunity is now present against this disease. What is your best response?

a. “Immunity can only be determined by obtaining a blood titer.”

b. “You are considered immune if you have completed the vaccine series.”

c. “You will be considered immune one month after your last injection.”

d. “The health department will review the dates of your vaccines and let you know.”

ANS: A

Antibody blood titers taken after the varicella zoster virus (VZV) vaccine series (called anti-VZV
antibody) can confirm the presence of antibodies and immunity.

DIF: Cognitive Level: Applying REF: p. 299

11. An adult patient who is the recipient of a kidney transplant is prescribed the antirejection drug
mycophenolate. What potential problem should you monitor this patient for?

a. Fluid retention and edema

b. Signs and symptoms of infection

c. Changes in cognition and memory

d. Decreased pulse rate and hypotension

ANS: B

All selective immunosuppressants reduce immunity to some extent and increase the patient’s risk
for infection. With reduced immunity and inflammation, the symptoms of infection may not be
present even when the patient has a significant infection.

DIF: Cognitive Level: Applying REF: p. 302

12. You are teaching a patient who is beginning an antiproliferative drug about potential toxicities
associated with this class of drugs. Which statement should be included in the teaching plan for
this patient?

Authentic Nurse Guru: For more of these materials :masterclasstutor1@gmail.com


Visovsky: Introduction to Clinical Pharmacology, 10th Edition 19

a. “Stay out of the sun and use sunscreen daily.”

b. “Eat high-fiber foods and increase your fluid intake.”

c. “Avoid drinking alcohol and taking acetaminophen.”

d. “These drugs can cause bleeding at the administration site.”

ANS: C

Liver toxicity and liver failure have occurred with all of the antiproliferatives and the calcineurin
inhibitors. The risk is increased if the patient has other liver problems or is exposed to other
substances that are liver toxic, such as alcohol and acetaminophen.

DIF: Cognitive Level: Applying REF: p. 304

13. A woman of childbearing age has just been prescribed an antiproliferative drug. What
statement made by this patient alerts you to the need for additional teaching?

a. “I can plan to become pregnant after I am on this drug for a minimum of 6 months.”

b. “I will use two reliable forms of birth control while taking this drug.”

c. “I will report any yellowing of my eyes or skin.”

d. “I will take my temperature daily.”

ANS: B

Pregnancy is an absolute contraindication for the use of antiproliferative drugs because they are
associated with birth defects and other severe problems. Tell sexually active women of
childbearing age to use two reliable methods of contraception during this therapy and for at least
12 weeks after the therapy is discontinued.

DIF: Cognitive Level: Applying REF: p. 303

14. You are working at a pediatric clinic and preparing to give vaccinations to 3 children of
differing ages: 1-month-old male infant, 15-month-old toddler, and a 4-year-old. The most
appropriate vaccination for the 1-month-old infant is ------------ --. According to recommended
vaccination schedules, the 15-month-old toddler may be due for ------ --. The 4-year-old should
Authentic Nurse Guru: For more of these materials :masterclasstutor1@gmail.com
Visovsky: Introduction to Clinical Pharmacology, 10th Edition 19

receive ------- --.

a. Inactivated poliovirus (IPV); HPV, RV, PPSV23; IPV, MMR, DTaP, and/or VAR

b. Pneumococcal conjugate (PCV13); HPV, RV, PPSV23; HepA and Tdap

c. hepatitis B; MMR, DTaP, and/or Varicella (VAR); IPV, MMR, DTaP, and/or

VAR

d. rotavirus (RV); HPV, RV, PPSV23; HepA and Tdap

ANS: C

Recommended vaccinations for children aged 1 month: Hepatitis B. Recommended vaccinations


for children aged 15 months: Hepatitis B, DTaP, Hib, PCV13, IPV, MMR, Varicella, Hep A.
Recommended vaccinations for a 4 year old: DTaP, IPV, MMR, Varicella.

DIF: Cognitive Level: Understanding REF: p. 298

MULTIPLE RESPONSE

1. Which of the following are examples of a toxoid that has been modified to be used as a
vaccine? (Select all that apply.)

a. Human papilloma virus

b. Hepatitis B

c. Pertussis

d. Tetanus

e. Rubella

f. Polio

ANS: A, B, C, D

Disease for which toxoid vaccines are commonly used includes tetanus, diphtheria, pertussis,
Authentic Nurse Guru: For more of these materials :masterclasstutor1@gmail.com
Visovsky: Introduction to Clinical Pharmacology, 10th Edition 19

human papilloma virus (HPV), and hepatitis B (HBV).

DIF: Cognitive Level: Remembering REF: p. 297

2. The LPN/LVN is educating a patient about the difference between vaccinations for artificial
acquired active immunity and vaccinations for artificial acquired passive immunity. Which of the
following statements made by the LPN/LVN are true? (Select all that apply.)

a. Artificial acquired active immunity vaccines require periodic booster shots.

b. Artificial acquired active immunity vaccination is immediate, but only temporary.

c. Artificial acquired active immunity vaccines can cause reactions at the injection site.

d. Artificial acquired passive immunity occurs when a woman transfers antibodies to her fetus
during pregnancy.

e. An example of when artificial acquired passive immunity is necessary is after a rabies


exposure.

f. Artificial acquired passive immunity vaccines are only used when a person is exposed to and
highly likely to be infected with a microorganism that can cause serious disease.

ANS: C, E, F

Vaccinations for artificial acquired active immunity usually require more than one injection to
ensure there are enough B cells to the specific antigen and can begin making antibodies.
Additional vaccinations (boosters) containing smaller doses of the original antigens are needed to
continue immunity. These vaccines are given to adults and children, and are used to protect the
population against disease outbreak. Reactions at the injection site are common and usually mild.
Vaccination for artificial acquired passive immunity is the type of immunity that is transferred as
pre-made antibodies and is used only after a person is exposed to and highly likely to be infected
with a microorganism that can cause serious disease, such as after an exposure to rabies,
poisonous snakebite, tetanus, or Ebola.

DIF: Cognitive Level: Remembering REF: p. 296

3. Which of the following selective immunosuppressants can cause an increase in serum


cholesterol levels? (Select all that apply.)
Authentic Nurse Guru: For more of these materials :masterclasstutor1@gmail.com
Visovsky: Introduction to Clinical Pharmacology, 10th Edition 19

a. Mycophenolate

b. Methotrexate

c. Cyclosporine

d. Everolimus

e. Tacrolimus

f. Sirolimus

ANS: C, D, E, F

Sirolimus and everolimus, and the calcineurin inhibitors cyclosporine and tacrolimus all increase
blood cholesterol levels, which leads to hypertension.

DIF: Cognitive Level: Remembering REF: p. 304

4. A patient in an ALF is taking oral cyclosporine. The LPN/LVN is preparing the drug for
administration. List the correct nursing implications for this drug. (Select all that apply.)

a. The LPN/LVN should monitor the patient’s kidney and liver function.

b. Mix the solution in a glass container.

c. Mix each drug with the recommended solution of milk or juice.

d. This drug should be taken with grapefruit juice for best absorption

e. Rinse the container with the same solution used to prepare it.

f. Mix oral preparations together using only water.

ANS: A, B, C, E

Cyclosporine can cause significant kidney and liver toxicity so it is important to monitor kidney
and liver function of patients taking this drug. When preparing this drug for administration, the
drug should be mixed exactly as directed, using the recommended solution (milk, orange juice,
apple juice) not water. After the patient drinks the suspension, the container should be rinsed with
the same solution and the patient should drink the rinse for better drug effectiveness. It’s
Authentic Nurse Guru: For more of these materials :masterclasstutor1@gmail.com
Visovsky: Introduction to Clinical Pharmacology, 10th Edition 19

important to remember that cyclosporine must be mixed in a glass container, not a plastic one.

DIF: Cognitive Level: Applying REF: p. 303

5. A 26-year-old patient presents for her first prenatal visit. She asks the nurse about vaccinations
during pregnancy. List the vaccinations that should not be given during pregnancy. (Select all that
apply.)

a. Polio

b. Hepatitis A

c. Tdap

d. Influenza

e. Varicella

f. Hepatitis B

ANS: A, E

Live viruses such as varicella, polio, and measles, mumps, and rubella should not be given during
pregnancy.

DIF: Cognitive Level: Remembering REF: p. 301

Authentic Nurse Guru: For more of these materials :masterclasstutor1@gmail.com


Visovsky: Introduction to Clinical Pharmacology, 10th Edition 19

Chapter 17: Drugs for Osteoporosis and Hormonal Problems Visovsky: Introduction to
Clinical Pharmacology, 10th Edition

MULTIPLE CHOICE

1. A patient diagnosed with hypothyroidism has been prescribed treatment with a thyroid
hormone agonist. The patient asks you how long this drug will need to be taken. What is your best
response?

a. “You will need to take this drug for the rest of your life.”

b. “You will need to take this drug until your symptoms improve.”

c. “You will need to take this drug until your thyroid hormone level normalizes.”

d. “You will need to take this drug incrementally until you reach the maximum tolerated dose.”

ANS: A

Hypothyroidism is a common problem in which the thyroid gland produces little or no thyroid
hormones, slowing all aspects of metabolism, and is treated with thyroid hormone agonists.
Usually this therapy is needed for the rest of the person’s life.

DIF: Cognitive Level: Applying REF: p. 313

2. A patient taking levothyroxine sodium is pregnant and tells you she plans to breast-feed her
baby. What should you tell this patient?

a. “While it is safe to breast-feed, this drug will limit your milk supply over time.”

b. “Breast-feeding is safe when taking this drug, but not with other thyroid hormones.”

c. “You should not breast-feed your infant because this drug passes into your breast milk.”

d. “You may breast-feed while hospitalized and your thyroid hormone levels can be checked.”

ANS: C

Women taking thyroid hormone agonists are advised not to breast-feed, as the drug can be found
in the mother’s breast milk.

Authentic Nurse Guru: For more of these materials :masterclasstutor1@gmail.com


Visovsky: Introduction to Clinical Pharmacology, 10th Edition 19

DIF: Cognitive Level: Applying REF: p. 315

3. The action of which drug is known to be increased by thyroid hormone replacement drugs?

a. Warfarin (Coumadin)

b. Acetaminophen (Tylenol)

c. Diphenhydramine (Benadryl)

d. Fiber supplements (FiberCon)

ANS: A

Thyroid agonists used for thyroid hormone replacement therapy enhance the action of drugs that
reduce blood clotting (anticoagulants), especially warfarin. Acetaminophen is a pain reliever.
Diphenhydramine is an antihistamine and fiber supplements are dietary supplements, not drugs,
and are unaffected by thyroid hormone replacement therapy although their use can inhibit
intestinal uptake of thyroid agonists.

DIF: Cognitive Level: Remembering REF: p. 314

4. Which assessment is most important to perform before giving an elderly patient who is
prescribed levothyroxine (Synthroid) the first dose of the drug?

a. Measuring heart rate and rhythm

b. Determining level of consciousness

c. Asking about an allergy to “sulfa” drugs

d. Checking intake and output for the last 24 hours

ANS: A

Adults over 65 years are usually prescribed a lower initial dose of thyroid hormone agonists
because they are more likely to have serious adverse cardiac and nervous system effects. For this
reason, doses are increased more slowly in older adults than in younger adults until an appropriate
maintenance dose is reached.

DIF: Cognitive Level: Applying REF: p. 316


Authentic Nurse Guru: For more of these materials :masterclasstutor1@gmail.com
Visovsky: Introduction to Clinical Pharmacology, 10th Edition 19

5. How do thyroid replacement drugs work to treat hypothyroidism?

a. Forcing the thyroid gland to secrete more thyroid hormones

b. Inhibiting the enzymes that break down thyroid hormones

c. Preventing damage to thyroid endocrine cells

d. Providing thyroid hormones

ANS: D

Hypothyroidism is low thyroid functioning, which is demonstrated by very low production of


thyroid hormones. Thyroid replacement drugs are thyroid hormone agonists. Taking these drugs
provides an exogenous source of thyroid hormones.

DIF: Cognitive Level: Remembering REF: p. 313

6. A patient who has been prescribed propylthiouracil (Propacil) asks you how this drug works to
reduce thyroid hormone levels. What is your best response?

a. “This drug combines with the enzyme that connects iodine to tyrosine.”

b. “This drug works by activating thyroid hormone in the pituitary gland.”

c. “This drug activates the gene for metabolism, speeding up cellular work.”

d. “This drug works by interfering with the thyroid hormone feedback loop.”

ANS: A

Antithyroid drugs work by entering the thyroid gland and combining with the enzyme responsible
for connecting iodine (iodide) with tyrosine. Without this iodide–tyrosine connection, thyroid
hormoneNurse
Authentic production is suppressed.
Guru: For more of these materials :masterclasstutor1@gmail.com
Visovsky: Introduction to Clinical Pharmacology, 10th Edition 19

DIF: Cognitive Level: Applying REF: p. 316

7. You are caring for a patient who is taking an antithyroid drug for the treatment of
hyperthyroidism. Which assessment should be performed before giving this drug?

a. Check the pulse rate for irregular rate and rhythm.

b. Check the blood pressure for hypertension.

c. Check the skin and sclera for yellowing.

d. Check the lower extremities for edema.

ANS: C

These drugs are hepatotoxic. Check the patient’s liver function tests before giving these drugs.
Both thyroid-suppressing drugs are hepatotoxic. Check the patient daily for yellowing of the skin
or sclera for jaundice.

DIF: Cognitive Level: Applying REF: p. 317

8. A patient taking an antithyroid drug for one week calls you and states that the symptoms of
hyperthyroidism have not improved yet. What is your best response?

a. “You will need to have your blood level of the drug checked.”

b. “This drug must be taken for 3 to 4 weeks to start being effective.”

c. “This may mean you have formed thyroid antibodies against this drug.”

d. “I will let your healthcare provider know so your dose can be adjusted.”

ANS: B

Teach patients that thyroid-suppressing drugs must be taken for 3 to 4 weeks to start being
effective because they have no effect on thyroid hormones already stored in the thyroid gland.

DIF: Cognitive Level: Applying REF: p. 316

9. You are caring for an elderly patient taking an antithyroid drug. Which of the following
conditions is associated with an adverse effect in this patient?
Authentic Nurse Guru: For more of these materials :masterclasstutor1@gmail.com
Visovsky: Introduction to Clinical Pharmacology, 10th Edition 20

a. Decreased complete blood count

b. Decreased blood pressure

c. Increased urine output

d. Increased platelet count

ANS: A

Older adults taking antithyroid drugs are more likely to have more severe adverse effects. The
older patient’s immune system is already lower than that of a younger person, which increases the
older person’s risk for infection. Bone marrow suppression from these drugs increases the risk for
severe infection.

DIF: Cognitive Level: Remembering REF: p. 317

10. A patient you are caring for develops a goiter. What does the appearance of a goiter mean?

a. Hypothyroidism

b. Hyperthyroidism

c. Thyroid problem

d. Premature menopause

ANS: C

Although a goiter is a distinct swelling of the thyroid gland and the neck, it only indicates a
thyroid problem. It is associated with some forms of hypothyroidism and some forms of
hyperthyroidism.

DIF: Cognitive Level: Remembering REF: p. 316

11. A patient with adrenal hypofunction has a known aldosterone deficiency and has been
prescribed Fludrocortisone (Florinef). The patient asks you how this drug works to treat this
problem. What is your best response?

a. “This drug acts like natural aldosterone in the body.”

Authentic Nurse Guru: For more of these materials :masterclasstutor1@gmail.com


Visovsky: Introduction to Clinical Pharmacology, 10th Edition 20

b. “This drug permits potassium to be retained in the body.”

c. “This drug acts to retain glucose for action within the body.”

d. “This drug permits the excretion of sodium from the body.”

ANS: A

Fludrocortisone (Florinef) is a synthetic drug that acts like natural aldosterone. With the use of
this drug, more sodium is retained to prevent excessive sodium wasting, and more potassium is
excreted to prevent dangerously high blood potassium levels.

DIF: Cognitive Level: Applying REF: p. 318

12. A patient prescribed Fludrocortisone (Florinef) calls to report a weight gain of 3 lb. in the last
week to you. What is your best response to this patient?

a. “You will need to cut all salt from your diet for a few days to see if this resolves.”

b. “Your diet may contain too many carbohydrates for your body to process.”

c. “I will notify your healthcare provider right away and get back to you.”

d. “This is an expected side effect of this drug.”

ANS: C

Congestive heart failure (CHF) is a serious adverse effect of fludrocortisone. It requires that the
drug dose be either reduced or stopped. A weight gain of 2 lb. in a day or 3 lb. in a week should
be reported to the healthcare provider immediately.

DIF: Cognitive Level: Applying REF: p. 318

13. Which side effects are most common when taking any drug to manage adrenal gland
hyperfunction?

a. Decreased urine output and increased sweating

b. Blurred vision and dry mouth

c. Headache and mouth sores


Authentic Nurse Guru: For more of these materials :masterclasstutor1@gmail.com
Visovsky: Introduction to Clinical Pharmacology, 10th Edition 20

d. Nausea and vomiting

ANS: D

The most common side effects for any drug used to manage adrenal gland function are nausea and
vomiting. They also cause many other gastrointestinal upsets. They are not associated with
blurred vision, dry mouth, headache, mouth sores, or increased sweating. Urine output is usually
increased, not decreased.

DIF: Cognitive Level: Remembering REF: p. 319

14. Which drug is used only for patients with type 2 diabetes and hypercortisolism?

a. Mitotane (Lysodren)

b. Mifepristone (Korlym)

c. Liothyronine sodium (Cytomel)

d. Methimazole (Northyx, Tapazole)

ANS: B

A specialized drug for hypercortisolism is Mifepristone (Korlym). This drug works by blocking
corticosteroid receptors. Although this does not reduce cortisol levels, it does inhibit cortisol
responses in different tissues. It is approved for use only in people who have type 2 diabetes and
hypercortisolism.

DIF: Cognitive Level: Remembering REF: p. 319

15. A patient experiencing symptoms related to menopause asks you to explain the cause of her
symptoms. What is your best response?

a. “Symptoms of menopause are caused by low levels of estrogen and high levels of

FSH.”

b. “Symptoms of menopause are caused by high levels of estrogen and low levels of

FSH.”

Authentic Nurse Guru: For more of these materials :masterclasstutor1@gmail.com


Visovsky: Introduction to Clinical Pharmacology, 10th Edition 20

c. “Symptoms of menopause are caused by low levels of estrogen and low levels of

FSH.”

d. “Symptoms of menopause are caused by high levels of estrogen and high levels of

FSH.”

ANS: A

Symptoms of menopause are caused by low levels of estrogen and high levels of FSH.

DIF: Cognitive Level: Applying REF: p. 320

16. What is one of the adverse effects of hormone replacement therapy for menopause?

a. Pregnancy

b. Blood clots

c. Miscarriage

d. Decreased mental concentration

ANS: B

Hormone replacement therapy for perimenopausal symptoms uses different types of estrogens and
sometimes progesterone. Exogenous use of these drugs increases the risk for forming blood clots
where they are not needed, leading to the potential venous thromboembolism complications of
deep vein thrombosis, strokes, and heart attacks.

DIF: Cognitive Level: Remembering REF: p. 322

17. Which side effects are most common among women taking hormone replacement therapy for
menopause?

a. Nausea, vomiting, and diarrhea

b. Skin itching and dark-colored urine

Authentic Nurse Guru: For more of these materials :masterclasstutor1@gmail.com


Visovsky: Introduction to Clinical Pharmacology, 10th Edition 20

c. Breast tenderness and fluid retention

d. Increased vaginal dryness and constipation

ANS: C

The most common side effects of hormone replacement therapy for perimenopausal symptoms are
breast tenderness, breakthrough bleeding, fluid retention, weight gain, and acne. Vaginal dryness
is reduced, not increased. Skin itching is reduced, not increased. Dark urine would be a symptom
of the adverse effect of liver toxicity. These drugs are not commonly a cause of any

gastrointestinal problems.

DIF: Cognitive Level: Remembering REF: p. 322

18. Which type of oral contraceptive drug must be taken every day of the month to be effective?

a. An estrogen–progestin combination

b. A progestin–drospirenone combination

c. An estrogen only “mini” pill

d. A progestin only “mini” pill

ANS: D

Progestin only mini pills contain very low concentrations of only one hormone. As the hormone
levels are low, they must be taken daily, without any time off, to be effective.

DIF: Cognitive Level: Remembering REF: p. 323

19. Which of the following patients is most at risk for adverse effects related to the use of oral
contraceptives?

a. A 37-year-old woman who has a history of smoking

b. A 25-year-old woman who had a baby 2 months ago

c. A 32-year-old woman who has never been pregnant

d. A 16-year-old
Authentic Nurse female
Guru:who
Forhas just become
more sexually
of these active :masterclasstutor1@gmail.com
materials
Visovsky: Introduction to Clinical Pharmacology, 10th Edition 20

ANS: A

Oral contraceptives increase the risk for blood clot formation. This problem can lead to deep vein
thrombosis, pulmonarybembolism, myocardial infection, and stroke. The risk increases among
women who smoke and in those older than 35 years.

DIF: Cognitive Level: Understanding REF: p. 324

20. Which oral contraceptive can increase serum potassium levels and potentially lead to irregular
heart rhythm?

a. Yasmin

b. Camila

c. Ovral

d. Zovia

ANS: A

Oral contraceptives that use drospirenone as the progestin (Ocella, Yasmin, YAZ28) can increase
the serum potassium level, which can lead to heart block and other irregular heart rhythms.
Women who have kidney, liver, or adrenal disease and those who are taking other drugs that
increase potassium levels (e.g., angiotensin-converting enzyme (ACE) inhibitors for hypertension,
potassium-sparing diuretics) are not recommended to use contraceptives containing drospirenone.

DIF: Cognitive Level: Understanding REF: p. 324

21. You are teaching a young female patient about the oral contraceptive she has been prescribed.
What instructions should be included in the teaching plan for this patient?

a. “Take oral contraceptives exactly as prescribed.”

b. “You may take over-the-counter drugs with oral contraceptives.”

c. “You may miss up to two doses in a row and still be protected from pregnancy.”

d. “You may use nicotine patches, but stop smoking when taking oral contraceptives.”

ANS: A
Authentic Nurse Guru: For more of these materials :masterclasstutor1@gmail.com
Visovsky: Introduction to Clinical Pharmacology, 10th Edition 20

Oral contraceptives are only effective at preventing pregnancy when taken exactly as prescribed.
Patients should not smoke or use nicotine in any form to reduce the risk for blood clots, heart
attacks, and strokes. If more than one dose within a cycle is missed, especially two doses in a row,
continue not only to use it for the rest of the cycle but also to use another method of contraception
for the rest of the cycle. Instruct the patient not to take any over-the-counter drug without
checking with your healthcare provider who prescribed the contraceptive to prevent possible
interactions.

DIF: Cognitive Level: Applying REF: p. 324

22. A male patient is prescribed androgen therapy with testosterone 100 mg IM every 4 weeks.
What finding will you teach the patient to report to the prescriber immediately?

a. Blood pressure decrease of 10 mm Hg systolic

b. Urine output increase of 200 mL/day

c. Fluid retention

d. Increased prostate size

ANS: C

Teach patients to check weight every day to assess for fluid retention and report weight gain of 5
or more pounds (indicating significant fluid retention) to the prescriber.

DIF: Cognitive Level: Applying REF: p. 325

23. Which bone or joint is most likely to be adversely affected by bisphosphonates?

a. Eye orbit

b. Jawbone

c. Big toe
Authentic Nurse Guru: For more of these materials :masterclasstutor1@gmail.com
Visovsky: Introduction to Clinical Pharmacology, 10th Edition 20

d. Wrist

ANS: B

The most serious adverse effect of the bisphosphonates is the development of jawbone necrosis
(osteonecrosis), especially with tooth extraction or other invasive dental procedures involving the
jawbone in which the bone is damaged. The exact mechanism by which these drugs cause
osteonecrosis is not known but is thought to occur because the drugs interfere with bone healing.

DIF: Cognitive Level: Remembering REF: p. 309

24. A patient is to receive a subcutaneous injection of denosumab (Prolia) for severe osteoporosis.
What action should you take before giving this injection?

a. Place emergency equipment in the room with the patient.

b. Assess for circulation in the umbilical area.

c. Monitor the patient’s liver function tests.

d. Assess the patient’s ability to walk.

ANS: A

Assess the patient for an allergic reaction during and after subcutaneous injection of denosumab
(Prolia). Keep emergency equipment in the room with the patient.

DIF: Cognitive Level: Applying REF: p. 311

MULTIPLE RESPONSE

1. Which findings indicate that thyroid hormone drug dose is appropriate and effective in a patient
with hypothyroidism? (Select all that apply.)

a. The patient’s vital signs are within normal limits.

b. The patient reports an increase in activity levels.

c. The patient’s body weight remains consistent.

d. The patient reports mild, bilateral edema.


Authentic Nurse Guru: For more of these materials :masterclasstutor1@gmail.com
Visovsky: Introduction to Clinical Pharmacology, 10th Edition 20

e. The patient has a bowel movement daily.

f. The patient’s goiter is reduced in size.

ANS: A, C, E

These changes indicate that the drug is effective and the dose is appropriate: The patient’s vital
signs (body temperature, heart rate, blood pressure, and respiratory rate) are within normal limits.
Patient’s activity level and mental status are normal for him or her. Patient’s body weight is
consistent with the amount of calories he or she eats and his or her activity level. Patient’s bowel
habits are what they were before the thyroid problem occurred.

DIF: Cognitive Level: Applying REF: p. 315

2. A patient is diagnosed with adrenal gland hyperfunction. The patient asks you to explain how
the adrenal glands work. Which of the following statements are true? (Select all that apply.)

a. “Adrenal hormones differ by age and gender.”

b. “Aldosterone controls sodium and water balance.”

c. “Glucocorticoid hormones are necessary for life.”

d. “The most common cause of adrenal gland hyperfunction is an adrenal gland tumor.”

e. “Low levels of adrenal hormones can lead to stroke.”

f. “The adrenal cortex secretes aldosterone and cortisol.”

g. “When adrenal hormones are overproduced, cancer can result.”

ANS: B, C, D, F

The adrenal cortex secretes aldosterone and cortisol. Aldosterone controls sodium and water
balance. Cortisol, of which there are many types, has many more functions that are essential for
life. Cortisol helps maintain critical blood glucose levels, the stress response, excitability of
cardiac muscle, immunity, and blood sodium levels. Since cortisol was first discovered to affect
blood glucose levels, it is also known as a glucocorticoid.

DIF: Cognitive Level: Remembering REF: p. 318


Authentic Nurse Guru: For more of these materials :masterclasstutor1@gmail.com
Visovsky: Introduction to Clinical Pharmacology, 10th Edition 20

3. You are teaching a patient with adrenal gland hyperfunction taking Mitotane (Lysodren) about
this drug. What signs and symptoms of adrenal insufficiency should you teach this patient to
report? (Select all that apply.)

a. Craving for salty food

b. Hyperglycemia

c. Feeling weak

d. Hypertension

e. Fatigue

f. Nausea

ANS: A, C, E

Teach patients the signs and symptoms of adrenal insufficiency: hypoglycemia, salt craving,
muscle weakness, hypotension, and fatigue.

DIF: Cognitive Level: Applying REF: p. 319

4. Which drugs belong to the bisphosphonate class of drug? (Select all that apply.)

a. Alendronate (Fosamax)

b. Denosumab (Prolia)

c. Estrogen/bazedoxifene (Duavee)

d. Ibandronate (Boniva)

e. Raloxifene (Evista)

f. Risedronate (Actonel)

g. Zoledronic acid (Reclast)

ANS: A, D, F, G

Alendronate, ibandronate, risedronate, and zoledronic acid all belong to the bisphosphonate class
Authentic Nurse Guru: For more of these materials :masterclasstutor1@gmail.com
Visovsky: Introduction to Clinical Pharmacology, 10th Edition 21

of drug. Denosumab is a monoclonal antibody. Estrogen/bazedoxifene and raloxifene are from the
estrogen agonist/antagonist class of drugs.

DIF: Cognitive Level: Remembering REF: p. 310

5. A 25-year-old female patient is diagnosed with hypothyroidism and has been prescribed
treatment with a thyroid hormone agonist. The patient asks if there is anything important that she
needs to know about this drug. List the correct responses. (Select all that apply.)

a. “You will need to take this drug for the rest of your life.”

b. “If you decide to become pregnant, during pregnancy you may need a higher dose.”

c. “Thyroid hormone agonist brands can be used interchangeably.”

d. “The drug should be taken 2 to 3 hours before or at least 3 hours after a meal.”

e. “You only need to take this drug until your symptoms improve.”

f. “Thyroid hormone agonists have few side effects because they mimic normal hormones.”

ANS: A, B, D, F

Hypothyroidism is a common problem in which the thyroid gland produces little or no thyroid
hormones, slowing all aspects of metabolism, and is treated with thyroid hormone agonists.
Thyroid hormone agonists have few side effects because they mimic normal hormones. Usually
this therapy is needed for the rest of the person’s life. Thyroid hormone agonist brands cannot be
used interchangeably because the strength of different brands and types varies and so can patient
responses. Thyroid hormone agonists should be taken 2 to 3 hours before meals or taking a fiber
supplement or at least 3 hours after meals or after taking a supplement because food and fiber
greatly decrease absorption of these drugs. During pregnancy, women may need a higher dose,
and women who are taking these drugs are advised not to breast-feed because the drug can be
found in breast milk.

DIF: Cognitive Level: Applying REF: p. 314

Authentic Nurse Guru: For more of these materials :masterclasstutor1@gmail.com


Visovsky: Introduction to Clinical Pharmacology, 10th Edition 21

Chapter 18: Drug Therapy for Diabetes Visovsky: Introduction to Clinical


Pharmacology, 10th Edition

MULTIPLE CHOICE

1. You are teaching a patient with newly diagnosed diabetes about insulin. The patient asks you to
explain the role of insulin in preventing hyperglycemia. What is your best response?

a. “Insulin plays a major role in converting complex carbohydrates to glucose.”

b. “Insulin allows cells to take up, use, and store carbohydrates, fat, and protein.”

c. “Insulin prevents the kidney from excreting of glucose.”

d. “Insulin prevents the formation of fat cells.”

ANS: B

Insulin is called the hormone of plenty, because its release is triggered by hyperglycemia. Insulin
allows cells to take up, use, and store carbohydrates, fat, and protein many body cells have
membranes that do not allow glucose to move into the cells to participate in metabolism. When
insulin binds to membrane insulin receptors, the membranes become more open to glucose,
allowing it to enter.

DIF: Cognitive Level: Applying REF: p. 328

2. In what way does insulin work to lower blood glucose levels?

a. Enhancing the enzymes that break down glucose

b. Helping glucose to move from the blood into cells

c. Converting glucose into proteins in the liver and brain

d. Converting glycogen into glucose in the liver and brain

ANS: B

When insulin binds to insulin receptors on cells, the cells’ membranes become more open
(permeable) to glucose and glucose transport proteins in the cell membranes become more active.
Authentic Nurse Guru: For more of these materials :masterclasstutor1@gmail.com
Visovsky: Introduction to Clinical Pharmacology, 10th Edition 21

The overall result is movement of glucose into the cells, which lowers blood glucose levels.

DIF: Cognitive Level: Remembering REF: p. 328

3. Which term is used to indicate an elevated blood sugar (glucose) level in a patient?

a. Glycogen

b. Euglycemia

c. Hypoglycemia

d. Hyperglycemia

ANS: D

Glycogen is a stored form of human carbohydrate. Euglycemia means a blood sugar level within
the normal range. Hypoglycemia is a lower than normal blood sugar level.

DIF: Cognitive Level: Remembering REF: p. 328

4. A nursing student working with you today is learning about blood glucose control. Which
statement made by the student about glucagon demonstrates an understanding of this hormone?

a. “Glycogen triggers the release of glucose from the liver and skeletal muscle.”

b. “Glucagon allows the cells to use fats, protein, and carbohydrates.”

c. “Glucagon is an important hormone for energy production.”

d. “Glucagon initiates the release of adenosine triphosphate.”

ANS: A

Glucagon, which is known as the hormone of starvation, is secreted from the alpha cells of the
pancreas when blood glucose levels are lower than normal. Glucagon goes to the liver and
triggers the breakdown of glycogen and release of glucose into the blood, raising the blood
glucose level. These actions are exactly the opposite of insulin’s actions.

DIF: Cognitive Level: Applying REF: p. 328

5. What is
Authentic the main
Nurse reason
Guru: thatmore
For insulinof
is these
needed as drug therapy
materials for people who have type 1
:masterclasstutor1@gmail.com
Visovsky: Introduction to Clinical Pharmacology, 10th Edition 21

diabetes?

a. The beta cells of the pancreas no longer make insulin.

b. The alpha cells of the pancreas no longer make insulin.

c. The beta cells of the pancreas make too much glucagon.

d. The alpha cells of the pancreas make too much glucagon.

ANS: A

With type 1 diabetes, the beta cells of the pancreas are destroyed and no longer produce any
insulin. Insulin is necessary for life, so drug therapy for type 1 diabetes requires insulin. People
who have type 2 diabetes often continue to secrete insulin from their beta cells; thus extra insulin
may not be needed. Drugs for this condition often work by forcing the beta cells to release the
patient’s own insulin.

DIF: Cognitive Level: Understanding REF: p. 329

6. Which statement regarding diabetes mellitus (DM) type 2 is true?

a. Some people with DM type 2 will need insulin.

b. This type of diabetes is found in children less than 1 year of age.

c. Drug therapy for DM type 2 eliminates the need for diet and exercise.

d. DM type 2 is characterized by sudden, complete shutdown of the pancreas.

ANS: A

Insulin may also be necessary for some people with DM type 2, although diet, weight reduction,
and non-insulin antidiabetic drugs are often effective in maintaining good blood glucose control.

DIF: Cognitive
Authentic NurseLevel: Remembering
Guru: For more REF: p. 329materials :masterclasstutor1@gmail.com
of these
Visovsky: Introduction to Clinical Pharmacology, 10th Edition 21

7. You are preparing to discuss a newly prescribed insulin stimulator with a patient. What
condition would need to be present for this drug to work effectively for a diabetic patient?

a. The patient would need to be able to self-inject the drug.

b. The patient would need to be in ketoacidosis to begin this drug.

c. The patient would need to have diabetes mellitus type 2 to use this drug.

d. The patient needs to have some functioning beta cells for this drug to work.

ANS: C

Insulin stimulators are oral drugs that lower blood glucose levels by stimulating the release of
insulin stored in the beta cells of the pancreas. Therefore, the patient must have some functioning
beta cells if these drugs are to work.

DIF: Cognitive Level: Remembering REF: p. 330

8. A patient with diabetes mellitus type 2 is prescribed glipizide (Glucotrol). You are preparing to
give the glipizide to the patient. Which of the following nursing actions would be your best action
when giving this drug?

a. Give this drug with milk.

b. Give this drug with a meal.

c. Assess the patient for signs of pancreatitis before giving this drug.

d. Take the patient’s pulse before giving this drug.

ANS: B

These drugs with, or just before, meals prevent hypoglycemia. If you must skip a meal, also skip
the drug dose.

DIF: Cognitive Level: Applying REF: p. 331

9. Which oral antidiabetic drug belongs to the biguanide class?

a. Acarbose (Precose)
Authentic Nurse Guru: For more of these materials :masterclasstutor1@gmail.com
Visovsky: Introduction to Clinical Pharmacology, 10th Edition 21

b. Nateglinide (Starlix)

c. Pioglitazone (Actos)

d. Metformin (Glucophage)

ANS: D

There is only one drug in the biguanide class, and it is metformin.

DIF: Cognitive Level: Remembering REF: p. 331

10. For which patient would a biguanide antidiabetic drug be contraindicated?

a. A female patient who is pregnant

b. A newly diagnosed type 2 diabetic

c. A patient with known kidney disease

d. A diabetic patient with some functioning pancreatic beta cells

ANS: C

Metformin should not be given to patients with kidney disease as it can cause kidney failure.

DIF: Cognitive Level: Understanding REF: p. 331

11. A diabetic patient taking metformin (Glucophage) is scheduled for a radiographic scan with
contrast dye. What instructions should you provide to this patient?

a. “Take the metformin as soon as the scan is completed.”

b. “Stop taking the metformin at least 24 hours before your scan.”

c. “Take your metformin the morning of the scan.”

d. “Stop your metformin the night before the scan, and resume 72 hours after the scan.”

ANS: B

Metformin is to be stopped at least 24 hours before radioactive dye is used and not started again
Authentic Nurse
until 48 hours Guru:
after For
the test more of these materials :masterclasstutor1@gmail.com
is completed.
Visovsky: Introduction to Clinical Pharmacology, 10th Edition 21

DIF: Cognitive Level: Applying REF: p. 331

12. You are teaching a diabetic patient who has been prescribed rosiglitazone about this drug.
Which statement should you include in the patient’s teaching plan?

a. “Call if you develop swelling of the legs, or weight gain.”

b. “If you miss a dose of this drug, take two tablets for your next dose.”

c. “You will not need to check your blood sugar when using this drug.”

d. “Vision changes are expected with this drug and are not of a concern.”

ANS: A

The thiazolidinediones have been associated with severe cardiovascular side effects and must be
used with care. Rosiglitazone can also cause fluid retention, liver problems, and macular edema.
Teach patients to report swelling of the feet or ankles, or rapid weight gain to the healthcare
provider.

DIF: Cognitive Level: Applying REF: p. 333

13. A 48-year-old male has been diagnosed with type 2 diabetes. He has been prescribed a
thiazolidinedione drug. Which precaution regarding alcohol use is important to teach this patient?

a. “Increase your water intake whenever you drink alcohol.”

b. “Avoid alcohol while taking this drug.”

c. “Only drink alcohol with a meal or shortly after a meal.”

d. “Alcohol should be limited to 4 ounces daily.”

ANS: B

Drinking alcoholic beverages while taking thiazolidinedione can result in hyperglycemia or


hypoglycemia depending on how much alcohol is ingested.
Authentic Nurse Guru: For more of these materials :masterclasstutor1@gmail.com
Visovsky: Introduction to Clinical Pharmacology, 10th Edition 21

DIF: Cognitive Level: Applying REF: p. 334

14. Which statement made by a diabetic patient who has been prescribed acarbose indicated an
understanding as to how acarbose

(Precose) prevents blood glucose levels from rising too high?

a. “Acarbose limits the enzyme that converts complex carbohydrates into glucose.”

b. “Acarbose blocks the absorption of carbohydrates and glucose in the intestines.”

c. “Acarbose enhances the binding of insulin to its membrane receptors.”

d. “Acarbose releases more insulin from pancreatic beta cells.”

ANS: A

Acarbose is an alpha-glucosidase inhibitor. Drugs from this class work by slowing the digestion
of dietary starches and other carbohydrates by inhibiting an enzyme that breaks them down into
glucose. The result of this action is that blood glucose does not rise as far or as fast after a meal.

DIF: Cognitive Level: Applying REF: p. 334

15. You are about to give miglitol (Glyset) to a patient with diabetes mellitus type 2. Which
nursing action is most important for this patient?

a. Rotate the injection site on a weekly basis.

b. Give this drug at the beginning of a meal.

c. Do not mix this drug in the same syringe as insulin.

d. Assess the patient for signs and symptoms of heart failure.

ANS: B

Miglitol is an oral antidiabetic drug from the alpha-glucosidase inhibitor class. Its action is to
slow the digestion of starches and other carbohydrates in the intestinal tract. In order to be
effective, it must be taken at the beginning of a meal.

DIF: Cognitive Level: Applying REF: p. 333


Authentic Nurse Guru: For more of these materials :masterclasstutor1@gmail.com
Visovsky: Introduction to Clinical Pharmacology, 10th Edition 21

16. In teaching a diabetic patient about DPP_$ inhibitors, which symptom should you tell the
patient to report as a potential serious complication from this drug?

a. Swelling of the lower extremities

b. Changes in visual acuity

c. Diarrhea

d. Abdominal pain

ANS: D

Diarrhea can be an expected side effect of this drug. However, pancreatitis is a serious
complication that may occur. Symptoms of acute pancreatitis that should be taught to be reported
immediately are upper abdominal radiating to the back, nausea and vomiting, fever, and rapid
pulse. This is a serious adverse effect of the drug.

DIF: Cognitive Level: Understanding REF: p. 337

17. A young, female patient with diabetes mellitus who is taking an oral antidiabetic drug comes
in for a regularly scheduled clinic appointment. She tells you that is currently taking an oral
contraceptive agent for birth control. What is your best response?

a. “Oral contraceptives are contraindicated for diabetics.”

b. “Oral contraceptives can be taken only if you are on insulin.”

c. “Oral contraceptives can be safely taken with oral antidiabetic agents.”

d. “Oral contraceptives reduce their effectiveness of oral antidiabetic agents.”

ANS: D

Oral contraceptives and antibiotics can interact with drugs for diabetes, reducing their
effectiveness.

DIF: Cognitive Level: Applying REF: p. 338

18. You are teaching a patient who has been prescribed a sodium-glucose cotransport inhibitor for
the treatment of diabetes mellitus type 2. The patient asks you to explain how this drug works.
Authentic Nurse Guru: For more of these materials :masterclasstutor1@gmail.com
Visovsky: Introduction to Clinical Pharmacology, 10th Edition 21

What is your best response?

a. “This drug works by inactivating normal gut hormones.”

b. “This drug works by preventing the kidney from reabsorbing glucose.”

c. “This drug works by making insulin receptors more sensitive to insulin.”

d. “This drug works by preventing enzymes from breaking down starches into glucose.”

ANS: B

Sodium-glucose cotransport inhibitors are a new category of non-insulin antidiabetic drug that
lower blood glucose levels by preventing the kidney from reabsorbing glucose that was filtered
from the blood into the urine. This glucose then remains in the urine and is excreted rather than
moved back into the blood.

DIF: Cognitive Level: Applying REF: p. 338

19. A patient who is taking dapagliflozin (Farxiga) for diabetes mellitus type 2 has also been
prescribed furosemide (Lasix) for another condition. What potential risk should you monitor for
on this patient?

a. Risk for infection

b. Risk for bleeding

c. Risk of dehydration

d. Risk for fluid retention

ANS: C

Combining sodium-glucose cotransport inhibitors with diuretics increases the frequency of


urination increases and can result in dehydration.

DIF: Cognitive Level: Understanding REF: p. 339

Authentic Nurse Guru: For more of these materials :masterclasstutor1@gmail.com


Visovsky: Introduction to Clinical Pharmacology, 10th Edition 22

20. A patient is taking a sodium-glucose cotransport inhibitor for the treatment of diabetes. Which
of the following laboratory values should you monitor in this patient?

a. Serum potassium

b. Serum amylase and lipase

c. Hemoglobin and hematocrit

d. Cholesterol and triglycerides

ANS: A

Patients with diabetes are at increased risk for hyperkalemia due to the presence of renal
impairment, and potential use of diuretics and antihypertensives used in this population.
Hyperkalemia is also potential serious adverse effect of sodium-glucose cotransport inhibitors.
Monitor the patient’s serum potassium level, and check for symptoms of hyperkalemia (muscle
twitching, numbness and tingling, irregular heart rate).

DIF: Cognitive Level: Remembering REF: p. 339

21. A patient is started on exenatide (Byetta), an injectable incretin mimetic. The patient asks you
how this drug will help lower blood glucose. Which is your best response?

a. “This drug restores insulin secretion.”

b. “This drug increases insulin secretion.”

c. “This drug works to increase the duration of insulin.”

d. “This drug works by increasing the action of glucagon.”

ANS: B

Incretin mimetics act like the natural gut hormones (e.g., GLP-1) secreted in response to food in
the stomach, increasing insulin secretion, decreasing glucagon secretion, and slowing the rate of
gastric emptying. This results in an increase in insulin secretion, a decrease in glucagon secretion,
and a slower the rate of gastric emptying.
Authentic Nurse Guru: For more of these materials :masterclasstutor1@gmail.com
Visovsky: Introduction to Clinical Pharmacology, 10th Edition 22

DIF: Cognitive Level: Applying REF: p. 334

22. A patient who is taking insulin each morning develops symptoms of nervousness, hunger,
weakness, and confusion. Which adverse effect of insulin is this patient experiencing?

a. Hyperglycemia

b. Hypoglycemia

c. Lactic acidosis

d. Diabetic ketoacidosis

ANS: B

Symptoms of hypoglycemia include nervousness; hunger; weakness; cold, clammy skin; and
change in the level of consciousness.

DIF: Cognitive Level: Applying REF: p. 341

23. What characteristic of insulin makes insulin a “high-alert” drug?

a. Insulin has a high potential for hypersensitivity reactions.

b. Serious harm can occur if the wrong dose is given.

c. Too little insulin can cause the respiratory system to become depressed.

d. Too much insulin can cause blood glucose levels to increase to dangerously high levels.

ANS: B

Insulin as a drug works very quickly. Great harm can come to a patient who receives it and does
not have diabetes. Also, if too much of it is given to a patient who does have diabetes, the patient
can become severely hypoglycemic and die. If too little insulin is given to a patient who has
diabetes, blood glucose levels remain too high and cause organ damage.

DIF: Cognitive Level: Understanding REF: p. 341

24. You Nurse


Authentic are preparing
Guru: to teach a patient
For more ofabout
thesethematerials
use of short-acting insulin. Which instruction
:masterclasstutor1@gmail.com
Visovsky: Introduction to Clinical Pharmacology, 10th Edition 22

should be stressed to the patient regarding this drug?

a. “Place pressure on the injection site for 5 full minutes.”

b. “You should eat a meal within 15 minutes of giving yourself the injection.”

c. “You should always aspirate before injecting this drug.”

d. “You may store your insulin for 6 months at room temperature.”

ANS: B

Whenever short-acting insulin is given before a meal, the patient will need to eat the meal within
15 minutes of receiving the injection to prevent hypoglycemia.

DIF: Cognitive Level: Applying REF: p. 344

25. A patient about to begin self-injections of insulin at home asks you why insulin can only be
given by injection, and not as an oral drug. What is your best response?

a. “Injected insulin works faster than oral drugs to lower blood glucose levels.”

b. “Insulin is a small protein that is destroyed by stomach acids and intestinal enzymes.”

c. “Insulin is a “high-alert drug” and can be abused if it were available as an oral agent.”

d. “Oral insulin has a high ‘first-pass loss’ rate in the liver that would require high dosages to be
effective.”

ANS: B

Since insulin is a small protein that is easily destroyed by stomach acids and intestinal enzymes, it
cannot be used as an oral drug. Most commonly it is injected subcutaneously.

DIF: Cognitive Level: Applying REF: p. 340

MULTIPLE RESPONSE

1. A 52-year-old male patient with diabetes has just arrived at the clinic for the first time to see his
new healthcare provider. The provider informs the patient that his diabetes has been poorly
controlled over the past 3 months. You are educating this patient about the conditions and
Authentic Nurse Guru: For more of these materials :masterclasstutor1@gmail.com
Visovsky: Introduction to Clinical Pharmacology, 10th Edition 22

complications associated with poorly controlled diabetes. List the conditions and/or complications
that are associated with poorly controlled diabetes. (Select all that apply.)

a. Stroke

b. Blindness

c. Kidney failure

d. Nonhealing wounds

e. Bone destruction

f. Cardiovascular disease

g. Insomnia

ANS: A, B, C, D, F

DM that is not well controlled can reduce the function of all organs and tissues. Complications of
uncontrolled or poorly controlled DM include hypertension, high blood lipid levels, early-onset
cardiovascular disease, kidney failure, strokes, and blindness to name only the more serious ones.
Wounds and ulcers can develop that fail to heal and may result in gangrene or amputation of a
limb. The complications of DM can be delayed or reduced with good blood glucose (glycemic)
control along with keeping blood pressure and blood cholesterol levels as close to normal as
possible.

DIF: Cognitive Level: Understanding REF: p. 329

2. You are taking the history of a new clinic patient whom you suspect of having undiagnosed
diabetes. Which of the following symptoms would you expect this patient to report? (Select all
that apply.)

a. Increased urination

b. Petechiae

c. Increased hunger

d. Decreased blood pressure


Authentic Nurse Guru: For more of these materials :masterclasstutor1@gmail.com
Visovsky: Introduction to Clinical Pharmacology, 10th Edition 22

e. Rhinitis

f. Increased thirst

ANS: A, C, F

The classic symptoms of DM are polydipsia (increased fluid intake), polyuria (excessive
urination), and polyphagia (hunger with excessive eating).

DIF: Cognitive Level: Remembering REF: p. 329

3. The family of a patient with diabetes who is taking an insulin stimulator calls you because the
patient may be experiencing hypoglycemia from the drug. Which symptoms would you expect the
patient to experience in a hypoglycemic episode? (Select all that apply.)

a. Bradycardia

b. Headache

c. Lack of appetite

d. Clammy skin

e. Confusion

f. Anxiety

g. Tremors

ANS: B, D, E, F, G

Signs of hypoglycemia include tremors, sweating, confusion, and rapid heart rate, hunger,
headache, nervousness, and inability to concentrate.

DIF: Cognitive Level: Remembering REF: p. 331

Authentic Nurse Guru: For more of these materials :masterclasstutor1@gmail.com


Visovsky: Introduction to Clinical Pharmacology, 10th Edition 22

Chapter 19: Drugs for Eye and Ear Problems Visovsky: Introduction to
Clinical Pharmacology, 10th Edition

MULTIPLE CHOICE

1. You are preparing to instill otic drops into the ear canal of an adult. What is the correct
technique for giving otic drugs to an adult?

a. Pull the external ear down and back.

b. Press the pinna against the mastoid bone.

c. Pull the external ear up and back.

d. Press the upper portion of the ear until the eustachian tube is seen.

ANS: C

When instilling ear drops into an adults’ ear, gently pull the external ear up and back.

DIF: Cognitive Level: Applying REF: p. 348

2. Which term refers to constriction of the pupil of the eye?

a. Miosis

b. Punctum

c. Glaucoma

d. Mydriasis

ANS: A

Miosis means constriction of the pupil; mydriasis means dilation of the pupil; glaucoma is an eye
disease that is caused by increased intraocular pressure; and punctum is the opening in the lower
lid that drains tears into systemic circulation.

DIF: Cognitive Level: Remembering REF: p. 349

3. Which drug group for glaucoma has the side effect of darkening the iris color?
Authentic Nurse Guru: For more of these materials :masterclasstutor1@gmail.com
Visovsky: Introduction to Clinical Pharmacology, 10th Edition 22

a. Adrenergic agonists

b. Beta blockers

c. Cholinergics

d. Prostaglandin agonists

ANS: D

Two side effects of the prostaglandins agonists are making the lashes grow and darkening the iris
color, sometimes changing a blue iris to one that appears browner. No other drugs for glaucoma
have these side effects.

DIF: Cognitive Level: Remembering REF: p. 355

4. A patient with glaucoma reports a possible infection in the affected eye and asks if the usual
dose of latanoprost (Xalatan) drops should be instilled. What is your best response?

a. “Do not take this dose until your healthcare provider is notified.”

b. “Rinse the affected eye with sterile saline and then instill the eye drops.”

c. “You should not put the eye drops in today but can resume them tomorrow.”

d. “Missing a dose of this drug will increase intraocular pressure and worsen your vision.”

ANS: A

Never instill prostaglandin agonists into an eye that has been scratched or has an infection.
Contact the healthcare provider for instructions about continuing glaucoma therapy.

DIF: Cognitive Level: Applying REF: p. 356

5. The LPN/LVN is preparing to give an eye drop to a patient for glaucoma. Which of the
following drugs for glaucoma is a beta blocker?

a. Timolol (Timoptic)

b. Travoprost (Travatan)

c. Apraclonidine
Authentic (Iopidine)
Nurse Guru: For more of these materials :masterclasstutor1@gmail.com
Visovsky: Introduction to Clinical Pharmacology, 10th Edition 22

d. Pilocarpine (Adsorbocarpine)

ANS: A

Timolol is a beta blocker, travoprost is a prostaglandins agonist, apraclonidine is an adrenergic


agonist; and pilocarpine is a cholinergic drug.

DIF: Cognitive Level: Remembering REF: p. 355

6. Which drug group for glaucoma can worsen underlying respiratory conditions?

a. Cholinergics

b. Beta blockers

c. Adrenergic agonists

d. Prostaglandin agonists

ANS: B

Beta blockers that are absorbed systemically cause constriction of bronchiolar smooth muscle and
can make asthma worse.

DIF: Cognitive Level: Remembering REF: p. 355

7. Which precaution is most important to teach a patient who is prescribed any type of eye drug
given as eye drops?

a. Apply only the number of drops prescribed.

b. Stop the drug immediately if eye redness occurs.

c. Wear dark glasses for 1 hours after placing the eye drops.

d. Apply pressure to the corner of the eye after the drug has been placed.

ANS: A

Not only can eye drops enter the circulatory system and have systemic effects, excessive drops for
some types of drugs for glaucoma can reduce intraocular pressure to dangerously low levels.
Many patients
Authentic Nurse doGuru:
not consider any topical
For more drug, including
of these materialseye:masterclasstutor1@gmail.com
drops, to be “real” drug and believe
Visovsky: Introduction to Clinical Pharmacology, 10th Edition 22

that more drug is better. Patients must be taught to use all eye drugs exactly as prescribed, and not
use more drug than is prescribed.

DIF: Cognitive Level: Applying REF: p. 351

8. A patient who has been prescribed timolol (Timoptic) for the last month reports that his asthma
is worse. What is your best first response?

a. Check the patient’s breathe sounds and pulse rate.

b. Hold the dose and notify the prescriber immediately.

c. Ask the patient what drugs he takes to control his asthma.

d. Reassure the patient that this is an expected response and requires no action.

ANS: A

Timolol is a nonspecific beta blocker that can have systemic effects. It can cause
bronchoconstriction and can make heart failure worse with backing of fluid into the lungs. The
first action is to determine whether the patient needs immediate attention for either
bronchoconstriction or possible heart failure. This is done by assessing breath sounds with a
stethoscope and checking the pulse for rate, strength, and regularity. If a pulse oximeter is
available, it should be used to assess the patient’s degree of oxygen saturation.

The results of this assessment along with the patient’s report are then provided to the prescriber. It
is likely that the class of drugs used to control this patient’s glaucoma will need to be changed.

DIF: Cognitive Level: Applying REF: p. 355

9. Which class of drugs for glaucoma therapy generally is used short term to prevent or reduce
increases in intraocular pressure (IOP) after eye surgery rather than for long-term IOP control?

a. Cholinergic drugs

b. Adrenergic agonists

c. Prostaglandins agonists

d. Beta-adrenergic blockers
Authentic Nurse Guru: For more of these materials :masterclasstutor1@gmail.com
Visovsky: Introduction to Clinical Pharmacology, 10th Edition 22

ANS: B

The adrenergic agonists generally are used short term for 1 to 4 weeks to prevent or reduce
increases in IOP. They are more likely to cause severe, systemic side effects if used long term.
The other three drug classes are used long term for IOP control.

DIF: Cognitive Level: Remembering REF: p. 358

10. You are about to give apraclonidine (Iopidine) eye drops to a patient with glaucoma. What
assessment should you perform before giving this drug?

a. Check the patient’s urinary output.

b. Check the patient’s pulse and blood pressure.

c. Check the patient’s pupils for responsiveness.

d. Check the patient’s level of orientation to time, place, and person.

ANS: B

Adverse effects of adrenergic agonists include bradycardia or tachycardia and decreased blood
pressure. These can result from giving too many drops of the drug causing systemic absorption.

DIF: Cognitive Level: Applying REF: p. 358

11. What safety instructions would be appropriate to give to a patient starting glaucoma treatment
with carbachol (Isopto Carbachol)?

a. Avoid driving at night due to decreased vision.

b. You may develop drowsiness, fatigue, or irritability.

c. Do not rub your eye even if it feels as though something is in it.

d. Check your blood sugar daily; these drops can cause hypoglycemia.

ANS: A

Patients may develop decreased night vision, so patients may need to avoid night driving due to
side effects of the cholinergic agonists.
Authentic Nurse Guru: For more of these materials :masterclasstutor1@gmail.com
Visovsky: Introduction to Clinical Pharmacology, 10th Edition 23

DIF: Cognitive Level: Applying REF: p. 358

12. Which drug reduces the production of aqueous humor to the greatest degree?

a. Acetazolamide (Diamox)

b. Pilocarpine (Isopto)

c. Dipivefrin (AK-Pro)

d. Travoprost (Travatan)

ANS: A

Acetazolamide is a carbonic anhydrase inhibitor that can reduce production of aqueous humor by
60% or more. Although drugs from some other classes can reduce the production of aqueous
humor, this effect is much less than that of acetazolamide.

DIF: Cognitive Level: Remembering REF: p. 359

13. You are taking the history from a patient who may be prescribed a carbonic anhydrase
inhibitor for the treatment of glaucoma. What information would indicate this category of drug
may not be appropriate for this patient?

a. The patient is a diabetic.

b. The patient has mydriasis.

c. A history of sulfa drug allergy.

d. The patient has unequal pupil size.

ANS: C

Never give a carbonic anhydrase inhibitor to a patient who has a “sulfa” allergy because these
drugs areNurse
Authentic a type of sulfonamide.
Guru: For more of these materials :masterclasstutor1@gmail.com
Visovsky: Introduction to Clinical Pharmacology, 10th Edition 23

DIF: Cognitive Level: Applying REF: p. 360

14. A parent presents to the clinic tells with her 4-year-old child and tells you that he has some
clear drainage coming from the right ear. This drainage is sometimes tinged with blood. The
parent asks if the ear canal should be irrigated. List the best response by the

LPN/LVN.

a. “Use a small amount of sterile saline instilled into the ear to flush out the fluid and blood.”

b. “The ear should not be irrigated when there is drainage present.”

c. “You may use over-the-counter ear drops in this ear to clear up this new infection.”

d. “You should use a cotton swab daily to clean out the ears.”

e. “The ear will need to be irrigated, and then an antibiotic for the ear instilled.”

f. “You should use a small amount of hydrogen peroxide on a cotton ball to swab the ear.”

ANS: B

Never place a drug into the ear canal or irrigate the ear canal if there is drainage present because it
could enter the middle ear and cause an infection. The healthcare provider should be notified.

DIF: Cognitive Level: Applying REF: p. 348

15. A child presents to the clinic with pain and inflammation in his right ear canal. He is
diagnosed with an ear infection. There is no drainage present and the tympanic membrane is
intact. List the appropriate drug route for treating this condition.

a. Oral route

b. Intradermal route

c. Topical route

d. Ophthalmic route

e. Intramuscular route

f. Subcutaneous
Authentic Nurse route
Guru: For more of these materials :masterclasstutor1@gmail.com
Visovsky: Introduction to Clinical Pharmacology, 10th Edition 23

ANS: C

Since the external ear can be reached from the outside, infections and inflammation of the pinna
and the ear canal are most often managed by topical drug application.

DIF: Cognitive Level: Remembering REF: p. 347

16. The LPN/LVN is preparing to give eye drops and eye ointment to a 75-year-old patient with
glaucoma. Which of the following are important for the LPN/LVN to remember about ophthalmic
drug therapy?

The LPN/LVN should place the eye drop in the ------------- of the eye and should apply pressure to
the ------------ of the eye to decrease systemic absorption of the drug. The LPN/LVN should wait a
minimum of in between the 2 eye drops.

a. lower eyelid; inner corner; 5 minutes

b. upper eyelid; outer corner; 1 minute

c. middle of the eye; inner corner; 1 hour

d. inner corner; middle of the eye; 10 minutes

ANS: A

The proper procedure for instilling an ophthalmic eye drop is to gently squeeze the bottle and
release the drops into the pocket of the lower eyelid. After giving the drops, to decrease systemic
absorption, the LPN/LVN can place the index finger over the tear duct in the inner corner of the
eye for about 3 minutes.

DIF: Cognitive Level: Applying REF: p. 350

17. A 75-year-old male patient presents to the clinic and tells you that he’s having a hard time
seeing out the sides of his eyes and is also noticing halos around lights.

The LPN/LVN knows that is the most likely diagnosis for this patient. The main problem that
accompanies this diagnosis is .

a. macular degeneration; decreased intraocular pressure

Authentic Nurse Guru: For more of these materials :masterclasstutor1@gmail.com


Visovsky: Introduction to Clinical Pharmacology, 10th Edition 23

b. glaucoma; increased intraocular pressure

c. conjunctivitis; cloudiness of the cornea

d. cataract; increased intracranial pressure

ANS: B

Glaucoma is a problem related to increased intraocular pressure that can put so much pressure on
ocular blood vessels, photoreceptors, and the optic nerve that blindness results. It does not affect
the lens, the cornea, or the intracranial pressure.

DIF: Cognitive Level: Understanding REF: p. 353

MULTIPLE RESPONSE

1. Which drugs for glaucoma cause marked mydriasis? (Select all that apply.)

a. Apraclonidine (Iopidine)

b. Bimatoprost (Lumigan)

c. Pilocarpine (Isopto Carpine, Pilopine)

d. Carbachol (Carboptic)

e. Carteolol (Ocupress)

f. Dipivefrin (AK-Pro)

g. Levobunolol (Betagan)

ANS: A, F

Apraclonidine and dipivefrin are adrenergic agonists that cause pupillary dilation (mydriasis).
This action allows aqueous humor to flow more freely through the pupil and be absorbed. The
others either do not affect pupillary size to any degree or cause pupillary constriction.

DIF: Cognitive Level: Remembering REF: p. 358

Authentic Nurse Guru: For more of these materials :masterclasstutor1@gmail.com


Visovsky: Introduction to Clinical Pharmacology, 10th Edition 23

Chapter 20: Over-the-Counter Drug Therapy

Visovsky: Introduction to Clinical Pharmacology, 10th Edition

MULTIPLE CHOICE

1. You are teaching a patient about taking over-the-counter (OTC) drugs. Which important safety
information should you include in your teaching plan?

a. “OTC drugs will only maintain their potency months after the expiration date.”

b. “OTC drugs often interact with other drugs, and with food or alcohol.”

c. “Be sure to take all the pills in the bottle.”

d. “Adjust the dosage to your specific needs.”

ANS: B

OTC drugs often interact with other drugs, and with food or alcohol, or they might affect other
existing health problems the patient has. Ask a pharmacist or the healthcare provider if you are
unsure. Also, the healthcare professional must be aware of all over-the-counter drugs being taken
because of the dangers of interaction.

DIF: Cognitive Level: Applying REF: p. 365

2. A patient informs you that she is buying over-the-counter (OTC) drugs online from another
country because they are cheaper. What is your best response?

a. “I agree with you, it is important to save money on expensive drugs when you can.”

b. “There is no difference between generic OTC drugs from different countries.”

c. “Avoid buying OTC drugs online, as they may be counterfeit or dangerous.”

d. “Only buy OTC drugs online if your insurance company covers them.”

ANS: C

Patients should avoid buying these products online, outside of well-known internet insurance
company sites, because many OTC preparations sold through the internet are counterfeit products
Authentic Nurse Guru: For more of these materials :masterclasstutor1@gmail.com
Visovsky: Introduction to Clinical Pharmacology, 10th Edition 23

and may be dangerous.

DIF: Cognitive Level: Applying REF: p. 365

3. What information is important to include when teaching parents about giving over-the-counter
(OTC) drugs to children?

a. Do not give drugs containing alcohol to children.

b. OTC drugs are safe for children younger than 2 years of age.

c. When giving OTC drugs to a child, always give one-half of the adult dose.

d. Refrain from using a child-resistant cap in the event the drug must be given quickly.

ANS: A

One of the most important considerations in reading OTC product labels is to notice the hidden
chemicals (used for various purposes) in the products. Drugs containing alcohol (such as some
cough preparations) should not be given to children.

DIF: Cognitive Level: Applying REF: p. 366

4. Which of the following drug types is not regulated by the Food and Drug Administration?

a. Opioids

b. Vitamins

c. Hormones

d. Herbal supplements

ANS: D

The FDA does not regulate herbal supplements but protects consumers from misleading health
claims by the supplement industry.

DIF: Cognitive Level: Remembering REF: p. 369

5. During your assessment of a clinic patient, the patient asks you if using expired over-the-
counter (OTC)
Authentic Nursedrugs would
Guru: Forcause
moreharm.
of What
theseis materials
your best answer?
:masterclasstutor1@gmail.com
Visovsky: Introduction to Clinical Pharmacology, 10th Edition 23

a. “These drugs are safer than prescription drugs, so using expired OTC drugs is harmless.”

b. “Disposing of expired drugs is only necessary if you have several drug allergies.”

c. “You should always safely dispose all expired or discolored drugs.”

d. “The chemical preservatives in OTC drugs prevent drug expiration.”

ANS: C

You should always tell patients to safely dispose of old, discolored, or expired drugs or any drug
that has lost its label instructions. Your pharmacist can tell you about the ways to safely dispose
of drugs.

DIF: Cognitive Level: Applying REF: p. 365

6. A patient with depression tells you she is using St. John’s wort to treat her symptoms because
she would rather use a natural product, as she already takes many prescription drugs. What is your
best response?

a. “St. John’s wort has been proven effective in treating mild to moderate depression.”

b. “You may take St. John’s wort with your other prescription drugs without concern.”

c. “Avoid taking St. John’s wort because it interferes with many prescription drugs.”

d. “Avoid using St. John’s wort unless you have weekly blood levels drawn.”

ANS: C

St. John’s wort may improve mild to moderate depression but has potentially dangerous
interactions with many prescription drugs.

DIF: Cognitive
Authentic NurseLevel: Applying
Guru: REF:of
For more p. these
368 materials :masterclasstutor1@gmail.com
Visovsky: Introduction to Clinical Pharmacology, 10th Edition 23

7. Which statement about multivitamins is true?

a. Most people will take multivitamins without consulting a healthcare professional.

b. Many multivitamins are used with prescription drugs to prevent disease.

c. Multivitamins are drug preparations that do not expire.

d. Multivitamins do not pose a risk for adverse effects.

ANS: A

People take vitamins to maintain health or to correct specific nutritional deficiencies. Most people
decided to take vitamins on their own without the advice of a healthcare professional.

DIF: Cognitive Level: Remembering REF: p. 369

8. A patient with a severe upper respiratory infection tells you she is taking mega-doses of vitamin
C to help her get rid of the infection quicker. What is your best response?

a. “Taking high doses of vitamin C is recommended for this purpose.”

b. “Taking high doses of vitamin C is not effective because it is excreted quickly.”

c. “Vitamin C is a fat-soluble vitamin, and high doses can be dangerous.”

d. “Vitamin C is only useful in treating malabsorption syndromes.”

ANS: B

Vitamin C is a water-soluble vitamin that is quickly excreted and thus mega-doses of this vitamin
produce no additional benefit.

DIF: Cognitive Level: Applying REF: p. 373

9. A patient in the clinic asks you if she should purchase an expensive brand name multivitamin
because cheaper vitamins may not be as good. What is your best response to this patient?

a. “All over-the-counter multivitamin preparations are the same.”

b. “There is a price–quality relationship to the vitamin product you buy.”

Authentic Nurse Guru: For more of these materials :masterclasstutor1@gmail.com


Visovsky: Introduction to Clinical Pharmacology, 10th Edition 23

c. “The cost of the vitamin preparation is not an indication of effectiveness.”

d. “The cost of multivitamins is related to the bioavailability of the vitamins in the formula.”

ANS: C

The costs for some products are high because of the claims made about their effectiveness, but
few of the manufacturer claims can be substantiated by evidence.

DIF: Cognitive Level: Applying REF: p. 371

10. A newly pregnant patient is taking oral vitamin A to encourage embryo growth. What would
you include in your teaching plan for this patient?

a. “Vitamin A is a water-soluble vitamin, so it is safe for the fetus.”

b. “Take only the amount prescribed; high doses can cause birth defects.”

c. “Taking vitamin A during the first 12 weeks of pregnancy is recommended.”

d. “Only take vitamin A in the last trimester of pregnancy to encourage fetal growth.”

ANS: B

Excess vitamin A ingestion during pregnancy can cause birth defects of the CNS.

DIF: Cognitive Level: Applying REF: p. 371

11. A patient has been admitted for suspected vitamin A overdose. What symptoms would you
expect the patient to exhibit?

a. Diarrhea and fever

b. Constipation and changes in taste

c. Ecchymosis of the lower extremities

d. Yellowing of the skin on the nose and ears

ANS: D

An early indication of vitamin A overdose is anorexia, abdominal pain, malaise, and yellowing of
Authentic Nurse Guru: For more of these materials :masterclasstutor1@gmail.com
Visovsky: Introduction to Clinical Pharmacology, 10th Edition 23

the skin, especially on the nose and ears.

DIF: Cognitive Level: Applying REF: p. 371

12. You are getting ready to give an intravenous preparation containing vitamin B1 (thiamine) to
a patient with alcoholism. What is the rationale for giving thiamine in an IV solution containing
glucose?

a. Glucose is necessary to assist thiamine in entering the cells.

b. Glucose prevents the thiamine from being excreted.

c. Glucose prevents precipitation of the IV fluid.

d. Glucose is needed for thiamine absorption.

ANS: C

If giving thiamine to an alcoholic or thiamine deficient patient, intravenous glucose should also be
given to prevent precipitation or worsening of Wernicke encephalopathy.

DIF: Cognitive Level: Remembering REF: p. 371

13. You are teaching a patient with a dietary deficiency of riboflavin about foods that contain
riboflavin that should be part of the patient’s diet. Which of the following foods would be
recommended?

a. Enriched flour

b. Dairy products

c. Whole grains such as barley

d. Orange-colored vegetables such as carrots

ANS: B

Teach patients that common food sources of riboflavin include dairy products, eggs, green leafy
vegetables, organ meats, and peanuts.
Authentic Nurse Guru: For more of these materials :masterclasstutor1@gmail.com
Visovsky: Introduction to Clinical Pharmacology, 10th Edition 24

DIF: Cognitive Level: Applying REF: p. 372

14. A patient you are caring for has been started on niacin as part of a regimen to reduce
hyperlipidemia. Which unpleasant side effects of niacin should you teach this patient to expect?

a. Skin warmth and flushing

b. Yellow-tinged skin and sclera

c. Beefy-red tongue

d. Watery eyes

ANS: A

Expected side effects of niacin supplementation are skin warmth, flushing, and itching that can be
relieved when giving niacin with aspirin.

DIF: Cognitive Level: Applying REF: p. 372

15. Which drug category is associated with pyridoxine deficiency?

a. Antibiotics

b. Beta blockers

c. Antilipidemics

d. Oral contraceptives

ANS: D

Oral contraceptives may induce pyridoxine deficiency.

DIF: Cognitive Level: Remembering REF: p. 372

16. A newly pregnant woman placed on folic acid states she does not want to take any drugs
during her pregnancy. What is your best response?

a. “Folic acid is crucial to the development of the sex organs of the fetus.”

b. “Folic acid is crucial to the development of the cardiovascular system.”


Authentic Nurse Guru: For more of these materials :masterclasstutor1@gmail.com
Visovsky: Introduction to Clinical Pharmacology, 10th Edition 24

c. “Folic acid is given to prevent neural tube defects of the spinal cord.”

d. “Folic acid will prevent nausea and vomiting during pregnancy.”

ANS: C

The US Public Health Service has issued an official recommendation that all women of
childbearing age in the United States who are pregnant should consume folic acid for reducing
their risk of having a pregnancy affected with spina bifida or other neural tube defects.

DIF: Cognitive Level: Applying REF: p. 372

17. You are evaluating a patient’s understanding of a newly prescribed drug regimen for the
prevention of heart disease. Which response made by the patient demonstrates understanding of
the drug regimen?

a. “I will take folic acid every other day.”

b. “I will take calcium supplements daily with meals.”

c. “I will take a daily dose of folic acid and vitamins B6 and B12.”

d. “I will take a quality multivitamin and additional ferrous sulfate daily.”

ANS: C

Folic acid and vitamins B6 and B12 have been shown to reduce homocysteine levels. Modestly
elevated homocysteine levels in the blood are a risk factor for heart disease.

DIF: Cognitive Level: Applying REF: p. 372

18. Which patient would be most likely to develop a vitamin B12 deficiency?

a. A patient who takes ibuprofen daily for arthritis

b. A patient who smokes cigarettes daily

c. A patient who is on a vegetarian diet

d. A patient with sickle cell anemia

ANS: C Nurse Guru: For more of these materials :masterclasstutor1@gmail.com


Authentic
Visovsky: Introduction to Clinical Pharmacology, 10th Edition 24

A patient who is on a strict vegetarian diet may develop vitamin B12 deficiency because vitamin
B12 is found only in animal products.

DIF: Cognitive Level: Remembering REF: p. 373

19. A patient with multiple vitamin deficiencies is prescribed oral vitamin C as part of a vitamin
supplementation treatment. Which of the following instructions regarding vitamin C should be
given to this patient?

a. “Keep vitamin C tablets away from heat and light sources.”

b. “Only take vitamin C when you are unable to eat citrus fruits.”

c. “There are no interactions of vitamin C with other drugs you may be taking.”

d. “Vitamin C is a fat-soluble vitamin primarily used to treat blood clotting disorders.”

ANS: A

Vitamin C is easily destroyed by air, heat, and light. This drug should be kept tightly capped in its
own container.

DIF: Cognitive Level: Applying REF: p. 374

20. You are caring for a patient who has a family history of cancer. The patient reports taking
vitamin E daily for cancer prevention.

What is your best response?

a. “Recent evidence shows that vitamin E is safe and effective for cancer prevention.”

b. “You may take vitamin E for cancer prevention as long as it is the prescription form.”

c. “There is currently no evidence that taking vitamin E will prevent cancer.”

d. “Vitamin E is only useful in the prevention of heart disease.”

ANS: C

Many suggested uses of vitamin E are controversial and unproved. New evidence suggests that
vitamin E supplements do not reduce the risk of cancer or major cardiovascular disease and may
Authentic Nurse Guru: For more of these materials :masterclasstutor1@gmail.com
Visovsky: Introduction to Clinical Pharmacology, 10th Edition 24

even increase the risk of heart failure.

DIF: Cognitive Level: Applying REF: p. 375

21. Which drug over-dosage would you expect vitamin K to be given as an antidote?

a. Warfarin

b. Heparin

c. Pyridoxine

d. Ascorbic acid

ANS: A

Vitamin K is the antidote for warfarin (Coumadin) overdose.

DIF: Cognitive Level: Remembering REF: p. 375

22. For which condition would you expect the healthcare provider to prescribe iron
supplementation?

a. Esophageal reflux

b. Constipation

c. Osteoporosis

d. Anemia

ANS: D

Iron is necessary for people with anemia caused by blood loss secondary to surgery or other
conditions resulting in iron-deficiency anemia.

DIF: Cognitive Level: Remembering REF: p. 376

23. A patient with a chronic malabsorption syndrome has been admitted to the hospital with a
severe magnesium deficiency. What complication of magnesium deficiency should you be alert
for?

Authentic Nurse Guru: For more of these materials :masterclasstutor1@gmail.com


Visovsky: Introduction to Clinical Pharmacology, 10th Edition 24

a. Ventricular tachycardia

b. Vasoconstriction

c. Severe diarrhea

d. Pancreatitis

ANS: A

Magnesium is important in cardiac function, and only slight deficiencies may prolong the Q-T
interval and lead to a very dangerous form of ventricular tachycardia (rapid heartbeat) called
torsades de pointes.

DIF: Cognitive Level: Remembering REF: p. 377

MULTIPLE RESPONSE

1. You are seeing a patient in the clinic who is just started using CBD. You are preparing patient
education regarding the side effects of CBD. Which of the following are common side effects of
cannabis? (Select all that apply.)

a. Cough

b. Red eyes

c. Tinnitus

d. Altered vision

e. Sedation

f. Altered sense of taste

ANS: A, B, D, E

Some common side effects of cannabis are dry mouth, sedation, cough, dysphoria, anxiety (with
higher doses), red eyes, altered visual perceptions, lack of coordination, and altered sense of time.

DIF: Cognitive Level: Applying REF: p. 368

2. You are
Authentic caringGuru:
Nurse for a patient in a long-term
For more carematerials
of these facility and :masterclasstutor1@gmail.com
you notice that he has symptoms
Visovsky: Introduction to Clinical Pharmacology, 10th Edition 24

that are consistent with a suspected vitamin B2 (riboflavin) deficiency. List the symptoms you
would expect to see in a patient diagnosed with a vitamin B2 deficiency. (Select all that apply.)

a. Low blood counts

b. Upper quadrant abdominal pain

c. Sore throat

d. Cracks in the corners of the mouth

e. Confusion and short-term memory loss

f. Burning of the tongue and lips

ANS: C, D, F

Symptoms of riboflavin deficiency may include cracks in the corner of the mouth, soreness and
burning of the tongue and lips, and sore throat.

DIF: Cognitive Level: Applying REF: p. 371

3. You are planning patient education for a 65-year-old patient diagnosed with a vitamin B6
deficiency. You know it is important for this patient to include eating foods high in vitamin B6
(pyridoxine) in his regular diet. List the appropriate foods that this patient should include in his
diet. (Select all that apply.)

a. Liver

b. Orange juice

c. Eggs

d. Whole grain bread

e. Soybeans

f. Bananas

ANS: A,Nurse
Authentic C, D, E Guru: For more of these materials :masterclasstutor1@gmail.com
Visovsky: Introduction to Clinical Pharmacology, 10th Edition 24

Food sources of vitamin B6 include yeast, wheat, corn, egg yolk, liver, kidney, and muscle meats,
soybeans, cereals, whole grain bread, and soybeans. Limited amounts are available from milk and
vegetables.

DIF: Cognitive Level: Applying REF: p. 372

4. You are taking a diet history from a patient who has a vitamin B12 deficiency from pernicious
anemia. Which of the following foods demonstrates understanding of an appropriate dietary
choice for this patient? (Select all that apply.)

a. Fermented cheeses such as Camembert

b. Organ meats, such as liver or kidneys

c. Shellfish, such as clams or scallops

d. Whole grain foods such as barley

e. Leafy greens, such as spinach

f. Nonfat milk

ANS: A, B, C, F

Food sources of B12 include organ meats, clams and oysters; nonfat dry milk; fermented cheese
such as Camembert and Limburger; and seafood such as lobster, scallops, flounder, haddock,
swordfish, and tuna.

DIF: Cognitive Level: Remembering REF: p. 373

5. Which of the following represent the effects of calcium on the body? (Select all that apply.)

a. Large doses of calcium will not harm the body when taken on a full stomach.

b. Calcium can limit the absorption of iron and other trace elements.
Authentic Nurse Guru: For more of these materials :masterclasstutor1@gmail.com
Visovsky: Introduction to Clinical Pharmacology, 10th Edition 24

c. Calcium can cause constipation and reduce kidney function.

d. Calcium may prevent bone loss in older men.

e. Calcium can reduce kidney function.

ANS: B, C, D, E

Large doses of calcium can limit the absorption of iron and other trace elements. Calcium can
cause constipation and reduce kidney function. Older men, as well as older women, need calcium
in their diet to prevent bone loss.

DIF: Cognitive Level: Remembering REF: p. 376

6. You are collecting information on a new patient’s drug history, which includes the use of over-
the-counter (OTC) drugs. The patient asks you why this important. List the following statements
that are true regarding OTC drugs. (Select all that apply.)

a. “Assessment of OTC drugs isn’t necessary unless the patient has an acute event.”

b. “There may be unsafe interactions of OTC drugs with prescription drugs.”

c. “Patients often neglect to tell healthcare providers about OTC drugs routinely taken.”

d. “OTC drugs may worsen existing medical problems.”

e. “The insurance company needs to determine the monthly costs of all drugs taken.”

f. “OTC preparations have been proven safe for infants and children.”

ANS: B, C, D

Nurses should always assess the patient’s use of OTC products, as many patients do not disclose
the use of OTC products to healthcare providers. Some of the active chemicals in OTC products
may make existing medical problems worse or interact with a patient’s prescribed drugs.

DIF: Cognitive Level: Applying REF: p. 364

7. What patient teaching points should be included in the care of a patient taking iron for the
treatment of iron deficiency anemia? (Select all that apply.)

Authentic Nurse Guru: For more of these materials :masterclasstutor1@gmail.com


Visovsky: Introduction to Clinical Pharmacology, 10th Edition 24

a. Liquid iron solutions can discolor teeth.

b. Take the iron tablets on an empty stomach.

c. Expect to take iron for a period of 4 to 6 months.

d. Do not eat foods high in iron while on iron supplements.

e. Calcium should be given along with iron for maximal absorption.

f. Iron supplements can cause the stool to appear dark green or black in color.

ANS: A, B, C, F

Patients taking iron should be instructed that liquid iron solutions can discolor teeth. They should
take the iron tablets on an empty stomach. Most iron regimens last for 4 to 6 months. Iron
supplements can cause the stool to appear dark green or black in color.

DIF: Cognitive Level: Applying REF: p. 376

8. A patient taking a diuretic for congestive heart failure (CHF) asks the nurse you to suggest
potassium-rich foods that can be eaten to supplement potassium loss from the diuretic. List the
following foods that should be included in this patient’s teaching plan. (Select all that apply.)

a. Dried fruits

b. Citrus fruits

c. Rice and pasta

d. Nuts

e. Cantaloupe and watermelon

f. Cruciferous vegetables

g. Apples

h. Green leafy vegetables

ANS: A, B, D, E, F, H

Authentic Nurse Guru: For more of these materials :masterclasstutor1@gmail.com


Visovsky: Introduction to Clinical Pharmacology, 10th Edition 24

Green leafy vegetables, such as spinach, bok choy, and Swiss chard, are some of the best sources
of potassium. A potassium-rich diet includes foods such as bananas, citrus fruits (especially
tomatoes and oranges), apricots, and dried fruits such as raisins, prunes, and dates. Fresh
cantaloupe and watermelon, nuts, dried beans, beef, and fowl also contain ample quantities of
potassium.

DIF: Cognitive Level: Remembering REF: p. 378

9. You are taking care of a patient and preparing to give thiamine IV. You know that it is
important to check the patient continuously throughout the infusion for signs of an allergic
reaction. List the following indications that would alert the LPN/LVN of an allergic reaction.
(Select all that apply.)

a. Headache

b. Dyspnea

c. Angioedema

d. Cyanosis

e. Joint pain

f. Pruritus

ANS: B, C, D, F

When giving parenteral thiamine, the patient should be continuously assessed for indications of an
allergic reaction, such as feelings of warmth, pruritus, urticaria, angioedema, dyspnea, sweating,
tightness of the throat, and cyanosis.

DIF: Cognitive Level: Understanding REF: p. 371

Authentic Nurse Guru: For more of these materials :masterclasstutor1@gmail.com

You might also like